[obm-l] Re: [obm-l] Re: [obm-l] Re: [obm-l] Re: [obm-l] Re: [obm-l] Ajuda em combinatória! Probleminha fuleiro!

2024-08-07 Por tôpico Daniel Jelin
Eu cheguei em 616. Assim:

Vamos primeiro contar os diferentes quadradões, sem considerar as
colorações repetidas por rotação

C8,2 (escolhe 2 cores) * C2,1 (escolhe 1 cor pra diagonal principal) = 56
C8,3 (escolhe 3 cores) * C3,1 (escolhe 1 delas pra repetir) * C2,1 (escolhe
a diagonal que terá cor repetida) * P2 (ordena as 2 cores restantes) = 672
C8,4 (escolhe 4 cores) * P4 (ordena as cores escolhidas) = 1680

Agora vamos considerar as rotações que geram quadradões de mesma coloração:

2 cores: uma rotação transforma cada quadradão em outro quadradão de mesma
coloração, 56/2 = 28 colorações
3 cores: três rotações transformam cada quadradão em outros três quadradões
de mesma coloração, 672/4 = 168 colorações
4 cores: três rotações de 90 graus transformam cada quadradão em outros
três quadradões de mesma coloração, 1680/4 = 420 colorações

Somando, 616.

On Wed, 7 Aug 2024 at 13:07, Anderson Torres 
wrote:

> Em qua., 7 de ago. de 2024 às 10:24, Armando Staib
>  escreveu:
> >
> > Em 1 diagonal eu fiz elas iguais  ou diferentes.
> > Qdo sao iguais 8*7*7*1/4
> > Qdo sao diferentes 8*7*6*6/4
> > Total 602
> >
> >
> > Em qua, 7 de ago de 2024 08:50, Prof. Douglas Oliveira <
> profdouglaso.del...@gmail.com> escreveu:
> >>
> >> A diferença do meu para o seu foi no segundo caso, em que considerei
> apenas 2 rotações.
> >>
> >> Em qua., 7 de ago. de 2024, 08:01, Marcelo Gonda Stangler <
> marcelo.gonda.stang...@gmail.com> escreveu:
> >>>
> >>> Dúvida o problema em diagonais!
>
> E se fosse um pentágono?
>
> >>>
> >>> Os casos em que a diagonal tem a mesma cor, e tem cores diferentes,
> são casos disjuntos que totalizam os casos totais, e caso ambas diagonais
> sejam iguais (dentro de seu par), só podemos ter 2 rotações, e se não
> sempre poderemos ter 4 rotações. Segue o desenvolvimento
> >>>
> >>> 2 diagonais iguais:
> >>>
> >>> 8*7 colorações
> >>>
> >>> 2 rotações
> >>>
> >>> 28 no total
> >>>
> >>> 1 diagonal igual:
> >>>
> >>> 8*7*6 colorações
> >>>
> >>> 4 rotações
> >>>
> >>> 84 no total
> >>>
> >>> 0 diagonais iguais:
> >>>
> >>> 8*7*6*5 colorações
> >>>
> >>> 4 rotações
> >>>
> >>> 420 no total
> >>>
> >>>
> >>> Total: 532
> >>>
> >>>
> >>>
> >>> Em ter., 6 de ago. de 2024 6:35 PM, Prof. Douglas Oliveira <
> profdouglaso.del...@gmail.com> escreveu:
> 
>  Olá amigos, estou bem curioso com o seguinte probleminha que
> encontrei na lista do POTI do Carlos Shine de combinatória, onde não sei se
> esqueci algum caso e encontrei 616 (acho), a resposta do Shine é 1044 e
> coloquei no chat gpt (rs) e ele falou a respeito de um tal de Burnside e
> encontrou 903. kk
> 
>  Preciso de uma ajudinha!
> 
>  Eis o problema:
> 
>  Exemplo 7. Mariana tem tinta guache de 8 cores diferentes e quer
> pintar os quatro quadradinhos unitários de um quadrado de lado 2 de modo
> que casas que têm um lado comum tenham cores diferentes. De quantas
> maneiras ela pode fazer isso? (Duas colorações são iguais se uma pode ser
> obtida a partir de outra através de uma rotação.)
> 
> 
>  Att:
> 
>  Douglas Oliveira
> 
>  --
>  Esta mensagem foi verificada pelo sistema de antivírus e
>  acredita-se estar livre de perigo.
> >>>
> >>>
> >>> --
> >>> Esta mensagem foi verificada pelo sistema de antivírus e
> >>> acredita-se estar livre de perigo.
> >>
> >>
> >> --
> >> Esta mensagem foi verificada pelo sistema de antivírus e
> >> acredita-se estar livre de perigo.
> >
> >
> > --
> > Esta mensagem foi verificada pelo sistema de antivírus e
> > acredita-se estar livre de perigo.
>
> --
> Esta mensagem foi verificada pelo sistema de antivírus e
>  acredita-se estar livre de perigo.
>
>
> =
> Instru�ões para entrar na lista, sair da lista e usar a lista em
> http://www.mat.puc-rio.br/~obmlistas/obm-l.html
> =
>

-- 
Esta mensagem foi verificada pelo sistema de antiv�rus e
 acredita-se estar livre de perigo.



[obm-l] Re: [obm-l] Re: [obm-l] Re: [obm-l] Re: [obm-l] Ajuda em combinatória! Probleminha fuleiro!

2024-08-07 Por tôpico Anderson Torres
Em qua., 7 de ago. de 2024 às 10:24, Armando Staib
 escreveu:
>
> Em 1 diagonal eu fiz elas iguais  ou diferentes.
> Qdo sao iguais 8*7*7*1/4
> Qdo sao diferentes 8*7*6*6/4
> Total 602
>
>
> Em qua, 7 de ago de 2024 08:50, Prof. Douglas Oliveira 
>  escreveu:
>>
>> A diferença do meu para o seu foi no segundo caso, em que considerei apenas 
>> 2 rotações.
>>
>> Em qua., 7 de ago. de 2024, 08:01, Marcelo Gonda Stangler 
>>  escreveu:
>>>
>>> Dúvida o problema em diagonais!

E se fosse um pentágono?

>>>
>>> Os casos em que a diagonal tem a mesma cor, e tem cores diferentes, são 
>>> casos disjuntos que totalizam os casos totais, e caso ambas diagonais sejam 
>>> iguais (dentro de seu par), só podemos ter 2 rotações, e se não sempre 
>>> poderemos ter 4 rotações. Segue o desenvolvimento
>>>
>>> 2 diagonais iguais:
>>>
>>> 8*7 colorações
>>>
>>> 2 rotações
>>>
>>> 28 no total
>>>
>>> 1 diagonal igual:
>>>
>>> 8*7*6 colorações
>>>
>>> 4 rotações
>>>
>>> 84 no total
>>>
>>> 0 diagonais iguais:
>>>
>>> 8*7*6*5 colorações
>>>
>>> 4 rotações
>>>
>>> 420 no total
>>>
>>>
>>> Total: 532
>>>
>>>
>>>
>>> Em ter., 6 de ago. de 2024 6:35 PM, Prof. Douglas Oliveira 
>>>  escreveu:

 Olá amigos, estou bem curioso com o seguinte probleminha que encontrei na 
 lista do POTI do Carlos Shine de combinatória, onde não sei se esqueci 
 algum caso e encontrei 616 (acho), a resposta do Shine é 1044 e coloquei 
 no chat gpt (rs) e ele falou a respeito de um tal de Burnside e encontrou 
 903. kk

 Preciso de uma ajudinha!

 Eis o problema:

 Exemplo 7. Mariana tem tinta guache de 8 cores diferentes e quer pintar os 
 quatro quadradinhos unitários de um quadrado de lado 2 de modo que casas 
 que têm um lado comum tenham cores diferentes. De quantas maneiras ela 
 pode fazer isso? (Duas colorações são iguais se uma pode ser obtida a 
 partir de outra através de uma rotação.)


 Att:

 Douglas Oliveira

 --
 Esta mensagem foi verificada pelo sistema de antivírus e
 acredita-se estar livre de perigo.
>>>
>>>
>>> --
>>> Esta mensagem foi verificada pelo sistema de antivírus e
>>> acredita-se estar livre de perigo.
>>
>>
>> --
>> Esta mensagem foi verificada pelo sistema de antivírus e
>> acredita-se estar livre de perigo.
>
>
> --
> Esta mensagem foi verificada pelo sistema de antivírus e
> acredita-se estar livre de perigo.

-- 
Esta mensagem foi verificada pelo sistema de antiv�rus e
 acredita-se estar livre de perigo.


=
Instru��es para entrar na lista, sair da lista e usar a lista em
http://www.mat.puc-rio.br/~obmlistas/obm-l.html
=


[obm-l] Re: [obm-l] Re: [obm-l] Re: [obm-l] Ajuda em combinatória! Probleminha fuleiro!

2024-08-07 Por tôpico Armando Staib
Em 1 diagonal eu fiz elas iguais  ou diferentes.
Qdo sao iguais 8*7*7*1/4
Qdo sao diferentes 8*7*6*6/4
Total 602

Em qua, 7 de ago de 2024 08:50, Prof. Douglas Oliveira <
profdouglaso.del...@gmail.com> escreveu:

> A diferença do meu para o seu foi no segundo caso, em que considerei
> apenas 2 rotações.
>
> Em qua., 7 de ago. de 2024, 08:01, Marcelo Gonda Stangler <
> marcelo.gonda.stang...@gmail.com> escreveu:
>
>> Dúvida o problema em diagonais!
>>
>> Os casos em que a diagonal tem a mesma cor, e tem cores diferentes, são
>> casos disjuntos que totalizam os casos totais, e caso ambas diagonais sejam
>> iguais (dentro de seu par), só podemos ter 2 rotações, e se não sempre
>> poderemos ter 4 rotações. Segue o desenvolvimento
>>
>> 2 diagonais iguais:
>>
>> 8*7 colorações
>>
>> 2 rotações
>>
>> 28 no total
>>
>> 1 diagonal igual:
>>
>> 8*7*6 colorações
>>
>> 4 rotações
>>
>> 84 no total
>>
>> 0 diagonais iguais:
>>
>> 8*7*6*5 colorações
>>
>> 4 rotações
>>
>> 420 no total
>>
>>
>> Total: 532
>>
>>
>> Em ter., 6 de ago. de 2024 6:35 PM, Prof. Douglas Oliveira <
>> profdouglaso.del...@gmail.com> escreveu:
>>
>>> Olá amigos, estou bem curioso com o seguinte probleminha que encontrei
>>> na lista do POTI do Carlos Shine de combinatória, onde não sei se esqueci
>>> algum caso e encontrei 616 (acho), a resposta do Shine é 1044 e coloquei no
>>> chat gpt (rs) e ele falou a respeito de um tal de Burnside e encontrou 903.
>>> kk
>>>
>>> Preciso de uma ajudinha!
>>>
>>> Eis o problema:
>>>
>>> Exemplo 7. Mariana tem tinta guache de 8 cores diferentes e quer pintar
>>> os quatro quadradinhos unitários de um quadrado de lado 2 de modo que casas
>>> que têm um lado comum tenham cores diferentes. De quantas maneiras ela pode
>>> fazer isso? (Duas colorações são iguais se uma pode ser obtida a partir de
>>> outra através de uma rotação.)
>>>
>>>
>>> Att:
>>>
>>> Douglas Oliveira
>>>
>>> --
>>> Esta mensagem foi verificada pelo sistema de antivírus e
>>> acredita-se estar livre de perigo.
>>
>>
>> --
>> Esta mensagem foi verificada pelo sistema de antivírus e
>> acredita-se estar livre de perigo.
>
>
> --
> Esta mensagem foi verificada pelo sistema de antivírus e
> acredita-se estar livre de perigo.

-- 
Esta mensagem foi verificada pelo sistema de antiv�rus e
 acredita-se estar livre de perigo.



[obm-l] Re: [obm-l] Re: [obm-l] Ajuda em combinatória! Probleminha fuleiro!

2024-08-07 Por tôpico Prof. Douglas Oliveira
A diferença do meu para o seu foi no segundo caso, em que considerei apenas
2 rotações.

Em qua., 7 de ago. de 2024, 08:01, Marcelo Gonda Stangler <
marcelo.gonda.stang...@gmail.com> escreveu:

> Dúvida o problema em diagonais!
>
> Os casos em que a diagonal tem a mesma cor, e tem cores diferentes, são
> casos disjuntos que totalizam os casos totais, e caso ambas diagonais sejam
> iguais (dentro de seu par), só podemos ter 2 rotações, e se não sempre
> poderemos ter 4 rotações. Segue o desenvolvimento
>
> 2 diagonais iguais:
>
> 8*7 colorações
>
> 2 rotações
>
> 28 no total
>
> 1 diagonal igual:
>
> 8*7*6 colorações
>
> 4 rotações
>
> 84 no total
>
> 0 diagonais iguais:
>
> 8*7*6*5 colorações
>
> 4 rotações
>
> 420 no total
>
>
> Total: 532
>
>
> Em ter., 6 de ago. de 2024 6:35 PM, Prof. Douglas Oliveira <
> profdouglaso.del...@gmail.com> escreveu:
>
>> Olá amigos, estou bem curioso com o seguinte probleminha que encontrei na
>> lista do POTI do Carlos Shine de combinatória, onde não sei se esqueci
>> algum caso e encontrei 616 (acho), a resposta do Shine é 1044 e coloquei no
>> chat gpt (rs) e ele falou a respeito de um tal de Burnside e encontrou 903.
>> kk
>>
>> Preciso de uma ajudinha!
>>
>> Eis o problema:
>>
>> Exemplo 7. Mariana tem tinta guache de 8 cores diferentes e quer pintar
>> os quatro quadradinhos unitários de um quadrado de lado 2 de modo que casas
>> que têm um lado comum tenham cores diferentes. De quantas maneiras ela pode
>> fazer isso? (Duas colorações são iguais se uma pode ser obtida a partir de
>> outra através de uma rotação.)
>>
>>
>> Att:
>>
>> Douglas Oliveira
>>
>> --
>> Esta mensagem foi verificada pelo sistema de antivírus e
>> acredita-se estar livre de perigo.
>
>
> --
> Esta mensagem foi verificada pelo sistema de antivírus e
> acredita-se estar livre de perigo.

-- 
Esta mensagem foi verificada pelo sistema de antiv�rus e
 acredita-se estar livre de perigo.



[obm-l] Re: [obm-l] Ajuda em combinatória! Probleminha fuleiro!

2024-08-07 Por tôpico Marcelo Gonda Stangler
Dúvida o problema em diagonais!

Os casos em que a diagonal tem a mesma cor, e tem cores diferentes, são
casos disjuntos que totalizam os casos totais, e caso ambas diagonais sejam
iguais (dentro de seu par), só podemos ter 2 rotações, e se não sempre
poderemos ter 4 rotações. Segue o desenvolvimento

2 diagonais iguais:

8*7 colorações

2 rotações

28 no total

1 diagonal igual:

8*7*6 colorações

4 rotações

84 no total

0 diagonais iguais:

8*7*6*5 colorações

4 rotações

420 no total


Total: 532


Em ter., 6 de ago. de 2024 6:35 PM, Prof. Douglas Oliveira <
profdouglaso.del...@gmail.com> escreveu:

> Olá amigos, estou bem curioso com o seguinte probleminha que encontrei na
> lista do POTI do Carlos Shine de combinatória, onde não sei se esqueci
> algum caso e encontrei 616 (acho), a resposta do Shine é 1044 e coloquei no
> chat gpt (rs) e ele falou a respeito de um tal de Burnside e encontrou 903.
> kk
>
> Preciso de uma ajudinha!
>
> Eis o problema:
>
> Exemplo 7. Mariana tem tinta guache de 8 cores diferentes e quer pintar os
> quatro quadradinhos unitários de um quadrado de lado 2 de modo que casas
> que têm um lado comum tenham cores diferentes. De quantas maneiras ela pode
> fazer isso? (Duas colorações são iguais se uma pode ser obtida a partir de
> outra através de uma rotação.)
>
>
> Att:
>
> Douglas Oliveira
>
> --
> Esta mensagem foi verificada pelo sistema de antivírus e
> acredita-se estar livre de perigo.

-- 
Esta mensagem foi verificada pelo sistema de antiv�rus e
 acredita-se estar livre de perigo.



[obm-l] Re: [obm-l] [Dúvida]

2024-07-15 Por tôpico Joel Soares Moreira
Ops: a *intersecção entre P e {a-1, a+1}* só conterá a+1 no final.

Em seg., 15 de jul. de 2024 às 20:42, Joel Soares Moreira 
escreveu:

> Sabendo o seu próprio número, a, um matemático sabe que o número do outro
> matemático pode ser ou a-1 ou a+1. Ele sabe com certeza o número do outro
> matemático se a intersecção entre {a-1, a+1} e o conjunto de potenciais
> valores do outro matemático, "P", só tem um elemento (i.e. ele excluiu ou
> a-1 ou a+1 dos valores do outro matemático por alguma razão).
>
> Inicialmente, P é todos os naturais. Se algum dos dois matemáticos possui
> 0 (i.e. o menor natural), eles sabem que o oponente possui 1. Assim, depois
> de uma rodada sem um sim, podemos excluir 0 de P.
>
> Numa dada rodada de perguntas, seja n o menor natural em P. Se alguim dos
> dois possui n, então eles sabem que o oponente possui n+1 (pois n-1 < n,
> portanto n-1 não pertence a P). Assim, depois dessa rodada, podemos excluir
> n de P.
>
> Assim, para qualquer valor de "a", teremos que P só contém "a+1" após
> "a-1" rodadas, e nesse momento o matemático que possui "a" saberá o valor
> do oponente.
>
> Em seg., 15 de jul. de 2024 às 19:55, Gilberto Azevedo <
> gil159...@gmail.com> escreveu:
>
>> Dois gênios matemáticos recebem dois numeros naturais consecutivos (eles
>> só sabem o próprio número e que são consecutivos, mas nao sabem quem é o
>> maior.)
>> Eles se alternam perguntando: vc ja sabe qual o meu número? E respondem
>> sinceramente.
>> Mostre que em algum momento algum dos dois diz sim.
>>
>> Como que responde isso formalmente? Não consigo terminar a linha de
>> raciocínio.
>>
>> --
>> Esta mensagem foi verificada pelo sistema de antivírus e
>> acredita-se estar livre de perigo.
>
>

-- 
Esta mensagem foi verificada pelo sistema de antiv�rus e
 acredita-se estar livre de perigo.



[obm-l] Re: [obm-l] [Dúvida]

2024-07-15 Por tôpico Joel Soares Moreira
Sabendo o seu próprio número, a, um matemático sabe que o número do outro
matemático pode ser ou a-1 ou a+1. Ele sabe com certeza o número do outro
matemático se a intersecção entre {a-1, a+1} e o conjunto de potenciais
valores do outro matemático, "P", só tem um elemento (i.e. ele excluiu ou
a-1 ou a+1 dos valores do outro matemático por alguma razão).

Inicialmente, P é todos os naturais. Se algum dos dois matemáticos possui 0
(i.e. o menor natural), eles sabem que o oponente possui 1. Assim, depois
de uma rodada sem um sim, podemos excluir 0 de P.

Numa dada rodada de perguntas, seja n o menor natural em P. Se alguim dos
dois possui n, então eles sabem que o oponente possui n+1 (pois n-1 < n,
portanto n-1 não pertence a P). Assim, depois dessa rodada, podemos excluir
n de P.

Assim, para qualquer valor de "a", teremos que P só contém "a+1" após "a-1"
rodadas, e nesse momento o matemático que possui "a" saberá o valor do
oponente.

Em seg., 15 de jul. de 2024 às 19:55, Gilberto Azevedo 
escreveu:

> Dois gênios matemáticos recebem dois numeros naturais consecutivos (eles
> só sabem o próprio número e que são consecutivos, mas nao sabem quem é o
> maior.)
> Eles se alternam perguntando: vc ja sabe qual o meu número? E respondem
> sinceramente.
> Mostre que em algum momento algum dos dois diz sim.
>
> Como que responde isso formalmente? Não consigo terminar a linha de
> raciocínio.
>
> --
> Esta mensagem foi verificada pelo sistema de antivírus e
> acredita-se estar livre de perigo.

-- 
Esta mensagem foi verificada pelo sistema de antiv�rus e
 acredita-se estar livre de perigo.



[obm-l] Re: limite de idade OBMU

2024-06-17 Por tôpico Luiz Eduardo Ardovino
ah, podem ignorar essa pergunta. eu já perguntei isso antes aqui e fui
respondido.

Em seg., 17 de jun. de 2024 às 12:55, Luiz Eduardo Ardovino <
luizeduardoardov...@gmail.com> escreveu:

> Olá a todos, Bom dia/tarde/noite.
>
> Há algum limite de idade para alguém participar da OBMU?
>

-- 
Esta mensagem foi verificada pelo sistema de antiv�rus e
 acredita-se estar livre de perigo.



[obm-l] Re: [obm-l] Re: [obm-l] Re: [obm-l] Re: [obm-l] Combinatória

2024-03-16 Por tôpico Anderson Torres
Em qua., 13 de mar. de 2024 às 13:07, Claudio Buffara
 escreveu:
>
> Mas este caso tem 7 pessoas. E o enunciado fala em 3 A e 3 C.
>
> On Wed, Mar 13, 2024 at 9:28 AM Pedro Júnior  
> wrote:
>>
>> Eu pensei sim, mas e os casos do tipo ACCACAC. Esse caso não entra na conta 
>> 6! - 2* 3!* 3!.
>>
>> Em qua., 13 de mar. de 2024 às 09:09, Claudio Buffara 
>>  escreveu:
>>>
>>> Pense no oposto: de quantas maneiras as crianças e adultos podem se sentar 
>>> separados uns dos outros.
>>>
>>> On Wed, Mar 13, 2024 at 8:39 AM Pedro Júnior  
>>> wrote:

 Olá pessoal, bom dia.
 Alguém poderia me ajudar nesse problema?

 Seis poltronas enfileiradas em um cinema e entram 3 adultos e 3 crianças. 
 De quantas maneiras podem sentar-se 2 crianças juntas e dois adultos 
 juntos?


Eu pensei numa maneira muito feia: enumeração na raça!

AAA CCC +

AACACC +
AACCAC +
AACCCA +

ACAACC +
ACACAC -
ACACCA -

CAAACC +
CAACAC -
CAACCC +

ACCAAC +
ACCACA -
ACCCAA +

CACAAC -
CACACA -
CACCAA +

CCAAAC +
CCAACA +
CCACAA +

CCC AAA +

20 possibilidades. Apenas 14 servem.
Cada uma nos permite permutar as crianças seis vezes, os adultos outras seis.
6x6x14, que dá meh.


 Desde já fico grato!

 --
 Esta mensagem foi verificada pelo sistema de antivírus e
 acredita-se estar livre de perigo.
>>>
>>>
>>> --
>>> Esta mensagem foi verificada pelo sistema de antivírus e
>>> acredita-se estar livre de perigo.
>>
>>
>>
>> --
>>
>> Pedro Jerônimo S. de O. Júnior
>>
>> Professor de Matemática
>>
>> Geo João Pessoa – PB
>>
>>
>> --
>> Esta mensagem foi verificada pelo sistema de antivírus e
>> acredita-se estar livre de perigo.
>
>
> --
> Esta mensagem foi verificada pelo sistema de antivírus e
> acredita-se estar livre de perigo.

-- 
Esta mensagem foi verificada pelo sistema de antiv�rus e
 acredita-se estar livre de perigo.


=
Instru��es para entrar na lista, sair da lista e usar a lista em
http://www.mat.puc-rio.br/~obmlistas/obm-l.html
=


[obm-l] Re: [obm-l] Re: [obm-l] Re: [obm-l] Combinatória

2024-03-13 Por tôpico Claudio Buffara
Mas este caso tem 7 pessoas. E o enunciado fala em 3 A e 3 C.

On Wed, Mar 13, 2024 at 9:28 AM Pedro Júnior 
wrote:

> Eu pensei sim, mas e os casos do tipo ACCACAC. Esse caso não entra na
> conta 6! - 2* 3!* 3!.
>
> Em qua., 13 de mar. de 2024 às 09:09, Claudio Buffara <
> claudio.buff...@gmail.com> escreveu:
>
>> Pense no oposto: de quantas maneiras as crianças e adultos podem se
>> sentar separados uns dos outros.
>>
>> On Wed, Mar 13, 2024 at 8:39 AM Pedro Júnior 
>> wrote:
>>
>>> Olá pessoal, bom dia.
>>> Alguém poderia me ajudar nesse problema?
>>>
>>> Seis poltronas enfileiradas em um cinema e entram 3 adultos e 3
>>> crianças. De quantas maneiras podem sentar-se 2 crianças juntas e dois
>>> adultos juntos?
>>>
>>>
>>> Desde já fico grato!
>>>
>>> --
>>> Esta mensagem foi verificada pelo sistema de antivírus e
>>> acredita-se estar livre de perigo.
>>
>>
>> --
>> Esta mensagem foi verificada pelo sistema de antivírus e
>> acredita-se estar livre de perigo.
>
>
>
> --
>
> Pedro Jerônimo S. de O. Júnior
>
> Professor de Matemática
>
> Geo João Pessoa – PB
>
> --
> Esta mensagem foi verificada pelo sistema de antivírus e
> acredita-se estar livre de perigo.

-- 
Esta mensagem foi verificada pelo sistema de antiv�rus e
 acredita-se estar livre de perigo.



[obm-l] Re: [obm-l] Re: [obm-l] Combinatória

2024-03-13 Por tôpico Pedro Júnior
Eu pensei sim, mas e os casos do tipo ACCACAC. Esse caso não entra na conta
6! - 2* 3!* 3!.

Em qua., 13 de mar. de 2024 às 09:09, Claudio Buffara <
claudio.buff...@gmail.com> escreveu:

> Pense no oposto: de quantas maneiras as crianças e adultos podem se sentar
> separados uns dos outros.
>
> On Wed, Mar 13, 2024 at 8:39 AM Pedro Júnior 
> wrote:
>
>> Olá pessoal, bom dia.
>> Alguém poderia me ajudar nesse problema?
>>
>> Seis poltronas enfileiradas em um cinema e entram 3 adultos e 3 crianças.
>> De quantas maneiras podem sentar-se 2 crianças juntas e dois adultos juntos?
>>
>>
>> Desde já fico grato!
>>
>> --
>> Esta mensagem foi verificada pelo sistema de antivírus e
>> acredita-se estar livre de perigo.
>
>
> --
> Esta mensagem foi verificada pelo sistema de antivírus e
> acredita-se estar livre de perigo.



-- 

Pedro Jerônimo S. de O. Júnior

Professor de Matemática

Geo João Pessoa – PB

-- 
Esta mensagem foi verificada pelo sistema de antiv�rus e
 acredita-se estar livre de perigo.



[obm-l] Re: [obm-l] Combinatória

2024-03-13 Por tôpico Claudio Buffara
Pense no oposto: de quantas maneiras as crianças e adultos podem se sentar
separados uns dos outros.

On Wed, Mar 13, 2024 at 8:39 AM Pedro Júnior 
wrote:

> Olá pessoal, bom dia.
> Alguém poderia me ajudar nesse problema?
>
> Seis poltronas enfileiradas em um cinema e entram 3 adultos e 3 crianças.
> De quantas maneiras podem sentar-se 2 crianças juntas e dois adultos juntos?
>
>
> Desde já fico grato!
>
> --
> Esta mensagem foi verificada pelo sistema de antivírus e
> acredita-se estar livre de perigo.

-- 
Esta mensagem foi verificada pelo sistema de antiv�rus e
 acredita-se estar livre de perigo.



[obm-l] Re: [obm-l] Re: [obm-l] Desigualdade e frações

2024-02-27 Por tôpico Pedro Júnior
Desculpas, Cláudio. É isso mesmo, com "a" e "b" inteiros e positivos.

Obrigado pela brilhante solução.

Em ter, 27 de fev de 2024 01:41, Claudio Buffara 
escreveu:

> Deveria ser a e b inteiros positivos, não?
> Pois se forem inteiros sem restrição, então como 2022/2023 < 2022,5/2023,5
> < 2023/2024, bastaria tomar a sequência:
> a(n) = -20225*n  e  b(n) = -20235*n.
> Daí teríamos 2022/2023 < a(n)/b(n) < 2023/2024 e a sequência a(n)+b(n)
> seria ilimitada inferiormente.
>
> Assim, suponhamos que a e b sejam inteiros positivos.
> 2022/2023 < a/b < 2023/2024 implica que b > a+1, já que a sequência
> (n/(n+1)) é crescente.
> Além disso, usando razões e proporções, achamos que:
> 2022 < a/(b-a) < 2023 < b/(b-a) < 2024
> ==> para que a+b seja o menor possível, b-a deverá ser o menor possível.
> E o menor valor possível de b-a é 2.
> Usando frações equivalentes, dá pra escrever 4044/4046 < a/b < 4046/4048 e
> daí teríamos uma única fração a/b com b - a = 2.
> Seria a/b = 4045/4047 ==> a+b mínimo = 8092.
>
> []s,
> Claudio.
>
>
>
>
> On Mon, Feb 26, 2024 at 10:12 PM Pedro Júnior 
> wrote:
>
>> Quem puder me ajudar, fixo grato.
>>
>> Sejam a e b dois números inteiros. Sabendo que 2022/2023 < a/b <
>> 2023/2024, determine o menos calor da soma a + b.
>>
>> --
>> Esta mensagem foi verificada pelo sistema de antivírus e
>> acredita-se estar livre de perigo.
>
>
> --
> Esta mensagem foi verificada pelo sistema de antivírus e
> acredita-se estar livre de perigo.

-- 
Esta mensagem foi verificada pelo sistema de antiv�rus e
 acredita-se estar livre de perigo.



[obm-l] Re: [obm-l] Desigualdade e frações

2024-02-26 Por tôpico Claudio Buffara
Deveria ser a e b inteiros positivos, não?
Pois se forem inteiros sem restrição, então como 2022/2023 < 2022,5/2023,5
< 2023/2024, bastaria tomar a sequência:
a(n) = -20225*n  e  b(n) = -20235*n.
Daí teríamos 2022/2023 < a(n)/b(n) < 2023/2024 e a sequência a(n)+b(n)
seria ilimitada inferiormente.

Assim, suponhamos que a e b sejam inteiros positivos.
2022/2023 < a/b < 2023/2024 implica que b > a+1, já que a sequência
(n/(n+1)) é crescente.
Além disso, usando razões e proporções, achamos que:
2022 < a/(b-a) < 2023 < b/(b-a) < 2024
==> para que a+b seja o menor possível, b-a deverá ser o menor possível.
E o menor valor possível de b-a é 2.
Usando frações equivalentes, dá pra escrever 4044/4046 < a/b < 4046/4048 e
daí teríamos uma única fração a/b com b - a = 2.
Seria a/b = 4045/4047 ==> a+b mínimo = 8092.

[]s,
Claudio.




On Mon, Feb 26, 2024 at 10:12 PM Pedro Júnior 
wrote:

> Quem puder me ajudar, fixo grato.
>
> Sejam a e b dois números inteiros. Sabendo que 2022/2023 < a/b <
> 2023/2024, determine o menos calor da soma a + b.
>
> --
> Esta mensagem foi verificada pelo sistema de antivírus e
> acredita-se estar livre de perigo.

-- 
Esta mensagem foi verificada pelo sistema de antiv�rus e
 acredita-se estar livre de perigo.



[obm-l] Re: [obm-l] Desigualdade e frações

2024-02-26 Por tôpico Alexandre Antunes
Vejam se este caminho é uma possibilidade (sujeita a ajustes e correções.
Fiquem à vontade!)
2022/2023 < a/b < 2023/2024 (I)
2022/2023 < (a+b-b)/b < 2023/2024
2022/2023 < (a+b)/b-b/b < 2023/2024
2022/2023 < (a+b)/b-1 < 2023/2024
2022/2023 +1< (a+b)/b-1 +1 < 2023/2024+1
(2022+2023)/2023 < (a+b)/b < (2023+2024)/2024
4045/2023 < (a+b)/b < 4047/2024
1,999505... aprox 2 < (a+b)/b < 1.999505... approx 2
*2 < (a+b)/b < 2 => (a+b)/b = 2(II)*

De (I), tem-se que  2022/2023 = 0,999505... aprox 1 < a/b < 2023/2024 =
0,999505... aprox 1
*1 < a/b < 1 =>   a/b = 1  (III)*

Sendo a e b inteiros, de (II) e (III), pode-se concluir que a=b=-1 e
somando a+b = -2.

Atenciosamente,

Prof. Dsc. Alexandre Antunes
www alexandre antunes com br


Em seg., 26 de fev. de 2024 às 22:11, Pedro Júnior <
pedromatematic...@gmail.com> escreveu:

> Quem puder me ajudar, fixo grato.
>
> Sejam a e b dois números inteiros. Sabendo que 2022/2023 < a/b <
> 2023/2024, determine o menos calor da soma a + b.
>
> --
> Esta mensagem foi verificada pelo sistema de antivírus e
> acredita-se estar livre de perigo.

-- 
Esta mensagem foi verificada pelo sistema de antiv�rus e
 acredita-se estar livre de perigo.



[obm-l] Re: [obm-l] Re: Const. de triângulo

2024-01-16 Por tôpico Anderson Torres
Em ter, 16 de jan de 2024 12:23, Claudio Buffara 
escreveu:

> "Há vários problemas de CT com duas soluções."
>
> Claro!...   Fora o óbvio , com infinitas soluções (todas
> semelhantes entre si...) tem o  se, por exemplo, A for agudo e a < b
> < a/sen(A).
>
> O Geogebra certamente é uma tremenda ferramenta.
> Mas quantos professores sabem usá-lo adequadamente?
>


1. Quantos professores têm contato? Mesmo fatorando o acesso à internet,
ainda creio serem poucos

2. Seria interessante se fosse adotado, é bem lúdico!


> []s,
> Claudio.
>
>
> On Mon, Jan 15, 2024 at 7:53 PM Luís Lopes  wrote:
>
>> Oi Claudio,
>>
>> Eu acho que para os problemas no contexto que estamos falando a álgebra
>> pode decidir. Como o 17-gon. É construtível mas talvez a construção em si
>> poderia não ser conhecida. Os problemas dados 3 pontos da lista do Wernick
>> também precisaram de pesquisas para se decidir. Mas não sei muito sobre o
>> assunto.
>>
>> Há vários problemas de CT com duas soluções.
>>
>> O problema do quadrilátero é muito legal e também muito difícil acho que
>> para qualquer um. Há soluções (não sei se são fundamentalmente diferentes)
>> no livro do Virgílio, Court e do FG-M.
>>
>> Mas, pra mim, a principal função destes problemas de construção e’
>> pedagógica.
>>
>> É isso aí. Muita criatividade. E o Geogebra pode ajudar muito.
>>
>> Abs,
>> Luís
>>
>>
>>
>> On Jan 14, 2024, at 11:21 AM, Claudio Buffara 
>> wrote:
>>
>> Não tenho dúvidas de que o nível de dificuldade destes problemas varia de
>> “trivial” até “extremamente difícil”. Talvez até existam problemas em
>> aberto - ninguém acha uma solução e nem consegue provar que não existe
>> solução.
>>
>> O problem dos dados e’ interessante: existem triplas de dados que
>> resultam em dois ou mais triângulos não congruentes? Os casos clássicos de
>> congruência sugerem que a resposta é não. Mas talvez alguns tipos de dado
>> sejam mais “fracos” e não determinem totalmente o triângulo.
>>
>> Saindo dos triângulos, um legal e não muito fácil (pra mim…) é construir
>> um quadrilátero inscritível dados os comprimentos dos lados.
>>
>> Mas, pra mim, a principal função destes problemas de construção e’
>> pedagógica. Inseridos num curso de geometria, eles são uma variante
>> interessante de problemas métricos (a enorme maioria dos problemas vistos
>> na escola) nos quais os estudantes precisam usar a criatividade pra aplicar
>> propriedades básicas de figuras geométricas simples mas de um jeito
>> diferente, com muito mais necessidade de visualização.
>>
>> []s,
>> Claudio
>>
>> Em dom., 14 de jan. de 2024 às 11:41, Luís Lopes 
>> escreveu:
>>
>>> Oi Claudio,
>>>
>>> Mando pra vc com CC pra lista pra fazer mais um teste e ver se a lista
>>> recebe. Reply não funciona.
>>>
>>> Outra maneira seria usando o triângulo AMaMb. Esse problema é simples.
>>> Mais interessantes são (d_a; e_a bissetrizes interna e externa)  e
>>>  e os primos esquecidos <,e_a>.
>>>
>>> Problemas com e_a não são muito vistos. Como aquele que apareceu no
>>> WhatsApp do Madeira: construir o triângulo retângulo dados D_b, D_c e X,
>>> ponto do incírculo na reta BC. Não considerei com E_b , E_c, a gente acaba
>>> esquecendo. Nem sei como seria. Ou até com X_a, ponto do
>>> A-exincírculo. A lista é enorme.
>>>
>>> Considere agora . Tirei o < _a>. Bem fácil. E como dados dois qq
>>> entre  o terceiro fica determinado (sem falar em
>>> B-C), então  e  também são fáceis. E  cai na
>>> categoria  e .
>>>
>>> O que pode ser um desafio é a discussão sobre os dados nos problemas
>>> . Todos eles têm somente uma solução (considerando triângulos não
>>> congruentes, a segunda solução no , m>=h> não conta). No  os
>>> dados têm que satisfazer d sin(A/2) < h <= d. Para  não sei como
>>> determinar.
>>>
>>> Abs,
>>> Luís
>>>
>>>
>>> On Jan 14, 2024, at 7:48 AM, Claudio Buffara 
>>> wrote:
>>>
>>> 
>>>
>>> Trace AM com comprimento m_a.
>>> Trace a circunferência com diâmetro AM.
>>> Trace AP com comprimento h_a e P na circunferência.
>>>
>>> * M será o ponto médio de BC e P o pé da altura relativa a A.
>>>
>>> Prolonga AM até MA', com AM = MA'.
>>>
>>> * AA' será a diagonal do paralelogramo ABA'C, cujas diagonais se
>>> bissectam em M.
>>>
>>> Traça arco capaz de 180-A sobre AA'.
>>>
>>> * Já que, num paralelogramo, ângulos consecutivos são suplementares.
>>>
>>> Chame de B o ponto de intersecção deste arco capaz com a reta PM.
>>> Marque C na reta PM tal que B-M-C e MC = MB.
>>> E acabou.
>>>
>>> Há outra solução marcando P na outra semicircunferência de diâmetro AM
>>> (a menos que h_a = m_a).
>>>
>>> []s,
>>> Claudio.
>>>
>>>
>>> On Sun, Jan 14, 2024 at 12:58 AM Luís Lopes 
>>> wrote:
>>>
 Saudações, oi Anderson,

 Soluções usando fórmulas servem para mostrar que o triângulo é
 construtível e qual é sua forma e tamanho. Já ajuda naquela parte - suponha
 o problema resolvido. Mas a construção procurada deverá ser feita usando as
 propriedades da figura.

 Posso mandar no privad

[obm-l] Re: Const. de triângulo

2024-01-16 Por tôpico Claudio Buffara
"Há vários problemas de CT com duas soluções."

Claro!...   Fora o óbvio , com infinitas soluções (todas semelhantes
entre si...) tem o  se, por exemplo, A for agudo e a < b < a/sen(A).

O Geogebra certamente é uma tremenda ferramenta.
Mas quantos professores sabem usá-lo adequadamente?

[]s,
Claudio.


On Mon, Jan 15, 2024 at 7:53 PM Luís Lopes  wrote:

> Oi Claudio,
>
> Eu acho que para os problemas no contexto que estamos falando a álgebra
> pode decidir. Como o 17-gon. É construtível mas talvez a construção em si
> poderia não ser conhecida. Os problemas dados 3 pontos da lista do Wernick
> também precisaram de pesquisas para se decidir. Mas não sei muito sobre o
> assunto.
>
> Há vários problemas de CT com duas soluções.
>
> O problema do quadrilátero é muito legal e também muito difícil acho que
> para qualquer um. Há soluções (não sei se são fundamentalmente diferentes)
> no livro do Virgílio, Court e do FG-M.
>
> Mas, pra mim, a principal função destes problemas de construção e’
> pedagógica.
>
> É isso aí. Muita criatividade. E o Geogebra pode ajudar muito.
>
> Abs,
> Luís
>
>
>
> On Jan 14, 2024, at 11:21 AM, Claudio Buffara 
> wrote:
>
> Não tenho dúvidas de que o nível de dificuldade destes problemas varia de
> “trivial” até “extremamente difícil”. Talvez até existam problemas em
> aberto - ninguém acha uma solução e nem consegue provar que não existe
> solução.
>
> O problem dos dados e’ interessante: existem triplas de dados que resultam
> em dois ou mais triângulos não congruentes? Os casos clássicos de
> congruência sugerem que a resposta é não. Mas talvez alguns tipos de dado
> sejam mais “fracos” e não determinem totalmente o triângulo.
>
> Saindo dos triângulos, um legal e não muito fácil (pra mim…) é construir
> um quadrilátero inscritível dados os comprimentos dos lados.
>
> Mas, pra mim, a principal função destes problemas de construção e’
> pedagógica. Inseridos num curso de geometria, eles são uma variante
> interessante de problemas métricos (a enorme maioria dos problemas vistos
> na escola) nos quais os estudantes precisam usar a criatividade pra aplicar
> propriedades básicas de figuras geométricas simples mas de um jeito
> diferente, com muito mais necessidade de visualização.
>
> []s,
> Claudio
>
> Em dom., 14 de jan. de 2024 às 11:41, Luís Lopes 
> escreveu:
>
>> Oi Claudio,
>>
>> Mando pra vc com CC pra lista pra fazer mais um teste e ver se a lista
>> recebe. Reply não funciona.
>>
>> Outra maneira seria usando o triângulo AMaMb. Esse problema é simples.
>> Mais interessantes são (d_a; e_a bissetrizes interna e externa)  e
>>  e os primos esquecidos <,e_a>.
>>
>> Problemas com e_a não são muito vistos. Como aquele que apareceu no
>> WhatsApp do Madeira: construir o triângulo retângulo dados D_b, D_c e X,
>> ponto do incírculo na reta BC. Não considerei com E_b , E_c, a gente acaba
>> esquecendo. Nem sei como seria. Ou até com X_a, ponto do
>> A-exincírculo. A lista é enorme.
>>
>> Considere agora . Tirei o < _a>. Bem fácil. E como dados dois qq
>> entre  o terceiro fica determinado (sem falar em
>> B-C), então  e  também são fáceis. E  cai na
>> categoria  e .
>>
>> O que pode ser um desafio é a discussão sobre os dados nos problemas
>> . Todos eles têm somente uma solução (considerando triângulos não
>> congruentes, a segunda solução no , m>=h> não conta). No  os
>> dados têm que satisfazer d sin(A/2) < h <= d. Para  não sei como
>> determinar.
>>
>> Abs,
>> Luís
>>
>>
>> On Jan 14, 2024, at 7:48 AM, Claudio Buffara 
>> wrote:
>>
>> 
>>
>> Trace AM com comprimento m_a.
>> Trace a circunferência com diâmetro AM.
>> Trace AP com comprimento h_a e P na circunferência.
>>
>> * M será o ponto médio de BC e P o pé da altura relativa a A.
>>
>> Prolonga AM até MA', com AM = MA'.
>>
>> * AA' será a diagonal do paralelogramo ABA'C, cujas diagonais se
>> bissectam em M.
>>
>> Traça arco capaz de 180-A sobre AA'.
>>
>> * Já que, num paralelogramo, ângulos consecutivos são suplementares.
>>
>> Chame de B o ponto de intersecção deste arco capaz com a reta PM.
>> Marque C na reta PM tal que B-M-C e MC = MB.
>> E acabou.
>>
>> Há outra solução marcando P na outra semicircunferência de diâmetro AM (a
>> menos que h_a = m_a).
>>
>> []s,
>> Claudio.
>>
>>
>> On Sun, Jan 14, 2024 at 12:58 AM Luís Lopes 
>> wrote:
>>
>>> Saudações, oi Anderson,
>>>
>>> Soluções usando fórmulas servem para mostrar que o triângulo é
>>> construtível e qual é sua forma e tamanho. Já ajuda naquela parte - suponha
>>> o problema resolvido. Mas a construção procurada deverá ser feita usando as
>>> propriedades da figura.
>>>
>>> Posso mandar no privado para quem se interessar as construções com as
>>> figuras que um correspondente me enviou. Esse que tem h_c/b como dado é bem
>>> interessante.
>>>
>>> Agora o problema  pode ser resolvido de 3 ou mais maneiras.
>>> Com medianas é sempre bom pensar em simetrias e paralelogramos.
>>>
>>> Luís
>>>
>>>
>>> --
>>> Esta mensagem foi verificada pelo sistema de antivíru

[obm-l] Re: [obm-l] Const. de triângulo

2024-01-14 Por tôpico Anderson Torres
Em dom., 14 de jan. de 2024 às 00:58, Luís Lopes
 escreveu:
>
> Saudações, oi Anderson,
>
> Soluções usando fórmulas servem para mostrar que o triângulo é construtível e 
> qual é sua forma e tamanho.

Mostrar que é construtível, neste caso, implica mostrar a construção.
E ela é recheada de

> Já ajuda naquela parte - suponha o problema resolvido. Mas a construção 
> procurada deverá ser feita usando as propriedades da figura.
>
> Posso mandar no privado para quem se interessar as construções com as figuras 
> que um correspondente me enviou. Esse que tem h_c/b como dado é bem 
> interessante.
>
> Agora o problema  pode ser resolvido de 3 ou mais maneiras. Com 
> medianas é sempre bom pensar em simetrias e paralelogramos.

Acabei de resolver. A minha ideia foi a fórmula da mediana, que é
basicamente uma lei dos cossenos no triângulo (b,c,180-A).
Com isso, fica fácil construir o arco de circunferência que olha o
dobro da mediana de um ângulo 180-A.

Como disse antes, a solução full eu mando outro dia.

>
> Luís
>
>
> --
> Esta mensagem foi verificada pelo sistema de antivírus e
>  acredita-se estar livre de perigo.
>
>
> =
> Instru�ões para entrar na lista, sair da lista e usar a lista em
> http://www.mat.puc-rio.br/~obmlistas/obm-l.html
> =

-- 
Esta mensagem foi verificada pelo sistema de antiv�rus e
 acredita-se estar livre de perigo.


=
Instru��es para entrar na lista, sair da lista e usar a lista em
http://www.mat.puc-rio.br/~obmlistas/obm-l.html
=


[obm-l] Re: [obm-l] Re: [obm-l] Const. de triângulo

2024-01-14 Por tôpico Claudio Buffara
Não tenho dúvidas de que o nível de dificuldade destes problemas varia de
“trivial” até “extremamente difícil”. Talvez até existam problemas em
aberto - ninguém acha uma solução e nem consegue provar que não existe
solução.

O problem dos dados e’ interessante: existem triplas de dados que resultam
em dois ou mais triângulos não congruentes? Os casos clássicos de
congruência sugerem que a resposta é não. Mas talvez alguns tipos de dado
sejam mais “fracos” e não determinem totalmente o triângulo.

Saindo dos triângulos, um legal e não muito fácil (pra mim…) é construir um
quadrilátero inscritível dados os comprimentos dos lados.

Mas, pra mim, a principal função destes problemas de construção e’
pedagógica. Inseridos num curso de geometria, eles são uma variante
interessante de problemas métricos (a enorme maioria dos problemas vistos
na escola) nos quais os estudantes precisam usar a criatividade pra aplicar
propriedades básicas de figuras geométricas simples mas de um jeito
diferente, com muito mais necessidade de visualização.

[]s,
Claudio

Em dom., 14 de jan. de 2024 às 11:41, Luís Lopes 
escreveu:

> Oi Claudio,
>
> Mando pra vc com CC pra lista pra fazer mais um teste e ver se a lista
> recebe. Reply não funciona.
>
> Outra maneira seria usando o triângulo AMaMb. Esse problema é simples.
> Mais interessantes são (d_a; e_a bissetrizes interna e externa)  e
>  e os primos esquecidos <,e_a>.
>
> Problemas com e_a não são muito vistos. Como aquele que apareceu no
> WhatsApp do Madeira: construir o triângulo retângulo dados D_b, D_c e X,
> ponto do incírculo na reta BC. Não considerei com E_b , E_c, a gente acaba
> esquecendo. Nem sei como seria. Ou até com X_a, ponto do
> A-exincírculo. A lista é enorme.
>
> Considere agora . Tirei o < _a>. Bem fácil. E como dados dois qq
> entre  o terceiro fica determinado (sem falar em
> B-C), então  e  também são fáceis. E  cai na
> categoria  e .
>
> O que pode ser um desafio é a discussão sobre os dados nos problemas
> . Todos eles têm somente uma solução (considerando triângulos não
> congruentes, a segunda solução no , m>=h> não conta). No  os
> dados têm que satisfazer d sin(A/2) < h <= d. Para  não sei como
> determinar.
>
> Abs,
> Luís
>
>
> On Jan 14, 2024, at 7:48 AM, Claudio Buffara 
> wrote:
>
> 
>
> Trace AM com comprimento m_a.
> Trace a circunferência com diâmetro AM.
> Trace AP com comprimento h_a e P na circunferência.
>
> * M será o ponto médio de BC e P o pé da altura relativa a A.
>
> Prolonga AM até MA', com AM = MA'.
>
> * AA' será a diagonal do paralelogramo ABA'C, cujas diagonais se bissectam
> em M.
>
> Traça arco capaz de 180-A sobre AA'.
>
> * Já que, num paralelogramo, ângulos consecutivos são suplementares.
>
> Chame de B o ponto de intersecção deste arco capaz com a reta PM.
> Marque C na reta PM tal que B-M-C e MC = MB.
> E acabou.
>
> Há outra solução marcando P na outra semicircunferência de diâmetro AM (a
> menos que h_a = m_a).
>
> []s,
> Claudio.
>
>
> On Sun, Jan 14, 2024 at 12:58 AM Luís Lopes  wrote:
>
>> Saudações, oi Anderson,
>>
>> Soluções usando fórmulas servem para mostrar que o triângulo é
>> construtível e qual é sua forma e tamanho. Já ajuda naquela parte - suponha
>> o problema resolvido. Mas a construção procurada deverá ser feita usando as
>> propriedades da figura.
>>
>> Posso mandar no privado para quem se interessar as construções com as
>> figuras que um correspondente me enviou. Esse que tem h_c/b como dado é bem
>> interessante.
>>
>> Agora o problema  pode ser resolvido de 3 ou mais maneiras.
>> Com medianas é sempre bom pensar em simetrias e paralelogramos.
>>
>> Luís
>>
>>
>> --
>> Esta mensagem foi verificada pelo sistema de antivírus e
>>  acredita-se estar livre de perigo.
>>
>>
>> =
>> Instru�ões para entrar na lista, sair da lista e usar a lista em
>> http://www.mat.puc-rio.br/~obmlistas/obm-l.html
>> =
>>
>
> --
>
> Esta mensagem foi verificada pelo sistema de antiv�rus e
>
> acredita-se estar livre de perigo.
>
>

-- 
Esta mensagem foi verificada pelo sistema de antiv�rus e
 acredita-se estar livre de perigo.



[obm-l] Re: [obm-l] Const. de triângulo

2024-01-14 Por tôpico Claudio Buffara


Trace AM com comprimento m_a.
Trace a circunferência com diâmetro AM.
Trace AP com comprimento h_a e P na circunferência.

* M será o ponto médio de BC e P o pé da altura relativa a A.

Prolonga AM até MA', com AM = MA'.

* AA' será a diagonal do paralelogramo ABA'C, cujas diagonais se bissectam
em M.

Traça arco capaz de 180-A sobre AA'.

* Já que, num paralelogramo, ângulos consecutivos são suplementares.

Chame de B o ponto de intersecção deste arco capaz com a reta PM.
Marque C na reta PM tal que B-M-C e MC = MB.
E acabou.

Há outra solução marcando P na outra semicircunferência de diâmetro AM (a
menos que h_a = m_a).

[]s,
Claudio.


On Sun, Jan 14, 2024 at 12:58 AM Luís Lopes  wrote:

> Saudações, oi Anderson,
>
> Soluções usando fórmulas servem para mostrar que o triângulo é
> construtível e qual é sua forma e tamanho. Já ajuda naquela parte - suponha
> o problema resolvido. Mas a construção procurada deverá ser feita usando as
> propriedades da figura.
>
> Posso mandar no privado para quem se interessar as construções com as
> figuras que um correspondente me enviou. Esse que tem h_c/b como dado é bem
> interessante.
>
> Agora o problema  pode ser resolvido de 3 ou mais maneiras. Com
> medianas é sempre bom pensar em simetrias e paralelogramos.
>
> Luís
>
>
> --
> Esta mensagem foi verificada pelo sistema de antivírus e
>  acredita-se estar livre de perigo.
>
>
> =
> Instru�ões para entrar na lista, sair da lista e usar a lista em
> http://www.mat.puc-rio.br/~obmlistas/obm-l.html
> =
>

-- 
Esta mensagem foi verificada pelo sistema de antiv�rus e
 acredita-se estar livre de perigo.



[obm-l] Re: [obm-l] Re: [obm-l] Re: [obm-l] Pesagens ( Balança Eletrônica)

2023-11-19 Por tôpico Claudio Buffara
Por que você não começa com um caso menor, tal como 4, 6 ou 9 moedas no
total?

Como você não consegue distinguir, numa dada pesagem, um grupo só com
moedas verdadeiras e um grupo com 2 moedas falsas, um algoritmo
pra resolver este problema com o menor número possível de pesagens não me
parece óbvio.  Daí, a análise de um caso menor pode dar alguma luz.

[]s,
Claudio.


On Sun, Nov 19, 2023 at 3:50 PM Jeferson Almir 
wrote:

> Eu tinha errado umas contas, mas sua cota está correta Ralph, preciso
> montar um exemplo com 21 pesagens
>
> Em dom., 19 de nov. de 2023 às 15:00, Jeferson Almir <
> jefersonram...@gmail.com> escreveu:
>
>> Pelo visto, está sim Ralph!! Agora temos que montar uma estratégia que
>> com 21 pesagens.
>>
>> Em dom., 19 de nov. de 2023 às 13:55, Ralph Costa Teixeira <
>> ralp...@gmail.com> escreveu:
>>
>>> Existem 2022*2021/2 possibilidades para as 2 falsas. Qualquer estratégia
>>> que seja criada com k pesagens que dão apenas 2 respostas cada distingue no
>>> máximo dentre 2^k possibilidades. Então devemos ter 2^k >= 2022*2021/2...
>>> hmm, isso daria k como pelo menos 21? Errei algo?
>>>
>>> On Sun, Nov 19, 2023, 12:16 Jeferson Almir 
>>> wrote:
>>>
 Amigos, encontrei como K mínimo o valor 11 mas desconfio que seja
 menos. Se alguém souber uma ideia que acabe o problema serei grato.


 Em Villa Par todas as moedas autênticas pesam uma quantidade par de
 gramas e todas as moedas falsas pesam uma quantidade impar de gramas.

 Se você tiver 2022 moedas entre as quais sabe que exatamente 2 são
 falsas.

 Se tiver uma balança eletrônica que informe apenas se o peso total dos
 objetos colocados nela é par ou impar.

 Determine o valor mínimo de k para qualquer estratégia que permita
 identificar as moedas falsas usando a balança  no máximo k vezes.

 --
 Esta mensagem foi verificada pelo sistema de antivírus e
 acredita-se estar livre de perigo.
>>>
>>>
>>> --
>>> Esta mensagem foi verificada pelo sistema de antivírus e
>>> acredita-se estar livre de perigo.
>>
>>
> --
> Esta mensagem foi verificada pelo sistema de antivírus e
> acredita-se estar livre de perigo.

-- 
Esta mensagem foi verificada pelo sistema de antiv�rus e
 acredita-se estar livre de perigo.



[obm-l] Re: [obm-l] Re: [obm-l] Pesagens ( Balança Eletrônica)

2023-11-19 Por tôpico Jeferson Almir
Eu tinha errado umas contas, mas sua cota está correta Ralph, preciso
montar um exemplo com 21 pesagens

Em dom., 19 de nov. de 2023 às 15:00, Jeferson Almir <
jefersonram...@gmail.com> escreveu:

> Pelo visto, está sim Ralph!! Agora temos que montar uma estratégia que com
> 21 pesagens.
>
> Em dom., 19 de nov. de 2023 às 13:55, Ralph Costa Teixeira <
> ralp...@gmail.com> escreveu:
>
>> Existem 2022*2021/2 possibilidades para as 2 falsas. Qualquer estratégia
>> que seja criada com k pesagens que dão apenas 2 respostas cada distingue no
>> máximo dentre 2^k possibilidades. Então devemos ter 2^k >= 2022*2021/2...
>> hmm, isso daria k como pelo menos 21? Errei algo?
>>
>> On Sun, Nov 19, 2023, 12:16 Jeferson Almir 
>> wrote:
>>
>>> Amigos, encontrei como K mínimo o valor 11 mas desconfio que seja menos.
>>> Se alguém souber uma ideia que acabe o problema serei grato.
>>>
>>>
>>> Em Villa Par todas as moedas autênticas pesam uma quantidade par de
>>> gramas e todas as moedas falsas pesam uma quantidade impar de gramas.
>>>
>>> Se você tiver 2022 moedas entre as quais sabe que exatamente 2 são
>>> falsas.
>>>
>>> Se tiver uma balança eletrônica que informe apenas se o peso total dos
>>> objetos colocados nela é par ou impar.
>>>
>>> Determine o valor mínimo de k para qualquer estratégia que permita
>>> identificar as moedas falsas usando a balança  no máximo k vezes.
>>>
>>> --
>>> Esta mensagem foi verificada pelo sistema de antivírus e
>>> acredita-se estar livre de perigo.
>>
>>
>> --
>> Esta mensagem foi verificada pelo sistema de antivírus e
>> acredita-se estar livre de perigo.
>
>

-- 
Esta mensagem foi verificada pelo sistema de antiv�rus e
 acredita-se estar livre de perigo.



[obm-l] Re: [obm-l] Re: [obm-l] Pesagens ( Balança Eletrônica)

2023-11-19 Por tôpico Jeferson Almir
Pelo visto, está sim Ralph!! Agora temos que montar uma estratégia que com
21 pesagens.

Em dom., 19 de nov. de 2023 às 13:55, Ralph Costa Teixeira <
ralp...@gmail.com> escreveu:

> Existem 2022*2021/2 possibilidades para as 2 falsas. Qualquer estratégia
> que seja criada com k pesagens que dão apenas 2 respostas cada distingue no
> máximo dentre 2^k possibilidades. Então devemos ter 2^k >= 2022*2021/2...
> hmm, isso daria k como pelo menos 21? Errei algo?
>
> On Sun, Nov 19, 2023, 12:16 Jeferson Almir 
> wrote:
>
>> Amigos, encontrei como K mínimo o valor 11 mas desconfio que seja menos.
>> Se alguém souber uma ideia que acabe o problema serei grato.
>>
>>
>> Em Villa Par todas as moedas autênticas pesam uma quantidade par de
>> gramas e todas as moedas falsas pesam uma quantidade impar de gramas.
>>
>> Se você tiver 2022 moedas entre as quais sabe que exatamente 2 são falsas.
>>
>> Se tiver uma balança eletrônica que informe apenas se o peso total dos
>> objetos colocados nela é par ou impar.
>>
>> Determine o valor mínimo de k para qualquer estratégia que permita
>> identificar as moedas falsas usando a balança  no máximo k vezes.
>>
>> --
>> Esta mensagem foi verificada pelo sistema de antivírus e
>> acredita-se estar livre de perigo.
>
>
> --
> Esta mensagem foi verificada pelo sistema de antivírus e
> acredita-se estar livre de perigo.

-- 
Esta mensagem foi verificada pelo sistema de antiv�rus e
 acredita-se estar livre de perigo.



[obm-l] Re: [obm-l] Pesagens ( Balança Eletrônica)

2023-11-19 Por tôpico Ralph Costa Teixeira
Existem 2022*2021/2 possibilidades para as 2 falsas. Qualquer estratégia
que seja criada com k pesagens que dão apenas 2 respostas cada distingue no
máximo dentre 2^k possibilidades. Então devemos ter 2^k >= 2022*2021/2...
hmm, isso daria k como pelo menos 21? Errei algo?

On Sun, Nov 19, 2023, 12:16 Jeferson Almir  wrote:

> Amigos, encontrei como K mínimo o valor 11 mas desconfio que seja menos.
> Se alguém souber uma ideia que acabe o problema serei grato.
>
>
> Em Villa Par todas as moedas autênticas pesam uma quantidade par de gramas
> e todas as moedas falsas pesam uma quantidade impar de gramas.
>
> Se você tiver 2022 moedas entre as quais sabe que exatamente 2 são falsas.
>
> Se tiver uma balança eletrônica que informe apenas se o peso total dos
> objetos colocados nela é par ou impar.
>
> Determine o valor mínimo de k para qualquer estratégia que permita
> identificar as moedas falsas usando a balança  no máximo k vezes.
>
> --
> Esta mensagem foi verificada pelo sistema de antivírus e
> acredita-se estar livre de perigo.

-- 
Esta mensagem foi verificada pelo sistema de antiv�rus e
 acredita-se estar livre de perigo.



[obm-l] Re: [obm-l] Re: [obm-l] equação

2023-10-25 Por tôpico Daniel Jelin
Obrigado, Marcelo, abs!

Em qua., 25 de out. de 2023 00:24, Marcelo Gonda Stangler <
marcelo.gonda.stang...@gmail.com> escreveu:

> Este problema, com um pouco de uso de substituição pode ser mostrado como
> análogo a isolar em x: k=x-e^(-1/x+1)
> Tu precisas limitar o "quanto" estás disposto a fatorar, pois poderiamos
> isolar x deixando-o em função de f(x) tal que f(x)-e^(-1/f(x)+1)=k. Mas
> suspeito que não é isto que queres.
> Se estamos falando de isolar algebricamente x, podemos notar alguns pontos:
>   Exp(x) para valores não triviais causa transformações relativas a x na
> base minimal que contém x de extensão sobre A, o corpo dos algébricos.
>   Se k é algébrico não nulo, x deve ser transcedental, visto que e é
> transcedental e (-1/x+1) pertence ao corpo dos A[x], assim x ser algebráico
> seria um absurdo.
>   Se x é algébrico, à exceção de 1, raiz de -1/x+1, k será transcedental
> uma vez que e o é.
> Assim, à exceção do caso (k,x)=(0,1), não haverá soluções em que x e k são
> algebráicos. Então, ao isolar o x, obteriamos algo em relação a "e" ou "ln".
> Como k=x-e^(-1/x+1), a base minimal de extensão que contém k é a união
> desta base de x, e da base transformada de x por Exp().
> Assim, a base minimal de x teria que ser a união da base de k e da base
> transformada de k por Exp() (1) ou Ln() (2).
> (1) implica que ambos são algébricos e (k,x)=(0,1)
> (2) implica que BM(x) = BM(k) U BM(Ln(k)) = BM(x) U BM(Exp(x)) U
> BM(Ln(k)), também implica (k,x)=(0,1)
>
> Dessa forma provamos que é impossível 'isolar' o x em função de k.
>
> Em ter, 24 de out de 2023 21:15, Daniel Jelin 
> escreveu:
>
>> Caros, olá. Tenho a seguinte equação: 1/ln(x) - 1/(x-1) = k, com x e k
>> reais. Quero isolar o x, mas não consigo. Pergunto: alguém tem alguma dica?
>> E pergunto tb: é possível que simplesmente não haja meios de isolar o x?
>> Nesse caso, como se prova isso? abs.
>>
>> --
>> Esta mensagem foi verificada pelo sistema de antivírus e
>> acredita-se estar livre de perigo.
>
>
> --
> Esta mensagem foi verificada pelo sistema de antivírus e
> acredita-se estar livre de perigo.

-- 
Esta mensagem foi verificada pelo sistema de antiv�rus e
 acredita-se estar livre de perigo.



[obm-l] Re: [obm-l] equação

2023-10-24 Por tôpico Marcelo Gonda Stangler
Este problema, com um pouco de uso de substituição pode ser mostrado como
análogo a isolar em x: k=x-e^(-1/x+1)
Tu precisas limitar o "quanto" estás disposto a fatorar, pois poderiamos
isolar x deixando-o em função de f(x) tal que f(x)-e^(-1/f(x)+1)=k. Mas
suspeito que não é isto que queres.
Se estamos falando de isolar algebricamente x, podemos notar alguns pontos:
  Exp(x) para valores não triviais causa transformações relativas a x na
base minimal que contém x de extensão sobre A, o corpo dos algébricos.
  Se k é algébrico não nulo, x deve ser transcedental, visto que e é
transcedental e (-1/x+1) pertence ao corpo dos A[x], assim x ser algebráico
seria um absurdo.
  Se x é algébrico, à exceção de 1, raiz de -1/x+1, k será transcedental
uma vez que e o é.
Assim, à exceção do caso (k,x)=(0,1), não haverá soluções em que x e k são
algebráicos. Então, ao isolar o x, obteriamos algo em relação a "e" ou "ln".
Como k=x-e^(-1/x+1), a base minimal de extensão que contém k é a união
desta base de x, e da base transformada de x por Exp().
Assim, a base minimal de x teria que ser a união da base de k e da base
transformada de k por Exp() (1) ou Ln() (2).
(1) implica que ambos são algébricos e (k,x)=(0,1)
(2) implica que BM(x) = BM(k) U BM(Ln(k)) = BM(x) U BM(Exp(x)) U BM(Ln(k)),
também implica (k,x)=(0,1)

Dessa forma provamos que é impossível 'isolar' o x em função de k.

Em ter, 24 de out de 2023 21:15, Daniel Jelin 
escreveu:

> Caros, olá. Tenho a seguinte equação: 1/ln(x) - 1/(x-1) = k, com x e k
> reais. Quero isolar o x, mas não consigo. Pergunto: alguém tem alguma dica?
> E pergunto tb: é possível que simplesmente não haja meios de isolar o x?
> Nesse caso, como se prova isso? abs.
>
> --
> Esta mensagem foi verificada pelo sistema de antivírus e
> acredita-se estar livre de perigo.

-- 
Esta mensagem foi verificada pelo sistema de antiv�rus e
 acredita-se estar livre de perigo.



[obm-l] Re: [obm-l] Números primos

2023-10-05 Por tôpico Anderson Torres
Em qua, 4 de out de 2023 15:49, carlos h Souza 
escreveu:

> Boa tarde,
>
> Para fins didáticos é mais fácil encontrar os números primos em forma de
> fatoração numérica ou usar o Crivo de Eratóstenes ?
>


Fatoração, de longe.

Os primos são definidos precisamente como "os infatoráveis".

Já o crivo de Eratóstenes é um algoritmo de classificação em massa.

Pensa da seguinte forma: para verificar se um número N é primo, o que é
mais natural:
- tentar dividir em k partes iguais, para todos os k pequenos;
- escrever todos os números de 1 a N num papel e ir furando o papel de
acordo com uma regra mágica?



> Obrigados a todos.
>
> --
> Esta mensagem foi verificada pelo sistema de antivírus e
> acredita-se estar livre de perigo.

-- 
Esta mensagem foi verificada pelo sistema de antiv�rus e
 acredita-se estar livre de perigo.



[obm-l] Re: [obm-l] Números primos

2023-10-04 Por tôpico Claudio Buffara
Fatoração, com certeza.  Por exemplo, diga pra garotada analisar os números
de 2 a 100 e determinar quais podem ser expressos como produto de números
naturais menores.  Como dica, pra facilitar o trabalho, diga pra eles
consultarem a tabuada (e também pra observarem que, na tabuada, nem todos
os números aparecem como resultado de alguma multiplicação).  Acho que
essa é uma boa motivação pra definição de número primo.
As dificuldades encontradas por eles nesta tarefa podem motivar a busca de
uma forma sistemática (um algoritmo) pra determinar os números primos na
sequência de números naturais. Esse seria o crivo de Eratóstenes, cuja
descoberta poderia ser guiada por perguntas e dicas pertinentes.

Outra forma de motivar a definição de primo é representar o natural N (N =
1, 2, 3, ...) por N bolinhas, que devem ser dispostas num arranjo
retangular com 2 ou mais linhas (ou colunas).  Para alguns valores de N,
isso será impossível.  Estes são os números primos.
Numa digressão, faça a garotada determinar pra quais N as bolinhas podem
ser particionadas em pares (conjuntos com 2 elementos)... daí o nome.
Há vários probleminhas interessantes que podem ser resolvidos com esta
representação dos números - o do jovem Gauss, por exemplo, ou o da soma dos
ímpares consecutivos, ou determinar pra quais N o arranjo pode ter o mesmo
número de linhas e de colunas.

[]s,
Claudio.


On Wed, Oct 4, 2023 at 3:49 PM carlos h Souza  wrote:

> Boa tarde,
>
> Para fins didáticos é mais fácil encontrar os números primos em forma de
> fatoração numérica ou usar o Crivo de Eratóstenes ?
>
> Obrigados a todos.
>
> --
> Esta mensagem foi verificada pelo sistema de antivírus e
> acredita-se estar livre de perigo.

-- 
Esta mensagem foi verificada pelo sistema de antiv�rus e
 acredita-se estar livre de perigo.



[obm-l] Re: [obm-l] Re: [obm-l] Re: [obm-l] Comunicação

2023-08-28 Por tôpico Tiago Sandino
https://t.me/+jz8XW7bgRqNlOTg5

Criei esse grupo no telegram. A principal vantagem do Telegram em relação
ao Whatsapp é que quem entra pode ter acesso a todas as mensagens e
arquivos anteriores. A quantidade de membros que podem entrar é de 200.000.
Tô pensando aqui em umas regras também, tais como proibição de pirataria,
spam e algumas coisas a mais que podemos discutir lá. Melhor que isso, só
se alguém fizesse um fórum e fosse possível escrever em latex lá. Tem o
AOPS ok, mas nada nosso mesmo. Fiquem à vontade para entrar. Se o link
expirar, podem me solicitar outro por aqui.

Att.
*Prof. Tiago Sandino*
*85 999134896*

Em seg., 28 de ago. de 2023 02:02, Rogerio Ponce 
escreveu:

> Ola pessoal!
>
> Nesta lista, da qual participamos, qualquer um (mesmo que não esteja
> inscrito na lista) pode acessar os arquivos, fazer pesquisas e ler
> todos os problemas e suas solucoes.
>
> No whatsapp, isto seria impossivel, a menos que o individuo ja
> estivesse participando desde o inicio.
>
> []'s
> Rogerio Ponce
>
> On Sat, Aug 26, 2023 at 1:36 PM Esdras Muniz 
> wrote:
> >
> > Seria muito legal se existisse.
> >
> > Em sex, 25 de ago de 2023 18:24, Priscila Santana <
> priscila@hotmail.com> escreveu:
> >>
> >>
> >> Olá!
> >>
> >> Existe algum grupo de discussão de questões olímpicas no WhatsApp?
> >>
> >> Atte.
> >>
> >> Priscila S. da Paz
> >>
> >>
> >>
> >> --
> >> Esta mensagem foi verificada pelo sistema de antivírus e
> >> acredita-se estar livre de perigo.
> >
> >
> > --
> > Esta mensagem foi verificada pelo sistema de antivírus e
> > acredita-se estar livre de perigo.
>
> --
> Esta mensagem foi verificada pelo sistema de antivírus e
>  acredita-se estar livre de perigo.
>
>
> =
> Instru�ões para entrar na lista, sair da lista e usar a lista em
> http://www.mat.puc-rio.br/~obmlistas/obm-l.html
> =
>

-- 
Esta mensagem foi verificada pelo sistema de antiv�rus e
 acredita-se estar livre de perigo.



[obm-l] Re: [obm-l] Re: [obm-l] Comunicação

2023-08-27 Por tôpico Rogerio Ponce
Ola pessoal!

Nesta lista, da qual participamos, qualquer um (mesmo que não esteja
inscrito na lista) pode acessar os arquivos, fazer pesquisas e ler
todos os problemas e suas solucoes.

No whatsapp, isto seria impossivel, a menos que o individuo ja
estivesse participando desde o inicio.

[]'s
Rogerio Ponce

On Sat, Aug 26, 2023 at 1:36 PM Esdras Muniz  wrote:
>
> Seria muito legal se existisse.
>
> Em sex, 25 de ago de 2023 18:24, Priscila Santana  
> escreveu:
>>
>>
>> Olá!
>>
>> Existe algum grupo de discussão de questões olímpicas no WhatsApp?
>>
>> Atte.
>>
>> Priscila S. da Paz
>>
>>
>>
>> --
>> Esta mensagem foi verificada pelo sistema de antivírus e
>> acredita-se estar livre de perigo.
>
>
> --
> Esta mensagem foi verificada pelo sistema de antivírus e
> acredita-se estar livre de perigo.

-- 
Esta mensagem foi verificada pelo sistema de antiv�rus e
 acredita-se estar livre de perigo.


=
Instru��es para entrar na lista, sair da lista e usar a lista em
http://www.mat.puc-rio.br/~obmlistas/obm-l.html
=


[obm-l] Re: [obm-l] Re: [obm-l] Comunicação

2023-08-26 Por tôpico Marcelo Gonda Stangler
https://chat.whatsapp.com/CNGgk3NcgwY4AFsB61COXB
Quem sabe?

On Sat, Aug 26, 2023 at 1:36 PM Esdras Muniz 
wrote:

> Seria muito legal se existisse.
>
> Em sex, 25 de ago de 2023 18:24, Priscila Santana <
> priscila@hotmail.com> escreveu:
>
>>
>> Olá!
>>
>> Existe algum grupo de discussão de questões olímpicas no WhatsApp?
>>
>> Atte.
>>
>> *Priscila S. da Paz*
>>
>>
>>
>> --
>> Esta mensagem foi verificada pelo sistema de antivírus e
>> acredita-se estar livre de perigo.
>>
>
> --
> Esta mensagem foi verificada pelo sistema de antivírus e
> acredita-se estar livre de perigo.

-- 
Esta mensagem foi verificada pelo sistema de antiv�rus e
 acredita-se estar livre de perigo.



[obm-l] Re: [obm-l] Comunicação

2023-08-26 Por tôpico Esdras Muniz
Seria muito legal se existisse.

Em sex, 25 de ago de 2023 18:24, Priscila Santana 
escreveu:

>
> Olá!
>
> Existe algum grupo de discussão de questões olímpicas no WhatsApp?
>
> Atte.
>
> *Priscila S. da Paz*
>
>
>
> --
> Esta mensagem foi verificada pelo sistema de antivírus e
> acredita-se estar livre de perigo.
>

-- 
Esta mensagem foi verificada pelo sistema de antiv�rus e
 acredita-se estar livre de perigo.



[obm-l] Re: [obm-l] Re: [obm-l] Re: teoria dos números

2023-08-13 Por tôpico Israel Meireles Chrisostomo
Muito obrigado!

Em qui., 10 de ago. de 2023 22:27, Ian Barquette <
ianbarquettelou...@gmail.com> escreveu:

> Se a função já está definida, e você quer apenas pontuar os limites dela,
> seria o conceito de imagem da função:
>
> Im(f) = (0, 1) = ]0, 1[
>
>
>
> Caso a função não esteja definida, a restrição seria o contradomínio da
> função:
>
> CD(f) = (0, 1) = ]0, 1[
>
> Ao definir a função, considerando C um conjunto qualquer,
>
> f: C -> (0, 1)
>
> Em qui., 10 de ago. de 2023 20:11, Israel Meireles Chrisostomo <
> israelmchrisost...@gmail.com> escreveu:
>
>> Como faço para definir em notação de conjuntos uma função com a
>> restrição, tipo 0> Seria (0,1]x(0,1]?
>>
>> Em qui., 10 de ago. de 2023 às 20:15, Israel Meireles Chrisostomo <
>> israelmchrisost...@gmail.com> escreveu:
>>
>>> Como faço para definir em notação de conjuntos uma função com a
>>> restrição, tipo f(x)<1
>>> Seria (0,1]x(0,1]?
>>>
>>> --
>>> Israel Meireles Chrisostomo
>>>
>>
>>
>> --
>> Israel Meireles Chrisostomo
>>
>> --
>> Esta mensagem foi verificada pelo sistema de antivírus e
>> acredita-se estar livre de perigo.
>
>
> --
> Esta mensagem foi verificada pelo sistema de antivírus e
> acredita-se estar livre de perigo.

-- 
Esta mensagem foi verificada pelo sistema de antiv�rus e
 acredita-se estar livre de perigo.



[obm-l] Re: [obm-l] Re: teoria dos números

2023-08-10 Por tôpico Ian Barquette
Se a função já está definida, e você quer apenas pontuar os limites dela,
seria o conceito de imagem da função:

Im(f) = (0, 1) = ]0, 1[



Caso a função não esteja definida, a restrição seria o contradomínio da
função:

CD(f) = (0, 1) = ]0, 1[

Ao definir a função, considerando C um conjunto qualquer,

f: C -> (0, 1)

Em qui., 10 de ago. de 2023 20:11, Israel Meireles Chrisostomo <
israelmchrisost...@gmail.com> escreveu:

> Como faço para definir em notação de conjuntos uma função com a restrição,
> tipo 0 Seria (0,1]x(0,1]?
>
> Em qui., 10 de ago. de 2023 às 20:15, Israel Meireles Chrisostomo <
> israelmchrisost...@gmail.com> escreveu:
>
>> Como faço para definir em notação de conjuntos uma função com a
>> restrição, tipo f(x)<1
>> Seria (0,1]x(0,1]?
>>
>> --
>> Israel Meireles Chrisostomo
>>
>
>
> --
> Israel Meireles Chrisostomo
>
> --
> Esta mensagem foi verificada pelo sistema de antivírus e
> acredita-se estar livre de perigo.

-- 
Esta mensagem foi verificada pelo sistema de antiv�rus e
 acredita-se estar livre de perigo.



[obm-l] Re: teoria dos números

2023-08-10 Por tôpico Israel Meireles Chrisostomo
Como faço para definir em notação de conjuntos uma função com a restrição,
tipo 0 escreveu:

> Como faço para definir em notação de conjuntos uma função com a restrição,
> tipo f(x)<1
> Seria (0,1]x(0,1]?
>
> --
> Israel Meireles Chrisostomo
>


-- 
Israel Meireles Chrisostomo

-- 
Esta mensagem foi verificada pelo sistema de antiv�rus e
 acredita-se estar livre de perigo.



[obm-l] Re: [obm-l] Sugestão para alunos do 6° ano

2023-07-01 Por tôpico Pazó 0192
Acho que os livros da SBM são muito bons para quem está começando:

Primeiros passos em Geometria
Primeiros passos em Combinatória, Aritmética e Álgebra
Problemas Olímpicos de Matemática para o Nível 1

On Fri, Jun 30, 2023 at 10:12 PM Marcelo Salhab Brogliato <
msbro...@gmail.com> wrote:

> Pessoal, tenho uma prima que está no 6° ano e adora matemática. Acabei de
> ensinar algumas brincadeiras de adivinhar o número usando álgebra básica e
> ela adorou!
>
> Alguém tem alguma recomendação de livros que possam incentivá-la ainda
> mais com matemática?
>
> Eu pensei no Círculos Matemáticos A Experiência Russa, mas não me parece
> ser adequado para a idade dela. Já comprei o conjunto de Matemática
> Elementar do Iezzi mas ainda não é a hora.
>
> Abraços,
> Salhab
>
> --
> Esta mensagem foi verificada pelo sistema de antivírus e
> acredita-se estar livre de perigo.

-- 
Esta mensagem foi verificada pelo sistema de antiv�rus e
 acredita-se estar livre de perigo.



[obm-l] Re: [obm-l] Re: [obm-l] Uma recorrência diferente

2023-04-06 Por tôpico Esdras Muniz
Vc pode pegar a função geratriz e usar a fórmula de Ramanujan pra calcular
o termo geral. Acredito que a função geratriz seja: e^{x^2/2+x}, já na
forma (x_n)x^n/n!...

Em qui, 6 de abr de 2023 19:03, Carlos Gustavo Tamm de Araujo Moreira <
g...@impa.br> escreveu:

> Caro Vanderlei,
> Não parece haver uma fórmula fechada muito simples. Veja
> https://oeis.org/A85 para várias referências
> sobre essa sequência.
> Abraços,
> Gugu
>
> On Wed, Apr 5, 2023 at 11:41 PM Professor Vanderlei Nemitz <
> vanderma...@gmail.com> wrote:
>
>> Oi, mestres!
>>
>> Estava resolvendo um problema de combinatória e obtive essa recorrência:
>>
>> *x(n) = x(n - 1) + (n - 1).x(n - 2), com x1 = 1 e x2 = 2*.
>>
>> Por exemplo, x3 = x2 + 2.x1 e x9 = x8 + 8.x7
>>
>> Como resolver quando os coeficientes não são todos constantes?
>>
>> Apenas como curiosidade, o problema que originou a recorrência é:
>> (IME - RJ) - Um professor dá um teste surpresa para uma turma de 9
>> alunos, e diz que o teste pode ser feito sozinho ou em grupos de 2 alunos.
>> De quantas formas a turma pode ser organizar para fazer o teste? (Por
>> exemplo, uma turma de 3 alunos pode ser organizar de 4 formas e uma turma
>> de 4 alunos pode se organizar de 10 formas)
>>
>>
>>
>> 
>>  Não
>> contém vírus.www.avast.com
>> 
>> <#m_-6699161896707809793_m_4118911927943204904_DAB4FAD8-2DD7-40BB-A1B8-4E2AA1F9FDF2>
>>
>> --
>> Esta mensagem foi verificada pelo sistema de antivírus e
>> acredita-se estar livre de perigo.
>
>
> --
> Esta mensagem foi verificada pelo sistema de antivírus e
> acredita-se estar livre de perigo.

-- 
Esta mensagem foi verificada pelo sistema de antiv�rus e
 acredita-se estar livre de perigo.



[obm-l] Re: [obm-l] Uma recorrência diferente

2023-04-06 Por tôpico Carlos Gustavo Tamm de Araujo Moreira
Caro Vanderlei,
Não parece haver uma fórmula fechada muito simples. Veja
https://oeis.org/A85 para várias referências
sobre essa sequência.
Abraços,
Gugu

On Wed, Apr 5, 2023 at 11:41 PM Professor Vanderlei Nemitz <
vanderma...@gmail.com> wrote:

> Oi, mestres!
>
> Estava resolvendo um problema de combinatória e obtive essa recorrência:
>
> *x(n) = x(n - 1) + (n - 1).x(n - 2), com x1 = 1 e x2 = 2*.
>
> Por exemplo, x3 = x2 + 2.x1 e x9 = x8 + 8.x7
>
> Como resolver quando os coeficientes não são todos constantes?
>
> Apenas como curiosidade, o problema que originou a recorrência é:
> (IME - RJ) - Um professor dá um teste surpresa para uma turma de 9 alunos,
> e diz que o teste pode ser feito sozinho ou em grupos de 2 alunos. De
> quantas formas a turma pode ser organizar para fazer o teste? (Por exemplo,
> uma turma de 3 alunos pode ser organizar de 4 formas e uma turma de 4
> alunos pode se organizar de 10 formas)
>
>
>
> 
>  Não
> contém vírus.www.avast.com
> 
> <#m_4118911927943204904_DAB4FAD8-2DD7-40BB-A1B8-4E2AA1F9FDF2>
>
> --
> Esta mensagem foi verificada pelo sistema de antivírus e
> acredita-se estar livre de perigo.

-- 
Esta mensagem foi verificada pelo sistema de antiv�rus e
 acredita-se estar livre de perigo.



[obm-l] Re: [obm-l] Uma recorrência diferente

2023-04-06 Por tôpico Anderson Torres
Em qua., 5 de abr. de 2023 às 23:40, Professor Vanderlei Nemitz <
vanderma...@gmail.com> escreveu:

> Oi, mestres!
>
> Estava resolvendo um problema de combinatória e obtive essa recorrência:
>
> *x(n) = x(n - 1) + (n - 1).x(n - 2), com x1 = 1 e x2 = 2*.
>
> Por exemplo, x3 = x2 + 2.x1 e x9 = x8 + 8.x7
>
> Como resolver quando os coeficientes não são todos constantes?
>

Nem sempre dá para esperar que uma recorrência seja "resolvível".

Você pode ir tabelando e graficando alguns valores para ver se surge alguma
coisa, e daí imaginar alguma fórmula, algo como "bem, isso aqui parece um
gráfico de um fatorial"...

E, na real, nem sempre é tão útil ter uma fórmula pronta. Um computador
pode muito bem calcular os termos dessa sequência, em um processo idêntico
ao que um ser humano faria só que mais rápido (e com menos chance de
errar). E o vestibular não exigiu calcular isso para n=100, mas apenas para
n=9.

Todavia, é uma boa fuçar. Pensei em algo como usar uma sequência auxiliar,
y(n)=x(n+1)/x(n). Isso nos daria uma outra recorrência, a saber,

y(n)=1+(n/y(n-1))

O que estranhamente gera uma espécie de fração contínua. Mas não consigo
imaginar algo muito melhor que isso.



>
> Apenas como curiosidade, o problema que originou a recorrência é:
> (IME - RJ) - Um professor dá um teste surpresa para uma turma de 9 alunos,
> e diz que o teste pode ser feito sozinho ou em grupos de 2 alunos. De
> quantas formas a turma pode ser organizar para fazer o teste? (Por exemplo,
> uma turma de 3 alunos pode ser organizar de 4 formas e uma turma de 4
> alunos pode se organizar de 10 formas)
>

>
>
> 
>  Não
> contém vírus.www.avast.com
> 
> <#m_-8406560445346393254_DAB4FAD8-2DD7-40BB-A1B8-4E2AA1F9FDF2>
>
> --
> Esta mensagem foi verificada pelo sistema de antivírus e
> acredita-se estar livre de perigo.

-- 
Esta mensagem foi verificada pelo sistema de antiv�rus e
 acredita-se estar livre de perigo.



[obm-l] Re: [obm-l] Seria por distribuição binomial ou alguma recorrência

2023-02-28 Por tôpico Anderson Torres
Em ter, 28 de fev de 2023 11:52, Bianca Flores 
escreveu:

> Alguém poderia ajudar com essa questão: estou frustrada porque não consigo
> chegar ao gabarito E.
>
> Um estudante preenche, aleatoriamente e de forma independente cada uma das
> questões, um exame de múltipla escolha com 5 respostas possíveis (das quais
> apenas uma é correta) para cada uma de 25 questões. A probabilidade que ele
> acerte um número par de questões é dada por:
>
> (A)(1-(4/5)^25)/2
> (B)(1-(3/5)^25)/2
> (C)((3/5)^25)/2
> (D)(1+(4/5)^25)/2
> (E)(1+(3/5)^25)/2
>

Bem, a probabilidade de acertar k questões de n é C(n,k) (1/5)^k
(4/5)^(n-k).
Soma isso para todo k par (lá ele) e vê no que dá.


>
> Tento de todas as formar usar a distribuição binomial, alguma recorrência,
> mas sem sucesso.
> Bianca
>
> --
> Esta mensagem foi verificada pelo sistema de antivírus e
> acredita-se estar livre de perigo.
>

-- 
Esta mensagem foi verificada pelo sistema de antiv�rus e
 acredita-se estar livre de perigo.



[obm-l] Re: [obm-l] Seria por distribuição binomial ou alguma recorrência

2023-02-28 Por tôpico joao pedro b menezes
A probabilidade do estudante acertar um número n de questões é [ (1/5)^n *
(4/5)^(25-n) ] * n!*(25-n)!/25! . ( o primeiro segmento, separo por [
...], indica a probabilidade de ele acertar n questões em uma ordem
definida, enquanto a segunda parte se refere ao número de combinações
possíveis em que ele acerta n questões ) . Agora é somar e fatorar

On Tue, Feb 28, 2023, 11:52 Bianca Flores  wrote:

> Alguém poderia ajudar com essa questão: estou frustrada porque não consigo
> chegar ao gabarito E.
>
> Um estudante preenche, aleatoriamente e de forma independente cada uma das
> questões, um exame de múltipla escolha com 5 respostas possíveis (das quais
> apenas uma é correta) para cada uma de 25 questões. A probabilidade que ele
> acerte um número par de questões é dada por:
>
> (A)(1-(4/5)^25)/2
> (B)(1-(3/5)^25)/2
> (C)((3/5)^25)/2
> (D)(1+(4/5)^25)/2
> (E)(1+(3/5)^25)/2
>
> Tento de todas as formar usar a distribuição binomial, alguma recorrência,
> mas sem sucesso.
> Bianca
>
> --
> Esta mensagem foi verificada pelo sistema de antivírus e
> acredita-se estar livre de perigo.
>

-- 
Esta mensagem foi verificada pelo sistema de antiv�rus e
 acredita-se estar livre de perigo.



[obm-l] Re: [obm-l] Seria por distribuição binomial ou alguma recorrência

2023-02-28 Por tôpico Esdras Muniz
Letra E na verdade 😅

Em ter., 28 de fev. de 2023 às 11:52, Bianca Flores 
escreveu:

> Alguém poderia ajudar com essa questão: estou frustrada porque não consigo
> chegar ao gabarito E.
>
> Um estudante preenche, aleatoriamente e de forma independente cada uma das
> questões, um exame de múltipla escolha com 5 respostas possíveis (das quais
> apenas uma é correta) para cada uma de 25 questões. A probabilidade que ele
> acerte um número par de questões é dada por:
>
> (A)(1-(4/5)^25)/2
> (B)(1-(3/5)^25)/2
> (C)((3/5)^25)/2
> (D)(1+(4/5)^25)/2
> (E)(1+(3/5)^25)/2
>
> Tento de todas as formar usar a distribuição binomial, alguma recorrência,
> mas sem sucesso.
> Bianca
>
> --
> Esta mensagem foi verificada pelo sistema de antivírus e
> acredita-se estar livre de perigo.
>
-- 
Esdras Muniz Mota
Mestrando em Matemática
Universidade Federal do Ceará

-- 
Esta mensagem foi verificada pelo sistema de antiv�rus e
 acredita-se estar livre de perigo.



[obm-l] Re: [obm-l] Seria por distribuição binomial ou alguma recorrência

2023-02-28 Por tôpico Esdras Muniz
Veja 1 como a soma de todas as probabilidades possíveis e (3/5)^25 como a
probabilidade de ele acertas uma quantidade par menos a probabilidade de
ele acertar uma quantidade ímpar.

Em ter., 28 de fev. de 2023 às 11:58, Esdras Muniz <
esdrasmunizm...@gmail.com> escreveu:

> Acredito que é letra B. Vc pode fazer usando binômio de Newton…
>
> Em ter., 28 de fev. de 2023 às 11:52, Bianca Flores 
> escreveu:
>
>> Alguém poderia ajudar com essa questão: estou frustrada porque não
>> consigo chegar ao gabarito E.
>>
>> Um estudante preenche, aleatoriamente e de forma independente cada uma
>> das questões, um exame de múltipla escolha com 5 respostas possíveis (das
>> quais apenas uma é correta) para cada uma de 25 questões. A probabilidade
>> que ele acerte um número par de questões é dada por:
>>
>> (A)(1-(4/5)^25)/2
>> (B)(1-(3/5)^25)/2
>> (C)((3/5)^25)/2
>> (D)(1+(4/5)^25)/2
>> (E)(1+(3/5)^25)/2
>>
>> Tento de todas as formar usar a distribuição binomial, alguma
>> recorrência, mas sem sucesso.
>> Bianca
>>
>> --
>> Esta mensagem foi verificada pelo sistema de antivírus e
>> acredita-se estar livre de perigo.
>>
> --
> Esdras Muniz Mota
> Mestrando em Matemática
> Universidade Federal do Ceará
>
>
> --
Esdras Muniz Mota
Mestrando em Matemática
Universidade Federal do Ceará

-- 
Esta mensagem foi verificada pelo sistema de antiv�rus e
 acredita-se estar livre de perigo.



[obm-l] Re: [obm-l] Seria por distribuição binomial ou alguma recorrência

2023-02-28 Por tôpico Esdras Muniz
Acredito que é letra B. Vc pode fazer usando binômio de Newton…

Em ter., 28 de fev. de 2023 às 11:52, Bianca Flores 
escreveu:

> Alguém poderia ajudar com essa questão: estou frustrada porque não consigo
> chegar ao gabarito E.
>
> Um estudante preenche, aleatoriamente e de forma independente cada uma das
> questões, um exame de múltipla escolha com 5 respostas possíveis (das quais
> apenas uma é correta) para cada uma de 25 questões. A probabilidade que ele
> acerte um número par de questões é dada por:
>
> (A)(1-(4/5)^25)/2
> (B)(1-(3/5)^25)/2
> (C)((3/5)^25)/2
> (D)(1+(4/5)^25)/2
> (E)(1+(3/5)^25)/2
>
> Tento de todas as formar usar a distribuição binomial, alguma recorrência,
> mas sem sucesso.
> Bianca
>
> --
> Esta mensagem foi verificada pelo sistema de antivírus e
> acredita-se estar livre de perigo.
>
-- 
Esdras Muniz Mota
Mestrando em Matemática
Universidade Federal do Ceará

-- 
Esta mensagem foi verificada pelo sistema de antiv�rus e
 acredita-se estar livre de perigo.



[obm-l] Re: [obm-l] Re: [obm-l] Re: [obm-l] cadeira de 3 pés

2023-01-23 Por tôpico Bernardo Freitas Paulo da Costa
On Mon, Jan 23, 2023 at 12:52 PM Claudio Buffara
 wrote:
>
> Obrigado, Wagner e Ponce:
>
> Eu tinha pensado em algo na linha do que o Ponce escreveu, ainda que em 
> certos casos patológicos (pelo menos de piso...) o terceiro pé pode não 
> encontrar apoio: imagine um piso com um pico fino em algum lugar (p.ex. a 
> superfície gerada pela revolução de z = 1/(1+(x^2+y^2)^20) em torno do eixo 
> z). Neste caso, precisaríamos de uma cadeira bem pequena, ou pelo menos com 
> as pontas dos pés bem próximas umas das outras.

Pois é, eu acho que tem uma questão interessante de estabilidade que é
a seguinte: se você pegar um banco de 3 pés e botar na encosta de uma
montanha (bem íngreme!), ele vai rolar ladeira abaixo.  Porque a
projeção do centro de gravidade cai fora do triângulo determinado pelo
contato com o solo (que define as normais que podem segurar a cadeira
em posição estática).  Uma cadeira de 4 pés só é bamba porque uma
pessoa não consegue ficar (infinitesimalmente) parada, e daí o centro
de gravidade oscila entre os triângulos formados por dois pares de
três apoios, por exemplo ABC e ACD.

Bernardo

-- 
Esta mensagem foi verificada pelo sistema de antiv�rus e
 acredita-se estar livre de perigo.


=
Instru��es para entrar na lista, sair da lista e usar a lista em
http://www.mat.puc-rio.br/~obmlistas/obm-l.html
=


[obm-l] Re: [obm-l] Re: [obm-l] cadeira de 3 pés

2023-01-23 Por tôpico Claudio Buffara
" No mundo 2D um banco de 2 pernas não balança, mas um de 3 pernas pode
balançar."

Aqui estamos falando de uma curva no plano e de um número real positivo d,
suficientemente pequeno de modo que, para todo ponto A na curva, existe um
ponto B na curva com dist(A,B) = d.
A e B são as pontas das pernas do banco 2D.
Isso será verdade se todo ponto da curva for o centro de uma circunferência
de raio d que intersecta a curva.

Um terceiro ponto C, a distâncias predeterminadas de A e B, pode não
pertencer à curva (ou seja, o triângulo ABC não estará inscrito na curva).
Mas é possível deslizar a corda AB, de comprimento constante, ao longo de
toda a curva.

No caso 3D, temos uma superfície no espaço e 3 números reais positivos a,
b, c, com cada um menor do que a soma dos outros dois, de modo que, para
cada A na superfície, é possível achar B e C na superfície com dist(A,B) =
c, dist(A,C) = b e dist(B,C) = a.
E é possível deslizar o triângulo ABC, com lados de comprimento constante,
ao longo de toda a superfície.
Mas, fixado A na superfície, não me parece tão simples construir B e C.


On Mon, Jan 23, 2023 at 11:29 AM Eduardo Wagner  wrote:

> O banco de 3 pernas não balança porque nosso mundo é tridimensional.
> Não tem nada a ver com plano ou triângulo. Um banco de 3 pernas não
> balança se for colocado
> no teto de um carro.
> No mundo 2D um banco de 2 pernas não balança, mas um de 3 pernas pode
> balançar.
> Em um mundo 4D uma cadeira de 4 pernas não balança.
> Wbs
> Wagner
>
>
> Em dom., 22 de jan. de 2023 às 23:24, Claudio Buffara <
> claudio.buff...@gmail.com> escreveu:
>
>> Achei na internet duas explicações distintas para a estabilidade de uma
>> cadeira (ou mesa ou banco) de 3 pés.
>> Aqui estão:
>> https://www.somatematica.com.br/curiosidades/c98.php
>>
>> http://colegiofarroupilha.com.br/site/qual-cadeira-e-mais-firme-a-que-tem-tres-ou-quatro-pes/
>>
>> Qual das duas é a explicação correta?
>> Ou nenhuma das duas? E, nesse caso, qual a explicação?
>>
>> []s,
>> Claudio.
>>
>> --
>> Esta mensagem foi verificada pelo sistema de antivírus e
>> acredita-se estar livre de perigo.
>
>
> --
> Esta mensagem foi verificada pelo sistema de antivírus e
> acredita-se estar livre de perigo.

-- 
Esta mensagem foi verificada pelo sistema de antiv�rus e
 acredita-se estar livre de perigo.



[obm-l] Re: [obm-l] Re: [obm-l] cadeira de 3 pés

2023-01-23 Por tôpico Claudio Buffara
Obrigado, Wagner e Ponce:

Eu tinha pensado em algo na linha do que o Ponce escreveu, ainda que em
certos casos patológicos (pelo menos de piso...) o terceiro pé pode não
encontrar apoio: imagine um piso com um pico fino em algum lugar (p.ex. a
superfície gerada pela revolução de z = 1/(1+(x^2+y^2)^20) em torno do eixo
z). Neste caso, precisaríamos de uma cadeira bem pequena, ou pelo menos com
as pontas dos pés bem próximas umas das outras.

Mas, pelo que o Wagner escreveu, acho que ainda tem um teorema mais
profundo aí.

[]s,
Claudio.


On Mon, Jan 23, 2023 at 11:54 AM Rogerio Ponce  wrote:

> Ola' Claudio!
> Eu diria que as duas explicações estão erradas, pois não se depende de
> ter apenas um plano definido pelas pontas dos pés, visto que uma
> cadeira de 4 pés pode, perfeitamente, ter as pontas dos 4 pés em
> apenas um plano, e, ainda assim, ela não é necessariamente estável.
>
> Um explicação menos ruim é que, numa cadeira de 3 pés, sempre podemos
> apoiar quaisquer 2 pés num piso (mesmo irregular), e, em torno do eixo
> definido pelos 2 pés já apoiados, podemos girar a cadeira até que o
> terceiro pé encontre o piso, de modo que a cadeira fique totalmente
> apoiada.
> Já numa cadeira de 4 pés, é comum que um dos pés fique sem contato com
> o chão, permitindo que a cadeira oscile em torno do eixo definido
> pelos 2 pés vizinhos ao pé sem contato.
>
> []'s
> Rogerio Ponce
>
> On Sun, Jan 22, 2023 at 11:23 PM Claudio Buffara
>  wrote:
> >
> > Achei na internet duas explicações distintas para a estabilidade de uma
> cadeira (ou mesa ou banco) de 3 pés.
> > Aqui estão:
> > https://www.somatematica.com.br/curiosidades/c98.php
> >
> http://colegiofarroupilha.com.br/site/qual-cadeira-e-mais-firme-a-que-tem-tres-ou-quatro-pes/
> >
> > Qual das duas é a explicação correta?
> > Ou nenhuma das duas? E, nesse caso, qual a explicação?
> >
> > []s,
> > Claudio.
> >
> > --
> > Esta mensagem foi verificada pelo sistema de antivírus e
> > acredita-se estar livre de perigo.
>
> --
> Esta mensagem foi verificada pelo sistema de antivírus e
>  acredita-se estar livre de perigo.
>
>
> =
> Instru�ões para entrar na lista, sair da lista e usar a lista em
> http://www.mat.puc-rio.br/~obmlistas/obm-l.html
> =
>

-- 
Esta mensagem foi verificada pelo sistema de antiv�rus e
 acredita-se estar livre de perigo.



[obm-l] Re: [obm-l] Re: [obm-l] cadeira de 3 pés

2023-01-23 Por tôpico Anderson Torres
Em seg, 23 de jan de 2023 11:54, Rogerio Ponce 
escreveu:

> Ola' Claudio!
> Eu diria que as duas explicações estão erradas, pois não se depende de
> ter apenas um plano definido pelas pontas dos pés, visto que uma
> cadeira de 4 pés pode, perfeitamente, ter as pontas dos 4 pés em
> apenas um plano, e, ainda assim, ela não é necessariamente estável.
>

Como não? A ideia de botar um calço é precisamente estabilizar o pé que não
encosta na superfície. E, exceto em feiras de ciências e circos, o chão
costuma ser plano.


> Um explicação menos ruim é que, numa cadeira de 3 pés, sempre podemos
> apoiar quaisquer 2 pés num piso (mesmo irregular), e, em torno do eixo
> definido pelos 2 pés já apoiados, podemos girar a cadeira até que o
> terceiro pé encontre o piso, de modo que a cadeira fique totalmente
> apoiada.
> Já numa cadeira de 4 pés, é comum que um dos pés fique sem contato com
> o chão, permitindo que a cadeira oscile em torno do eixo definido
> pelos 2 pés vizinhos ao pé sem contato.
>
> []'s
> Rogerio Ponce
>
> On Sun, Jan 22, 2023 at 11:23 PM Claudio Buffara
>  wrote:
> >
> > Achei na internet duas explicações distintas para a estabilidade de uma
> cadeira (ou mesa ou banco) de 3 pés.
> > Aqui estão:
> > https://www.somatematica.com.br/curiosidades/c98.php
> >
> http://colegiofarroupilha.com.br/site/qual-cadeira-e-mais-firme-a-que-tem-tres-ou-quatro-pes/
> >
> > Qual das duas é a explicação correta?
> > Ou nenhuma das duas? E, nesse caso, qual a explicação?
> >
> > []s,
> > Claudio.
> >
> > --
> > Esta mensagem foi verificada pelo sistema de antivírus e
> > acredita-se estar livre de perigo.
>
> --
> Esta mensagem foi verificada pelo sistema de antivírus e
>  acredita-se estar livre de perigo.
>
>
> =
> Instru�ões para entrar na lista, sair da lista e usar a lista em
> http://www.mat.puc-rio.br/~obmlistas/obm-l.html
> =
>

-- 
Esta mensagem foi verificada pelo sistema de antiv�rus e
 acredita-se estar livre de perigo.



[obm-l] Re: [obm-l] Re: [obm-l] Re: [obm-l] cadeira de 3 pés

2023-01-23 Por tôpico Anderson Torres
Em seg, 23 de jan de 2023 11:15, Claudio Buffara 
escreveu:

> Será que o argumento usando apenas o plano é suficiente?  Pois um banco de
> 3 pés também fica estável num piso irregular.
>

Mas nem toda cadeira de quatro pés fica estável em qualquer piso irregular.

A ideia subjacente ainda é a de "planidade". Os pés da cadeira estão na
intersecção entre o "plano dos pés" e o piso.
Mas o mesmo não se aplica às cadeiras quadrúpedes, pois nem sempre existe
um "plano dos pés".

Se bem que neste último caso é possível que os pés da mesa sejam coplanares
mas a superfície não o seja. Ou pior ainda, forçando um pouco na
continuidade, pode ser que qualquer cadeira de quatro pés se encaixe em
qualquer superfície não-plana.

Ou não. Se pegarmos por exemplo o mapa topográfico de uma planície com um
poço escavado e a distância entre dois pés da mesa sempre for maior que o
diâmetro do poço, não tem como encaixar uma mesa torta de quatro pés.

Estranho...

O resultado mais geral em que pensei foi o seguinte: dada qualquer
> superfície bi-dimensional contínua (por exemplo, que seja o gráfico de uma
> função contínua de RxR em R - uma suposição razoável se estamos tentando
> modelar um piso), você sempre consegue encostar nela as pontas dos 3 pés do
> banco, de modo que o banco fique "firme" ou sem folgas.
> Ou, mais formalmente, dado um triângulo ABC no espaço, existe uma
> isometria (do espaço) tal que as imagens de A, B e C por esta isometria
> estão em S.
>

Pensei algo do gênero, mas de maneira bem menos elaborada: dados um
triângulo ABC e dois pontos A',B' em uma superfície contínua que distam AB,
existe um ponto C´ tal que ABC = A'B'C'


> Enfim, esse talvez seja um problema mais de topologia do que de geometria.
> Pois, no fim das contas, "3 pontos não colineares determinam um único
> plano", assim como "2 pontos determinam uma única reta" são afirmações que
> têm um certo ar topológico, pelo menos pra mim.
>

> []s,
> Claudio.
>
> On Mon, Jan 23, 2023 at 7:02 AM Anderson Torres <
> torres.anderson...@gmail.com> wrote:
>
>>
>>
>> Em dom, 22 de jan de 2023 23:23, Claudio Buffara <
>> claudio.buff...@gmail.com> escreveu:
>>
>>> Achei na internet duas explicações distintas para a estabilidade de uma
>>> cadeira (ou mesa ou banco) de 3 pés.
>>> Aqui estão:
>>> https://www.somatematica.com.br/curiosidades/c98.php
>>>
>>> http://colegiofarroupilha.com.br/site/qual-cadeira-e-mais-firme-a-que-tem-tres-ou-quatro-pes/
>>>
>>
>> Nesse caso específico, a primeira me parece mais correta. Ou melhor, a
>> segunda tem falhas.
>>
>> A rigidez dos triângulos (TCC caso LLL de igualdade de triângulos) é
>> irrelevante para a questão da cadeira bamba. Em qualquer cadeira física e
>> palpável, as pernas são rígidas - portanto o polígono formado pelas
>> extremidades dessas pernas é rígido também.
>>
>> Mas um polígono rígido não é necessariamente um polígono bidimensional -
>> o que a cadeira bamba de 4 pernas exemplifica perfeitamente.
>>
>> Contate o webmaster da segunda página sugerindo correções!
>>
>>
>>> Qual das duas é a explicação correta?
>>> Ou nenhuma das duas? E, nesse caso, qual a explicação?
>>>
>>> []s,
>>> Claudio.
>>>
>>> --
>>> Esta mensagem foi verificada pelo sistema de antivírus e
>>> acredita-se estar livre de perigo.
>>
>>
>> --
>> Esta mensagem foi verificada pelo sistema de antivírus e
>> acredita-se estar livre de perigo.
>
>
> --
> Esta mensagem foi verificada pelo sistema de antivírus e
> acredita-se estar livre de perigo.

-- 
Esta mensagem foi verificada pelo sistema de antiv�rus e
 acredita-se estar livre de perigo.



[obm-l] Re: [obm-l] cadeira de 3 pés

2023-01-23 Por tôpico Rogerio Ponce
Ola' Claudio!
Eu diria que as duas explicações estão erradas, pois não se depende de
ter apenas um plano definido pelas pontas dos pés, visto que uma
cadeira de 4 pés pode, perfeitamente, ter as pontas dos 4 pés em
apenas um plano, e, ainda assim, ela não é necessariamente estável.

Um explicação menos ruim é que, numa cadeira de 3 pés, sempre podemos
apoiar quaisquer 2 pés num piso (mesmo irregular), e, em torno do eixo
definido pelos 2 pés já apoiados, podemos girar a cadeira até que o
terceiro pé encontre o piso, de modo que a cadeira fique totalmente
apoiada.
Já numa cadeira de 4 pés, é comum que um dos pés fique sem contato com
o chão, permitindo que a cadeira oscile em torno do eixo definido
pelos 2 pés vizinhos ao pé sem contato.

[]'s
Rogerio Ponce

On Sun, Jan 22, 2023 at 11:23 PM Claudio Buffara
 wrote:
>
> Achei na internet duas explicações distintas para a estabilidade de uma 
> cadeira (ou mesa ou banco) de 3 pés.
> Aqui estão:
> https://www.somatematica.com.br/curiosidades/c98.php
> http://colegiofarroupilha.com.br/site/qual-cadeira-e-mais-firme-a-que-tem-tres-ou-quatro-pes/
>
> Qual das duas é a explicação correta?
> Ou nenhuma das duas? E, nesse caso, qual a explicação?
>
> []s,
> Claudio.
>
> --
> Esta mensagem foi verificada pelo sistema de antivírus e
> acredita-se estar livre de perigo.

-- 
Esta mensagem foi verificada pelo sistema de antiv�rus e
 acredita-se estar livre de perigo.


=
Instru��es para entrar na lista, sair da lista e usar a lista em
http://www.mat.puc-rio.br/~obmlistas/obm-l.html
=


[obm-l] Re: [obm-l] cadeira de 3 pés

2023-01-23 Por tôpico Eduardo Wagner
O banco de 3 pernas não balança porque nosso mundo é tridimensional.
Não tem nada a ver com plano ou triângulo. Um banco de 3 pernas não balança
se for colocado
no teto de um carro.
No mundo 2D um banco de 2 pernas não balança, mas um de 3 pernas pode
balançar.
Em um mundo 4D uma cadeira de 4 pernas não balança.
Wbs
Wagner


Em dom., 22 de jan. de 2023 às 23:24, Claudio Buffara <
claudio.buff...@gmail.com> escreveu:

> Achei na internet duas explicações distintas para a estabilidade de uma
> cadeira (ou mesa ou banco) de 3 pés.
> Aqui estão:
> https://www.somatematica.com.br/curiosidades/c98.php
>
> http://colegiofarroupilha.com.br/site/qual-cadeira-e-mais-firme-a-que-tem-tres-ou-quatro-pes/
>
> Qual das duas é a explicação correta?
> Ou nenhuma das duas? E, nesse caso, qual a explicação?
>
> []s,
> Claudio.
>
> --
> Esta mensagem foi verificada pelo sistema de antivírus e
> acredita-se estar livre de perigo.

-- 
Esta mensagem foi verificada pelo sistema de antiv�rus e
 acredita-se estar livre de perigo.



[obm-l] Re: [obm-l] Re: [obm-l] cadeira de 3 pés

2023-01-23 Por tôpico Claudio Buffara
Será que o argumento usando apenas o plano é suficiente?  Pois um banco de
3 pés também fica estável num piso irregular.
O resultado mais geral em que pensei foi o seguinte: dada qualquer
superfície bi-dimensional contínua (por exemplo, que seja o gráfico de uma
função contínua de RxR em R - uma suposição razoável se estamos tentando
modelar um piso), você sempre consegue encostar nela as pontas dos 3 pés do
banco, de modo que o banco fique "firme" ou sem folgas.
Ou, mais formalmente, dado um triângulo ABC no espaço, existe uma isometria
(do espaço) tal que as imagens de A, B e C por esta isometria estão em S.

Enfim, esse talvez seja um problema mais de topologia do que de geometria.
Pois, no fim das contas, "3 pontos não colineares determinam um único
plano", assim como "2 pontos determinam uma única reta" são afirmações que
têm um certo ar topológico, pelo menos pra mim.

[]s,
Claudio.

On Mon, Jan 23, 2023 at 7:02 AM Anderson Torres <
torres.anderson...@gmail.com> wrote:

>
>
> Em dom, 22 de jan de 2023 23:23, Claudio Buffara <
> claudio.buff...@gmail.com> escreveu:
>
>> Achei na internet duas explicações distintas para a estabilidade de uma
>> cadeira (ou mesa ou banco) de 3 pés.
>> Aqui estão:
>> https://www.somatematica.com.br/curiosidades/c98.php
>>
>> http://colegiofarroupilha.com.br/site/qual-cadeira-e-mais-firme-a-que-tem-tres-ou-quatro-pes/
>>
>
> Nesse caso específico, a primeira me parece mais correta. Ou melhor, a
> segunda tem falhas.
>
> A rigidez dos triângulos (TCC caso LLL de igualdade de triângulos) é
> irrelevante para a questão da cadeira bamba. Em qualquer cadeira física e
> palpável, as pernas são rígidas - portanto o polígono formado pelas
> extremidades dessas pernas é rígido também.
>
> Mas um polígono rígido não é necessariamente um polígono bidimensional - o
> que a cadeira bamba de 4 pernas exemplifica perfeitamente.
>
> Contate o webmaster da segunda página sugerindo correções!
>
>
>> Qual das duas é a explicação correta?
>> Ou nenhuma das duas? E, nesse caso, qual a explicação?
>>
>> []s,
>> Claudio.
>>
>> --
>> Esta mensagem foi verificada pelo sistema de antivírus e
>> acredita-se estar livre de perigo.
>
>
> --
> Esta mensagem foi verificada pelo sistema de antivírus e
> acredita-se estar livre de perigo.

-- 
Esta mensagem foi verificada pelo sistema de antiv�rus e
 acredita-se estar livre de perigo.



[obm-l] Re: [obm-l] cadeira de 3 pés

2023-01-23 Por tôpico Anderson Torres
Em dom, 22 de jan de 2023 23:23, Claudio Buffara 
escreveu:

> Achei na internet duas explicações distintas para a estabilidade de uma
> cadeira (ou mesa ou banco) de 3 pés.
> Aqui estão:
> https://www.somatematica.com.br/curiosidades/c98.php
>
> http://colegiofarroupilha.com.br/site/qual-cadeira-e-mais-firme-a-que-tem-tres-ou-quatro-pes/
>

Nesse caso específico, a primeira me parece mais correta. Ou melhor, a
segunda tem falhas.

A rigidez dos triângulos (TCC caso LLL de igualdade de triângulos) é
irrelevante para a questão da cadeira bamba. Em qualquer cadeira física e
palpável, as pernas são rígidas - portanto o polígono formado pelas
extremidades dessas pernas é rígido também.

Mas um polígono rígido não é necessariamente um polígono bidimensional - o
que a cadeira bamba de 4 pernas exemplifica perfeitamente.

Contate o webmaster da segunda página sugerindo correções!


> Qual das duas é a explicação correta?
> Ou nenhuma das duas? E, nesse caso, qual a explicação?
>
> []s,
> Claudio.
>
> --
> Esta mensagem foi verificada pelo sistema de antivírus e
> acredita-se estar livre de perigo.

-- 
Esta mensagem foi verificada pelo sistema de antiv�rus e
 acredita-se estar livre de perigo.



[obm-l] Re: [obm-l] Mostrar que [n!]/e é sempre par

2022-12-15 Por tôpico Anderson Torres
Em sex, 16 de dez de 2022 00:53, Artur Costa Steiner <
artur.costa.stei...@gmail.com> escreveu:

> Problema interessante: Mostre que, para todo inteiro n >= 0,  [n!]/e é
> sempre par, sendo [x] o piso de x.
>

você quis dizer [n!/e] onde e é a base do log natural?

Bem, 1/e=e^(-1)=
(1/0!-1/1!)+(1/2!-1/3!)+(1/4!-1/5!)+(1/6!-1/7!)+... =
2/3! + 4/5! + 6/7! + 8/9! + ...

Dessa forma, [n!/e]=

[2n!/3! + 4n!/5! + 6n!/7! + 8n!/9! + ...]

Que, obviamente, redunda numa sema de pares após truncado (denominador
>n!),


> Abraços
>
> Artur
>
> --
> Esta mensagem foi verificada pelo sistema de antivírus e
> acredita-se estar livre de perigo.

-- 
Esta mensagem foi verificada pelo sistema de antiv�rus e
 acredita-se estar livre de perigo.



[obm-l] Re: [obm-l] Teoria dos números, trigonometria e racionalidade

2022-12-11 Por tôpico Anderson Torres
Em dom., 11 de dez. de 2022 às 10:32, Anderson Torres
 escreveu:
>
> Em sáb., 10 de dez. de 2022 às 22:08, marcone augusto araújo borges
>  escreveu:
> >
> > Seja p um número primo tal que p = = 3 (mod4) e @ um ângulo tal que tan@ é 
> > racional. Prove que tan((p+1)@) também é racional com numerador múltiplo de 
> > p
> > Desde já agradeço por algum esclarecimento ou solução.
>
> Bem, o que eu consigo pensar é em algo desse tipo.
>
> Sabemos que tan(m+n) = (tan(m) + tan(n))/(1-tan(m)* tan(n))
>
> Escrevamos tan(nX)=p(n)/q(n), onde p e q são polinômios em t=tan(X).
> Temos então a seguinte recorrência:
>
> p(1)=t; p(n+1)=p(n)+tq(n)
> q(1)=1; q(n+1)=-tp(n)+q(n)
>
> Jogando aqui e ali, temos
>
> p(1)=t; p(2)=2t; p(n+2)=2p(n+1)-(t^2+1)p(n)
> q(1)=1; q(2)=1-t^2; q(n+2)=2q(n+1)-(t^2+1)q(n)
>
> De cara, se nota que p sempre será múltiplo de p, e que q sempre deixa
> resto 1 módulo t, o que já dá uma pista do que procurar...
> Decerto, vai aparecer alguma coisa do tipo x^2+1, e com isso se usa o
> fato de p ser primo da forma 4k-1...
>

Acho que dá para melhorar. Suponha tan(nX)=A(n)/B(n). Assim,

A(n+1) =  B*A(n) + A*B(n)
B(n+1) = -A*A(n) + B*B(n)

E portanto

A(n+2) = 2B*A(n+1) - (A^2+B^2)*A(n), A(1)=A, A(2)=2AB
B(n+2) = 2B*B(n+1) - (A^2+B^2)*B(n), B(1)=B, B(2)=B^2-A^2

A ideia então seria demonstrar que A(p+1) é múltiplo de p para p primo
da forma 4k-1, e B(p+1) não é múltiplo de p para p primo da forma
4k-1.

Dessa forma, ao menos em princípio seria possível verificar a segunda
premissa, pois a primeira é óbvia.

> >
> > --
> > Esta mensagem foi verificada pelo sistema de antivírus e
> > acredita-se estar livre de perigo.

-- 
Esta mensagem foi verificada pelo sistema de antiv�rus e
 acredita-se estar livre de perigo.


=
Instru��es para entrar na lista, sair da lista e usar a lista em
http://www.mat.puc-rio.br/~obmlistas/obm-l.html
=


[obm-l] Re: [obm-l] OBM e Olímpiadas internacionais

2022-12-11 Por tôpico Anderson Torres
Em qua., 7 de dez. de 2022 às 03:39, Obindinachukwu Desire Yema
 escreveu:
>
> Bom dia a todos,
> Nesse ano eu despertei um interesse em matemática pura, pensando um pouco 
> decidi que iria tentar no próximo ano fazer a OBM nivel universitário. 
> Pesquisando no site da OBM, eu não achei nada relacionado com o conteúdo que 
> cai na prova.
> Eu queria perguntar para vocês como que me preparo para a prova, no sentido 
> de: conteúdo que devo saber.
> Desde já agradeço a atenção.

De fato tem pouca coisa além das provas. Então, te sugiro pegar
pesado: estude a PUTNAM e a IMC. Com isso você vai ter mais material.

>
> --
> Esta mensagem foi verificada pelo sistema de antivírus e
> acredita-se estar livre de perigo.

-- 
Esta mensagem foi verificada pelo sistema de antiv�rus e
 acredita-se estar livre de perigo.


=
Instru��es para entrar na lista, sair da lista e usar a lista em
http://www.mat.puc-rio.br/~obmlistas/obm-l.html
=


[obm-l] Re: [obm-l] Teoria dos números, trigonometria e racionalidade

2022-12-11 Por tôpico Anderson Torres
Em sáb., 10 de dez. de 2022 às 22:08, marcone augusto araújo borges
 escreveu:
>
> Seja p um número primo tal que p = = 3 (mod4) e @ um ângulo tal que tan@ é 
> racional. Prove que tan((p+1)@) também é racional com numerador múltiplo de p
> Desde já agradeço por algum esclarecimento ou solução.

Bem, o que eu consigo pensar é em algo desse tipo.

Sabemos que tan(m+n) = (tan(m) + tan(n))/(1-tan(m)* tan(n))

Escrevamos tan(nX)=p(n)/q(n), onde p e q são polinômios em t=tan(X).
Temos então a seguinte recorrência:

p(1)=t; p(n+1)=p(n)+tq(n)
q(1)=1; q(n+1)=-tp(n)+q(n)

Jogando aqui e ali, temos

p(1)=t; p(2)=2t; p(n+2)=2p(n+1)-(t^2+1)p(n)
q(1)=1; q(2)=1-t^2; q(n+2)=2q(n+1)-(t^2+1)q(n)

De cara, se nota que p sempre será múltiplo de p, e que q sempre deixa
resto 1 módulo t, o que já dá uma pista do que procurar...
Decerto, vai aparecer alguma coisa do tipo x^2+1, e com isso se usa o
fato de p ser primo da forma 4k-1...

>
> --
> Esta mensagem foi verificada pelo sistema de antivírus e
> acredita-se estar livre de perigo.

-- 
Esta mensagem foi verificada pelo sistema de antiv�rus e
 acredita-se estar livre de perigo.


=
Instru��es para entrar na lista, sair da lista e usar a lista em
http://www.mat.puc-rio.br/~obmlistas/obm-l.html
=


[obm-l] Re: Teoria dos números

2022-12-11 Por tôpico marcone augusto araújo borges
Correção: não é (@+1)p, é (p+1)@


De: owner-ob...@mat.puc-rio.br  em nome de marcone 
augusto araújo borges 
Enviado: sábado, 10 de dezembro de 2022 07:38
Para: obm-l@mat.puc-rio.br 
Assunto: [obm-l] Teoria dos números

Seja p = = 3(mod4) um número primo e @ um ângulo tal que tan@ é racional. Prove 
que tan((@+1)p) é também racional com numerador múltiplo de p.
Se alguém puder resolver eu agradeço

--
Esta mensagem foi verificada pelo sistema de antivírus e
acredita-se estar livre de perigo.

-- 
Esta mensagem foi verificada pelo sistema de antivírus e
 acredita-se estar livre de perigo.



[obm-l] Re: [obm-l] OBM e Olímpiadas internacionais

2022-12-07 Por tôpico Claudio Buffara
Eu começaria olhando as provas de anos anteriores, por exemplo aqui:
https://www.obm.org.br/como-se-preparar/provas-e-gabaritos/

On Wed, Dec 7, 2022 at 3:39 AM Obindinachukwu Desire Yema <
obindinachukwu.y...@usp.br> wrote:

> Bom dia a todos,
> Nesse ano eu despertei um interesse em matemática pura, pensando um pouco
> decidi que iria tentar no próximo ano fazer a OBM nivel universitário.
> Pesquisando no site da OBM, eu não achei nada relacionado com o conteúdo
> que cai na prova.
> Eu queria perguntar para vocês como que me preparo para a prova, no
> sentido de: conteúdo que devo saber.
> Desde já agradeço a atenção.
>
> --
> Esta mensagem foi verificada pelo sistema de antivírus e
> acredita-se estar livre de perigo.

-- 
Esta mensagem foi verificada pelo sistema de antiv�rus e
 acredita-se estar livre de perigo.



[obm-l] Re: [obm-l] Re: [obm-l] Re: [obm-l] Caracterização de Inteiros

2022-11-17 Por tôpico Claudio Buffara
Só completando...

Apesar de números irracionais serem conhecidos desde a época de Pitágoras
(vide a famosa historinha do pitagórico Hipaso, que supostamente foi
afogado por ter "vazado" o segredo da existência dos irracionais), me
parece que eles só começaram a realmente fazer falta no século 19, quando
os matemáticos estavam preocupados em formalizar a análise. Foi naquele
ponto que o Dedekind se deu conta da importância da completeza (essa
palavra tá certa?...) dos reais e da necessidade dela ser garantida por um
axioma. Se não me engano, o teorema que ele queria provar é o de que uma
sequência monótona e limitada de números reais sempre converge. E descobriu
que não conseguiria sem fazer uma hipótese adicional sobre o corpo
dos reais, capaz de diferenciá-lo dos racionais (também um corpo ordenado),
mas dentro do qual nem toda sequência converge (por exemplo, x(n+1) =
(1/2)*(x(n) + 2/x(n)), com x(0) = 1, a qual converge pra raiz(2): um
irracional). A princípio, bastaria ele incluir o axioma de completeza dos
reais e provar que este axioma não contradiz os demais axiomas. Mas ele foi
mais longe e acabou inventando uma construção dos reais a partir dos
racionais (via os chamados "cortes de Dedekind"). Talvez (estou
conjecturando aqui) ele só tenha feito isso por uma necessidade psicológica
de provar que algo é possível (no caso a extensão dos racionais aos reais)
exibindo uma construção explícita. Isso talvez seja um reflexo da ojeriza
que matemáticos da época tinham (e alguns poucos têm até hoje) de
argumentos não construtivos pra provar a existência de fatos ou objetos
matemáticos. Por exemplo, demonstrações usando o princípio das casas de
pombos são desse tipo.

Sobre "entender sem compreender", lembrei de um ditado do John von
Neumann: "Em matemática, você nunca entende as coisas. Apenas se acostuma
com elas."

[]s,
Claudio.

On Wed, Nov 16, 2022 at 6:52 PM Claudio Buffara 
wrote:

> Não entendi como uma homotetia poderia reduzir um par ordenado a um único
> número... enfim...
>
> O que se faz, no caso da relação de equivalência que descrevi, é
> representar o par (a,b) pela notação a-b.
> Daí, (a,b) e (c,d) são equivalentes sss a-b = c-d.
> E a novidade são os números negativos: as classes de equivalência de pares
> (a,b) com a < b, representadas, por exemplo, pelo par (0,c), onde c = b-a.
> Ou, na notação usual, -c.
>
> Mas não acho que se deva perder muito tempo com a construção de sistemas
> numéricos via classes de equivalência, estendendo naturais para inteiros
> para racionais para reais e para complexos.  Até porque é extremamente
> sacal, a cada etapa, checar que as operações usuais (+ e *), quando
> aplicadas aos novos números, têm todas as propriedades que conhecemos da
> escola.
> Essas construções foram a maneira que os matemáticos acharam pra
> formalizar os sistemas numéricos, a partir de conceitos mais básicos (no
> caso, pares ordenados e relações de equivalência) - é o programa do Hilbert
> (ou de Russell e Whitehead), de reduzir toda a matemática à teoria dos
> conjuntos.
> Mas, no fundo, esta é uma construção artificial, ex post.  Pois
> matemáticos já usavam todos os números muito antes dessa formalização ser
> inventada.
> E não acho que ela renda muitos frutos, nem pedagógicos (a menos que seu
> objetivo seja "entender sem compreender") e nem pra ampliação da fronteira
> do conhecimento, exceto colocar os sistemas numéricos numa base axiomática
> sólida.
> Em particular, no que diz respeito aos números reais, a única coisa que
> interessa é que eles são um corpo ordenado completo. Tanto é que vários
> livros de análise partem deste axioma e não se preocupam em construir os
> reais a partir dos naturais.
>
> []s,
> Claudio.
>
>
>
> On Tue, Nov 15, 2022 at 5:07 PM Pedro José  wrote:
>
>> Obrigado a você e ao Cláudio. Mas não sou criativo para inventar. Mas já
>> vi que terei que fazer uma homotetia, para as classes de equivalência para
>> representar só como um número e não como um par, creio eu.
>>
>> Cordialmente,
>> PJMS
>>
>> Em ter., 15 de nov. de 2022 às 16:00, Anderson Torres <
>> torres.anderson...@gmail.com> escreveu:
>>
>>>
>>>
>>> Em ter, 15 de nov de 2022 14:33, Pedro José 
>>> escreveu:
>>>
 Boa tarde!
 Para os |Naturais, temos os postulados de Peano.

 Para os Inteiros há alguma formalização?

>>>
>>> invente uma!
>>>
>>> Pode ser por exemplo o conjunto de pares (p,q) tais que p-q é constante.
>>>
>>> ou melhor (p1,q1)=(p2,q2) se e só se p1+q2=p2+q1.
>>>
>>>
 Acho pobre dizer que é necessário ter outros números devido ao problema
 de fechamento nos naturais para a subtração que é fato e daí introduzir os
 simétricos que são inteiros e ainda não foram caracterizados.

 No meu antigo ginásio aprendi que os Reais era a união dos conjuntos
 disjuntos irracionais e racionais. Os racionais haviam sido bem definidos.
 Aí questionei e o que são irracionais? resposta: são os Reais que não são
 racionais, os que não podem ser

[obm-l] Re: [obm-l] Re: [obm-l] Re: [obm-l] Caracterização de Inteiros

2022-11-16 Por tôpico Claudio Buffara
Não entendi como uma homotetia poderia reduzir um par ordenado a um único
número... enfim...

O que se faz, no caso da relação de equivalência que descrevi, é
representar o par (a,b) pela notação a-b.
Daí, (a,b) e (c,d) são equivalentes sss a-b = c-d.
E a novidade são os números negativos: as classes de equivalência de pares
(a,b) com a < b, representadas, por exemplo, pelo par (0,c), onde c = b-a.
Ou, na notação usual, -c.

Mas não acho que se deva perder muito tempo com a construção de sistemas
numéricos via classes de equivalência, estendendo naturais para inteiros
para racionais para reais e para complexos.  Até porque é extremamente
sacal, a cada etapa, checar que as operações usuais (+ e *), quando
aplicadas aos novos números, têm todas as propriedades que conhecemos da
escola.
Essas construções foram a maneira que os matemáticos acharam pra formalizar
os sistemas numéricos, a partir de conceitos mais básicos (no caso, pares
ordenados e relações de equivalência) - é o programa do Hilbert (ou de
Russell e Whitehead), de reduzir toda a matemática à teoria dos conjuntos.
Mas, no fundo, esta é uma construção artificial, ex post.  Pois matemáticos
já usavam todos os números muito antes dessa formalização ser inventada.
E não acho que ela renda muitos frutos, nem pedagógicos (a menos que seu
objetivo seja "entender sem compreender") e nem pra ampliação da fronteira
do conhecimento, exceto colocar os sistemas numéricos numa base axiomática
sólida.
Em particular, no que diz respeito aos números reais, a única coisa que
interessa é que eles são um corpo ordenado completo. Tanto é que vários
livros de análise partem deste axioma e não se preocupam em construir os
reais a partir dos naturais.

[]s,
Claudio.



On Tue, Nov 15, 2022 at 5:07 PM Pedro José  wrote:

> Obrigado a você e ao Cláudio. Mas não sou criativo para inventar. Mas já
> vi que terei que fazer uma homotetia, para as classes de equivalência para
> representar só como um número e não como um par, creio eu.
>
> Cordialmente,
> PJMS
>
> Em ter., 15 de nov. de 2022 às 16:00, Anderson Torres <
> torres.anderson...@gmail.com> escreveu:
>
>>
>>
>> Em ter, 15 de nov de 2022 14:33, Pedro José 
>> escreveu:
>>
>>> Boa tarde!
>>> Para os |Naturais, temos os postulados de Peano.
>>>
>>> Para os Inteiros há alguma formalização?
>>>
>>
>> invente uma!
>>
>> Pode ser por exemplo o conjunto de pares (p,q) tais que p-q é constante.
>>
>> ou melhor (p1,q1)=(p2,q2) se e só se p1+q2=p2+q1.
>>
>>
>>> Acho pobre dizer que é necessário ter outros números devido ao problema
>>> de fechamento nos naturais para a subtração que é fato e daí introduzir os
>>> simétricos que são inteiros e ainda não foram caracterizados.
>>>
>>> No meu antigo ginásio aprendi que os Reais era a união dos conjuntos
>>> disjuntos irracionais e racionais. Os racionais haviam sido bem definidos.
>>> Aí questionei e o que são irracionais? resposta: são os Reais que não são
>>> racionais, os que não podem ser escritos na forma p/q p e q inteiros e
>>> q<>0. Mas me deram um tombo. Definiram os |Reais com base nos irracionais e
>>> os irracionais com base nos |Reais. 3 +2i também não pode ser inscrito na
>>> forma p/q. Só mais tarde no científico, é que meu professor definiu
>>> irracional como um número que não podia ser escrito na forma p/q e cuja
>>> representação decimal tinha uma infinidade de algarismos, sem haver uma
>>> periodicidade.
>>> Na época foi o maior nó que tive com a matemática. O mestre demonstrou
>>> que os racionais eram densos, mas entre eles ainda cabiam os irracionais.
>>> Não satisfeito mostrou que os racionais eram enumeráveis e por absurdo
>>> mostrou que os |Reais não. Não satisfeito mostrou que a cardinalidade do
>>> intervalo [0,1] era maior que a dos |Naturais. Não conseguia conceber que
>>> havia um infinito maior que outro. Outra coisa que demorei a aceitar,mesmo
>>> vendo a bijeção, era que os inteiros e naturais tinham a mesma
>>> cardinalidade. Na minha cabeça, os inteiros têm todos os naturais ainda
>>> sobram os negativos, como é igual?
>>> Hoje, depois de velho, arrumei uma enteada, que muito me pergunta e
>>> estou enrolado. Para dar um ar de superioridade, questionei se conhecia os
>>> inteiros de Gaus, que 5 não era primo nos inteiros de Gaus. Estrepei-me, a
>>> danada foi pesquisar e me questiona sobre o que não tenho um domínio pleno.
>>> Em suma, como apresentei a ela os postulados de Peano para a
>>> caracterização dos Naturais, ela me cobra por algo semelhante para os
>>> Inteiros, e não sei responder.
>>> HELP! SOCORRO! AU SECOURS! AYUDA! AIUTO! HILFE!
>>> Cordialmente,
>>> PJMS
>>>
>>> --
>>> Esta mensagem foi verificada pelo sistema de antivírus e
>>> acredita-se estar livre de perigo.
>>
>>
>> --
>> Esta mensagem foi verificada pelo sistema de antivírus e
>> acredita-se estar livre de perigo.
>
>
> --
> Esta mensagem foi verificada pelo sistema de antivírus e
> acredita-se estar livre de perigo.

-- 
Esta mensagem foi verificada pelo sistema de antiv

[obm-l] Re: [obm-l] Re: [obm-l] Re: [obm-l] Caracterização de Inteiros

2022-11-15 Por tôpico Anderson Torres
Em ter, 15 de nov de 2022 17:07, Pedro José  escreveu:

> Obrigado a você e ao Cláudio. Mas não sou criativo para inventar. Mas já
> vi que terei que fazer uma homotetia, para as classes de equivalência para
> representar só como um número e não como um par, creio eu.
>

Eu lembro de quando li o Guidorizzi formalizando os reais. Até hoje sinto
que entendo sem compreender, haha!

Por outro lado, números reais (irracionais, no caso) são bem menos
palpáveis que os outros. Dívidas e frações são fáceis de entender, afinal.


> Cordialmente,
> PJMS
>
> Em ter., 15 de nov. de 2022 às 16:00, Anderson Torres <
> torres.anderson...@gmail.com> escreveu:
>
>>
>>
>> Em ter, 15 de nov de 2022 14:33, Pedro José 
>> escreveu:
>>
>>> Boa tarde!
>>> Para os |Naturais, temos os postulados de Peano.
>>>
>>> Para os Inteiros há alguma formalização?
>>>
>>
>> invente uma!
>>
>> Pode ser por exemplo o conjunto de pares (p,q) tais que p-q é constante.
>>
>> ou melhor (p1,q1)=(p2,q2) se e só se p1+q2=p2+q1.
>>
>>
>>> Acho pobre dizer que é necessário ter outros números devido ao problema
>>> de fechamento nos naturais para a subtração que é fato e daí introduzir os
>>> simétricos que são inteiros e ainda não foram caracterizados.
>>>
>>> No meu antigo ginásio aprendi que os Reais era a união dos conjuntos
>>> disjuntos irracionais e racionais. Os racionais haviam sido bem definidos.
>>> Aí questionei e o que são irracionais? resposta: são os Reais que não são
>>> racionais, os que não podem ser escritos na forma p/q p e q inteiros e
>>> q<>0. Mas me deram um tombo. Definiram os |Reais com base nos irracionais e
>>> os irracionais com base nos |Reais. 3 +2i também não pode ser inscrito na
>>> forma p/q. Só mais tarde no científico, é que meu professor definiu
>>> irracional como um número que não podia ser escrito na forma p/q e cuja
>>> representação decimal tinha uma infinidade de algarismos, sem haver uma
>>> periodicidade.
>>> Na época foi o maior nó que tive com a matemática. O mestre demonstrou
>>> que os racionais eram densos, mas entre eles ainda cabiam os irracionais.
>>> Não satisfeito mostrou que os racionais eram enumeráveis e por absurdo
>>> mostrou que os |Reais não. Não satisfeito mostrou que a cardinalidade do
>>> intervalo [0,1] era maior que a dos |Naturais. Não conseguia conceber que
>>> havia um infinito maior que outro. Outra coisa que demorei a aceitar,mesmo
>>> vendo a bijeção, era que os inteiros e naturais tinham a mesma
>>> cardinalidade. Na minha cabeça, os inteiros têm todos os naturais ainda
>>> sobram os negativos, como é igual?
>>> Hoje, depois de velho, arrumei uma enteada, que muito me pergunta e
>>> estou enrolado. Para dar um ar de superioridade, questionei se conhecia os
>>> inteiros de Gaus, que 5 não era primo nos inteiros de Gaus. Estrepei-me, a
>>> danada foi pesquisar e me questiona sobre o que não tenho um domínio pleno.
>>> Em suma, como apresentei a ela os postulados de Peano para a
>>> caracterização dos Naturais, ela me cobra por algo semelhante para os
>>> Inteiros, e não sei responder.
>>> HELP! SOCORRO! AU SECOURS! AYUDA! AIUTO! HILFE!
>>> Cordialmente,
>>> PJMS
>>>
>>> --
>>> Esta mensagem foi verificada pelo sistema de antivírus e
>>> acredita-se estar livre de perigo.
>>
>>
>> --
>> Esta mensagem foi verificada pelo sistema de antivírus e
>> acredita-se estar livre de perigo.
>
>
> --
> Esta mensagem foi verificada pelo sistema de antivírus e
> acredita-se estar livre de perigo.

-- 
Esta mensagem foi verificada pelo sistema de antiv�rus e
 acredita-se estar livre de perigo.



[obm-l] Re: [obm-l] Re: [obm-l] Caracterização de Inteiros

2022-11-15 Por tôpico Pedro José
Obrigado a você e ao Cláudio. Mas não sou criativo para inventar. Mas já vi
que terei que fazer uma homotetia, para as classes de equivalência para
representar só como um número e não como um par, creio eu.

Cordialmente,
PJMS

Em ter., 15 de nov. de 2022 às 16:00, Anderson Torres <
torres.anderson...@gmail.com> escreveu:

>
>
> Em ter, 15 de nov de 2022 14:33, Pedro José 
> escreveu:
>
>> Boa tarde!
>> Para os |Naturais, temos os postulados de Peano.
>>
>> Para os Inteiros há alguma formalização?
>>
>
> invente uma!
>
> Pode ser por exemplo o conjunto de pares (p,q) tais que p-q é constante.
>
> ou melhor (p1,q1)=(p2,q2) se e só se p1+q2=p2+q1.
>
>
>> Acho pobre dizer que é necessário ter outros números devido ao problema
>> de fechamento nos naturais para a subtração que é fato e daí introduzir os
>> simétricos que são inteiros e ainda não foram caracterizados.
>>
>> No meu antigo ginásio aprendi que os Reais era a união dos conjuntos
>> disjuntos irracionais e racionais. Os racionais haviam sido bem definidos.
>> Aí questionei e o que são irracionais? resposta: são os Reais que não são
>> racionais, os que não podem ser escritos na forma p/q p e q inteiros e
>> q<>0. Mas me deram um tombo. Definiram os |Reais com base nos irracionais e
>> os irracionais com base nos |Reais. 3 +2i também não pode ser inscrito na
>> forma p/q. Só mais tarde no científico, é que meu professor definiu
>> irracional como um número que não podia ser escrito na forma p/q e cuja
>> representação decimal tinha uma infinidade de algarismos, sem haver uma
>> periodicidade.
>> Na época foi o maior nó que tive com a matemática. O mestre demonstrou
>> que os racionais eram densos, mas entre eles ainda cabiam os irracionais.
>> Não satisfeito mostrou que os racionais eram enumeráveis e por absurdo
>> mostrou que os |Reais não. Não satisfeito mostrou que a cardinalidade do
>> intervalo [0,1] era maior que a dos |Naturais. Não conseguia conceber que
>> havia um infinito maior que outro. Outra coisa que demorei a aceitar,mesmo
>> vendo a bijeção, era que os inteiros e naturais tinham a mesma
>> cardinalidade. Na minha cabeça, os inteiros têm todos os naturais ainda
>> sobram os negativos, como é igual?
>> Hoje, depois de velho, arrumei uma enteada, que muito me pergunta e estou
>> enrolado. Para dar um ar de superioridade, questionei se conhecia os
>> inteiros de Gaus, que 5 não era primo nos inteiros de Gaus. Estrepei-me, a
>> danada foi pesquisar e me questiona sobre o que não tenho um domínio pleno.
>> Em suma, como apresentei a ela os postulados de Peano para a
>> caracterização dos Naturais, ela me cobra por algo semelhante para os
>> Inteiros, e não sei responder.
>> HELP! SOCORRO! AU SECOURS! AYUDA! AIUTO! HILFE!
>> Cordialmente,
>> PJMS
>>
>> --
>> Esta mensagem foi verificada pelo sistema de antivírus e
>> acredita-se estar livre de perigo.
>
>
> --
> Esta mensagem foi verificada pelo sistema de antivírus e
> acredita-se estar livre de perigo.

-- 
Esta mensagem foi verificada pelo sistema de antiv�rus e
 acredita-se estar livre de perigo.



[obm-l] Re: [obm-l] Caracterização de Inteiros

2022-11-15 Por tôpico Anderson Torres
Em ter, 15 de nov de 2022 14:33, Pedro José  escreveu:

> Boa tarde!
> Para os |Naturais, temos os postulados de Peano.
>
> Para os Inteiros há alguma formalização?
>

invente uma!

Pode ser por exemplo o conjunto de pares (p,q) tais que p-q é constante.

ou melhor (p1,q1)=(p2,q2) se e só se p1+q2=p2+q1.


> Acho pobre dizer que é necessário ter outros números devido ao problema de
> fechamento nos naturais para a subtração que é fato e daí introduzir os
> simétricos que são inteiros e ainda não foram caracterizados.
>
> No meu antigo ginásio aprendi que os Reais era a união dos conjuntos
> disjuntos irracionais e racionais. Os racionais haviam sido bem definidos.
> Aí questionei e o que são irracionais? resposta: são os Reais que não são
> racionais, os que não podem ser escritos na forma p/q p e q inteiros e
> q<>0. Mas me deram um tombo. Definiram os |Reais com base nos irracionais e
> os irracionais com base nos |Reais. 3 +2i também não pode ser inscrito na
> forma p/q. Só mais tarde no científico, é que meu professor definiu
> irracional como um número que não podia ser escrito na forma p/q e cuja
> representação decimal tinha uma infinidade de algarismos, sem haver uma
> periodicidade.
> Na época foi o maior nó que tive com a matemática. O mestre demonstrou que
> os racionais eram densos, mas entre eles ainda cabiam os irracionais. Não
> satisfeito mostrou que os racionais eram enumeráveis e por absurdo mostrou
> que os |Reais não. Não satisfeito mostrou que a cardinalidade do intervalo
> [0,1] era maior que a dos |Naturais. Não conseguia conceber que havia um
> infinito maior que outro. Outra coisa que demorei a aceitar,mesmo vendo a
> bijeção, era que os inteiros e naturais tinham a mesma cardinalidade. Na
> minha cabeça, os inteiros têm todos os naturais ainda sobram os negativos,
> como é igual?
> Hoje, depois de velho, arrumei uma enteada, que muito me pergunta e estou
> enrolado. Para dar um ar de superioridade, questionei se conhecia os
> inteiros de Gaus, que 5 não era primo nos inteiros de Gaus. Estrepei-me, a
> danada foi pesquisar e me questiona sobre o que não tenho um domínio pleno.
> Em suma, como apresentei a ela os postulados de Peano para a
> caracterização dos Naturais, ela me cobra por algo semelhante para os
> Inteiros, e não sei responder.
> HELP! SOCORRO! AU SECOURS! AYUDA! AIUTO! HILFE!
> Cordialmente,
> PJMS
>
> --
> Esta mensagem foi verificada pelo sistema de antivírus e
> acredita-se estar livre de perigo.

-- 
Esta mensagem foi verificada pelo sistema de antiv�rus e
 acredita-se estar livre de perigo.



[obm-l] Re: [obm-l] Sugestões para Iniciação Criança

2022-09-14 Por tôpico Tiago Machado
Não sei como andam as atualizações, mas talvez a revista Eureka seja um dos
bons lugares para começar
https://www.obm.org.br/revista-eureka/

On Wed, Sep 14, 2022 at 10:49 AM Esaú Gomes  wrote:

> Olá pessoal, tenho um filho de 10 anos (quinto ano) e ele curte
> matemática. Gostaria de iniciá-lo em material referente às olimpíadas para
> ver se ele pega gosto. Alguém indica material para fazer sua iniciação?
>
> Desde já, obrigado.
>
> --
> Esta mensagem foi verificada pelo sistema de antivírus e
> acredita-se estar livre de perigo.

-- 
Esta mensagem foi verificada pelo sistema de antiv�rus e
 acredita-se estar livre de perigo.



[obm-l] Re: [obm-l] Perguntas mínimas no Tabuleiro

2022-09-14 Por tôpico Anderson Torres
Em ter, 13 de set de 2022 22:59, Jeferson Almir 
escreveu:

> Os números de 1 a 49 são arbitrariamente dispostos num tabuleiro quadrado
> 7x7 . Podemos escolher qualquer quadrado composto de múltiplas células e
> perguntar quais números estão contidos nele. Ao menos quantas perguntas são
> necessárias para determinarmos a configuração exata dos números?
>
> Alguém tem uma ideia ótima ??
>

tenho não, mas para o caso 3x3 a resposta é no máximo 4. Perguntando quem
são os elementos dos quadradinhos 2x2, é possível obter quem são os 9 caras
usando uniões e intersecções.

com isso já podemos recursivamente cortar o quadrado 7x7 em quadrados 3x3.
Se fosse 9x9, isso daria no máximo 24.

vou pensar mais em casa.


>
> --
> Esta mensagem foi verificada pelo sistema de antivírus e
> acredita-se estar livre de perigo.

-- 
Esta mensagem foi verificada pelo sistema de antiv�rus e
 acredita-se estar livre de perigo.



[obm-l] Re: [obm-l] Séries

2022-08-25 Por tôpico Esdras Muniz
Me manda.

Em qui, 25 de ago de 2022 17:36, Israel Meireles Chrisostomo <
israelmchrisost...@gmail.com> escreveu:

> Olá pessoal, recentemente eu tive umas ideias sobre séries envolvendo o
> número e (napier), o seno e o cosseno.Alguém por favor poderia me
> corrigir?São ideias originais e séries infinitas nunca antes pensadas.
> Alguém por favor me ajuda a corrigir.Ver se estou viajandoMeu desejo é
> que vcs digam que esteja certo, sejam pacientes por favor.Quem se dispor,
> por favor, chama inbox
>
> -
> Somente a Deus Glória.
> Israel Meireles Chrisostomo
>
> --
> Esta mensagem foi verificada pelo sistema de antivírus e
> acredita-se estar livre de perigo.

-- 
Esta mensagem foi verificada pelo sistema de antiv�rus e
 acredita-se estar livre de perigo.



[obm-l] Re: [obm-l] Re: [obm-l] Fwd: Módulo

2022-08-14 Por tôpico Rubens Vilhena Fonseca
Uma coisa que você deve definir é a paridade de n. Vamos reescrever em
linguagem de congruências :
2^n==1 (mod 3). Sabendo que 2== -1 (mod 3), então  (-1)^n == 1 (mod 3). O
que só será verdade se n for par.
Então, para n = 2k, temos 4^k = 3x +1.  Por experimentação, você pode
concluir alguns pares (k, x) de solução, (0, 0);
(1, 1); (2, 5); (3, 21)...Então, seu trabalho é mostrar que o par (k,
(4^k-1)/3 ) é uma solução.

Em qui., 11 de ago. de 2022 às 17:38, Anderson Torres <
torres.anderson...@gmail.com> escreveu:

>
>
> Em qui, 11 de ago de 2022 16:12, Esaú Gomes 
> escreveu:
>
>> Alguém poderia me falar o que estudar mais especificamente na questão
>> abaixo?
>>
>> Para quais valores naturais  de *n* e *x*, existe solução
>> 2^n = 3x + 1.
>>
>
>   Provas antigas.
>
> Esses problemas são resolvidos geralmente apelando para fatos padrão de
> congruências, em especial potenciação, ordem etc.
>
> E, no geral, a melhor maneira de entender e aplicar estes fatos é mediante
> treino, treino e mais treino.
>
> --
>> Esta mensagem foi verificada pelo sistema de antivírus e
>> acredita-se estar livre de perigo.
>
>
> --
> Esta mensagem foi verificada pelo sistema de antivírus e
> acredita-se estar livre de perigo.

-- 
Esta mensagem foi verificada pelo sistema de antiv�rus e
 acredita-se estar livre de perigo.



[obm-l] Re: [obm-l] Fwd: Módulo

2022-08-11 Por tôpico Anderson Torres
Em qui, 11 de ago de 2022 16:12, Esaú Gomes  escreveu:

> Alguém poderia me falar o que estudar mais especificamente na questão
> abaixo?
>
> Para quais valores naturais  de *n* e *x*, existe solução
> 2^n = 3x + 1.
>

  Provas antigas.

Esses problemas são resolvidos geralmente apelando para fatos padrão de
congruências, em especial potenciação, ordem etc.

E, no geral, a melhor maneira de entender e aplicar estes fatos é mediante
treino, treino e mais treino.

-- 
> Esta mensagem foi verificada pelo sistema de antivírus e
> acredita-se estar livre de perigo.

-- 
Esta mensagem foi verificada pelo sistema de antiv�rus e
 acredita-se estar livre de perigo.



[obm-l] Re: [obm-l] Fwd: Módulo

2022-08-11 Por tôpico Carlos Gomes
Ola amigo. Normalmente essas equações diofantinas nao lineares tem solução
passando por congruência.



Em qui., 11 de ago. de 2022 16:11, Esaú Gomes 
escreveu:

> Alguém poderia me falar o que estudar mais especificamente na questão
> abaixo?
>
> Para quais valores naturais  de *n* e *x*, existe solução
> 2^n = 3x + 1.
>
> --
> Esta mensagem foi verificada pelo sistema de antivírus e
> acredita-se estar livre de perigo.

-- 
Esta mensagem foi verificada pelo sistema de antiv�rus e
 acredita-se estar livre de perigo.



[obm-l] Re: [obm-l] Provar que a inteira f é um polinômio de grau positivo

2022-07-15 Por tôpico Artur Costa Steiner
Obrigado, abordagem bem interessante

Eu dei a seguinte prova:

Para z em C/{0}, seja g(z) = f(1/z), obtendo-se uma função holomorfa tal
que lim z —> 0 g(z) = lim z—> oo f(z) = oo. Assim, g é meromorfa em C,
tendo em 0 seu único polo. Sendo n > 0 a ordem deste polo, g é expandida em
C/{0} por uma série de Laurent em torno de 0, havendo portanto complexos
c(-n), … c(0), c(1) ….tais que

g(z) = Soma (k = -n, oo) c(k) z^k, z em C/{0}

Para z em  C/{0} temos então que

f(z) = g(1/z) =Soma (k = n, -oo c(k) z^(k) (1)

Em (1), temos a série de Laurent que, em C/{0}, expande f e em torno de 0.
Se nesta série houvesse algum coeficiente não nulo associado a potência
negativa de z, f apresentaria uma singularidade em 0. Mas sendo uma função
 inteira, f não apresenta nenhuma singularidade em C, do que deduzimos que,
em (1), todos os coeficientes associados a potências negativas de z são
nulos. Logo, em C/{0} f é o polinômio de grau n dado por

f(z) = c(-n) z^n + ….  c(1) z + c(0) (2)

Como f é contínua, temos que f(0) = lim z —> 0 f(z) = c(0), o que mostra
que (2) vale em todo o C. Logo, f é em C um polinômio de grau positivo.

Abs
Artur

Em qui., 14 de jul. de 2022 às 19:23, Claudio Buffara <
claudio.buff...@gmail.com> escreveu:

> Use o fato de que toda função meromorfica  em C união {inf} é da forma
> f(z)/g(z), onde f, g são polinômios.
> Daí, como a função do enunciado é inteira, g(z) é constante (e não nula).
> E como f(z) rende a inf quando z tende a inf, f é um polinômio não
> constante.
>
> Enviado do meu iPhone
>
> > Em 14 de jul. de 2022, à(s) 16:41, Artur Costa Steiner <
> artur.costa.stei...@gmail.com> escreveu:
> >
> > Oi amigos!
> >
> > Um teorema da Análise Complexa diz que, se f é inteira e lim z —> oo
> f(z) = oo, então f é um polinômio (claramente não constante). Nos
> livros em que estudei isso era dado como exercício, de modo que nunca vi a
> demonstração deste teorema. Eu consegui dar duas demonstrações para
> ele, sendo que uma delas sei que está certa A outra acho que também está
> certa, mas a primeira me parece bem melhor.Â
> >
> > Alguém aqui pode dar uma prova, para comparar com a minha? Se houver
> interesse (Análise Complexa não costuma aparecer aqui) eu dou as minhas.Â
> >
> > Obrigado
> >
> > Artur
> >
> >
> >
> > --
> > Esta mensagem foi verificada pelo sistema de antivírus e
> > acredita-se estar livre de perigo.
>
> --
> Esta mensagem foi verificada pelo sistema de antivírus e
>  acredita-se estar livre de perigo.
>
>
> =
> Instru�ões para entrar na lista, sair da lista e usar a lista em
> http://www.mat.puc-rio.br/~obmlistas/obm-l.html
> =
>

-- 
Esta mensagem foi verificada pelo sistema de antiv�rus e
 acredita-se estar livre de perigo.



[obm-l] Re: [obm-l] Função phi de Euler

2022-07-15 Por tôpico Anderson Torres
Em qui, 14 de jul de 2022 11:52, Rubens Vilhena Fonseca <
rubens.vilhen...@gmail.com> escreveu:

> Saudações a todos da lista.
> É um fato que para primos p ímpares, a função de Euler phi(p)=p-1 é sempre
> um valor par.
> Os primos 7, 13, 19, 31, 37, 67, 73, 79, 97, ... tem valores pares
> múltiplos de 3.
> Existe algum caminho a tomar para determinar quando phi(p) = 3 .(2k)?
>

quem é esse k?

Agradeço qualquer solução ou  informação ou indicação de leituras sobre o
> problema.
> Att
>
>
> --
> Esta mensagem foi verificada pelo sistema de antivírus e
> acredita-se estar livre de perigo.

-- 
Esta mensagem foi verificada pelo sistema de antiv�rus e
 acredita-se estar livre de perigo.



[obm-l] Re: [obm-l] Re: [obm-l] Função phi de Euler

2022-07-15 Por tôpico Anderson Torres
Em qui, 14 de jul de 2022 12:19, Esdras Muniz 
escreveu:

> Quis dizer φ(p)=p-1.
>
> Em qui, 14 de jul de 2022 12:02, Esdras Muniz 
> escreveu:
>
>> Oi(o)=p-1, aí isso só vale se o primo for da firma 6k+1.
>>
>
phi(4+3)=7-1


>> Em qui, 14 de jul de 2022 11:52, Rubens Vilhena Fonseca <
>> rubens.vilhen...@gmail.com> escreveu:
>>
>>> Saudações a todos da lista.
>>> É um fato que para primos p ímpares, a função de Euler phi(p)=p-1 é
>>> sempre um valor par.
>>> Os primos 7, 13, 19, 31, 37, 67, 73, 79, 97, ... tem valores pares
>>> múltiplos de 3.
>>> Existe algum caminho a tomar para determinar quando phi(p) = 3 .(2k)?
>>> Agradeço qualquer solução ou  informação ou indicação de leituras sobre
>>> o  problema.
>>> Att
>>>
>>>
>>> --
>>> Esta mensagem foi verificada pelo sistema de antivírus e
>>> acredita-se estar livre de perigo.
>>
>>
> --
> Esta mensagem foi verificada pelo sistema de antivírus e
> acredita-se estar livre de perigo.

-- 
Esta mensagem foi verificada pelo sistema de antiv�rus e
 acredita-se estar livre de perigo.



[obm-l] Re: [obm-l] Função phi de Euler

2022-07-14 Por tôpico Esdras Muniz
Quis dizer φ(p)=p-1.

Em qui, 14 de jul de 2022 12:02, Esdras Muniz 
escreveu:

> Oi(o)=p-1, aí isso só vale se o primo for da firma 6k+1.
>
> Em qui, 14 de jul de 2022 11:52, Rubens Vilhena Fonseca <
> rubens.vilhen...@gmail.com> escreveu:
>
>> Saudações a todos da lista.
>> É um fato que para primos p ímpares, a função de Euler phi(p)=p-1 é
>> sempre um valor par.
>> Os primos 7, 13, 19, 31, 37, 67, 73, 79, 97, ... tem valores pares
>> múltiplos de 3.
>> Existe algum caminho a tomar para determinar quando phi(p) = 3 .(2k)?
>> Agradeço qualquer solução ou  informação ou indicação de leituras sobre
>> o  problema.
>> Att
>>
>>
>> --
>> Esta mensagem foi verificada pelo sistema de antivírus e
>> acredita-se estar livre de perigo.
>
>

-- 
Esta mensagem foi verificada pelo sistema de antiv�rus e
 acredita-se estar livre de perigo.



[obm-l] Re: [obm-l] Função phi de Euler

2022-07-14 Por tôpico Esdras Muniz
Oi(o)=p-1, aí isso só vale se o primo for da firma 6k+1.

Em qui, 14 de jul de 2022 11:52, Rubens Vilhena Fonseca <
rubens.vilhen...@gmail.com> escreveu:

> Saudações a todos da lista.
> É um fato que para primos p ímpares, a função de Euler phi(p)=p-1 é sempre
> um valor par.
> Os primos 7, 13, 19, 31, 37, 67, 73, 79, 97, ... tem valores pares
> múltiplos de 3.
> Existe algum caminho a tomar para determinar quando phi(p) = 3 .(2k)?
> Agradeço qualquer solução ou  informação ou indicação de leituras sobre o
> problema.
> Att
>
>
> --
> Esta mensagem foi verificada pelo sistema de antivírus e
> acredita-se estar livre de perigo.

-- 
Esta mensagem foi verificada pelo sistema de antiv�rus e
 acredita-se estar livre de perigo.



[obm-l] Re: [obm-l] Questão de probabilidade

2022-06-29 Por tôpico Rogerio Ponce
Ola' Vanderlei e pessoal da lista!
Pediram-me para resolver o problema por inteiro.
Ok, vamos la'!


Em um pet shop ha' 3 gatos e 5 caes. Sabemos que 3 desses animais sao
pretos, 4 sao brancos e 1 e' malhado. Alem disso, pelo menos 1
cachorro e' preto. Assinale o que for correto.
01) A probabilidade de haver exatamente 1 cachorro preto e' de 1/6.
02) A probabilidade de haver pelo menos 1 gato branco e pelo menos 2
cachorros brancos e' de 2/3.
04) A probabilidade de haver um cachorro malhado e' maior do que a
probabilidade de haver um gato malhado.
08) Se um animal for escolhido ao acaso, a probabilidade de ele ser um
cachorro preto e' de 1/8.
16) Se um animal for escolhido ao acaso, a probabilidade de ele ser um
gato malhado e' de 1/16.



Afirmacao 01) A probabilidade de haver exatamente 1 cachorro preto e' de 1/6.

-

O cachorro que ja' e' preto, deixaremos de lado, de modo que temos
3Gatos,4Caes para pintar com 2Preto,4Branco,1Malhado.
Bem, dizer que existe exatamente 1 cachorro preto, significa dizer que
as outras 2 pinturas pretas foram para os gatos, ou que existem
exatamente 2 gatos pretos.

Entao, vamos comecar a pintar com a tinta preta, e verificar de
quantas formas podemos fazer isso.

Repare que para ponderar corretamente as ocorrencias que interessam,
eu considero todas as permutacoes possiveis, ou das tintas, ou dos
animais. Assim, posso, por exemplo, fixar a ordem das tintas, e
aplicar isso em todas as permutacoes possiveis entre os animais.
A razao entre os casos que interessam e os casos possiveis e'
justamente a probabilidade de ocorrencia de um caso que interessa.

Como sao 2 pinturas pretas, temos um total de 7 escolhas de animal
para a primeira pintura, e 6 escolhas para a segunda, com um total de
42 escolhas para a primeira e segunda pinturas pretas.

Deste total, temos 3 gatos para a primeira escolha e 2 gatos para a
segunda, com um total de 6 escolhas para termos 2 gatos pretos.
Deste total, vamos escolher 1 entre 3 gatos para a primeira pintura, e
1 entre 2 gatos para a segunda pintura, com um total de 6 escolhas
para pintarmos 2 gatos de preto.

Assim, a probabilidade de termos exatamente 2 gatos pretos vale 6/42 = 1/7
Ou seja, a probabilidade de termos exatamente um cachorro preto vale
1/7, de modo que a afirmacao 01 esta' errada.

OBS: note que, entre os gatos g1, g2 e g3 (por exemplo), mesmo que
pintemos g1 e g2 de preto, a ordem e' importante, pois ordens
diferentes correspondem a permutacoes diferentes. Pintar de preto o
gato g1, e depois o gato g2, e' uma permutacao diferente daquela em
que pintamos de preto primeiramente o gato g2, e depois o gato g1,
embora o resultado final seja obtermos g1 e g2 pretos.



Afirmacao 02) A probabilidade de haver pelo menos 1 gato branco e pelo
menos 2 cachorros brancos e' de 2/3.

-

O cachorro que ja' e' preto pode ser deixado de lado.

Seguindo agora com as 4 pinturas brancas, temos um total de 7 escolhas
para o primeiro animal, 6 para o segundo, 5 para o terceiro, e 4 para
o quarto, de modo que existem 7*6*5*4 escolhas diferentes para as
pinturas brancas.

Entre os animais que receberao as pinturas brancas, existem os seguintes casos:

1o caso: 4 caes brancos
O numero de combinacoes de 4 caes entre 4 caes e' C(4,4)=1.
Assim, considerando-se as permutacoes entre estes 4 animais
brancos, obtemos um total de permutacoes diferentes igual a
C(4,4)*4! = 1*4! = 24

2o caso: 3 caes e 1 gato brancos
Como sao 3 caes de um total de 4 caes possiveis, e 1 gato de
um total de 3 gatos possiveis, o numero total de permutacoes vale
C(4,3) * C(3,1) * 4! = 4*3*4! = 288

3o caso: 2 caes e 2 gatos brancos
o numero total de permutacoes vale
C(4,2) * C(3,2) * 4! = 6*3*4! = 432

4o caso: 1 cao e 3 gatos brancos
o numero total de permutacoes vale
C(4,1) * C(3,3) * 4! = 4*1*4! = 96


Contabilizando-se o segundo e o terceiro casos (que sao os que nos
interessam), temos um total de (288+432)=720 permutacoes, de um
universo de 840 permutacoes, de modo que a probabilidade pedida vale
720/840 = 6/7

Portanto, a afirmacao 02 esta' errada.



Afirmacao 04) A probabilidade de haver um cachorro malhado e' maior do
que a probabilidade de haver um gato malhado.

-

Deixando de lado o cachorro que ja' e' preto, e seguindo agora com a
pintura malhada, temos 7 escolhas de animais, das quais 4 sao de
cachorros, e 3 sao de gatos.

Assim, a probabilidade de haver um cachorro malhado e' 4/7 , e a
probabilidade de haver um gato malhado e' 3/7.

Portanto, a afirmacao 04 esta' correta.



Afirmacao 08) Se um animal for escolhido ao acaso, a probabilidade de
ele ser um cachorro preto e' de 1/8.

-

Se houvesse apenas um cachorro preto - aquele que ja' seria mesmo
preto, por definicao - a probabilidade de escolhermos um cacho

[obm-l] Re: [obm-l] Re: [obm-l] Re: [obm-l] Re: [obm-l] Re: [obm-l] Questão de probabilidade

2022-06-24 Por tôpico Rogerio Ponce
Otima explicacao!
Obrigado, Ralph!

PS: e sim, a provocacao foi pra voce mesmo!
:)

[]'s
Rogerio Ponce


On Wed, Jun 22, 2022 at 1:00 PM Ralph Costa Teixeira  wrote:
>
> Ponce está provocando a gente... senti que esta flecha tinha um bocado a 
> minha direção...  :D :D :D
>
> Olha, tem duas "visões" sobre o que "probabilidade" significa.
>
> A primeira vai na linha de que só podemos falar de probabilidade sobre coisas 
> que ainda não aconteceram. Vai nessa linha: se os eventos estão no passado, 
> então já aconteceram, já estão definidos, e não faz sentido dizer que tinha x 
> de chance de ser assim ou y de ser assado. Se você já jogou a moeda justa, 
> não é mais 50/50 --  é 100% de ser cara, ou 100% de ser coroa, dependendo do 
> que ocorreu. Quem pensa assim vai dizer que dado um certo evento (sempre no 
> futuro), ele tem uma probabilidade dada; se duas pessoas diferentes derem 
> duas probabilidades diferentes para o mesmo evento, uma delas errou.
>
> Outra linha diz que podemos falar de probabilidade sempre que houver 
> incerteza; não interessa o que aconteceu ou o que vai acontecer, o que 
> interessa é o que você SABE sobre o acontecimento. Se você jogou a moeda 
> justa mas eu não sei nada mais sobre o lançamento, continua sendo 50/50 
> **PARA MIM**. Probabilidade passa a ser um conceito sobre INFORMAÇÃO, não 
> sobre os fatos em si (a probabilidade não está na moeda, está no que você 
> sabe sobre a moeda). Quem pensa assim vai dizer que a probabilidade do evento 
> depende não apenas do evento em si, mas da informação que se tem em mãos. 
> Quem pensa assim admite que duas pessoas diferentes podem dar probabilidades 
> diferentes ao mesmo evento SE SOUBEREM FATOS DIFERENTES a respeito do evento, 
> ou seja, probabilidade passa a ser bastante "subjetivo".
>
> Eu talvez tenha descrito mal a primeira interpretação, pois sou ferrenho 
> defensor da segunda. Ela simplesmente engloba a primeira, porque você pode 
> ter informação parcial sobre fatos que ocorrem no futuro. E falar de 
> probabilidade para descrever incerteza presente ou passada é MUITO útil! Eu 
> quero poder expressar incerteza sobre fatos passados com frases do tipo 
> "fulano tem x% de probabilidade de ter cometido tal crime", ou "tem y% de 
> probabilidade de ter petróleo nesse poço", ou "tem z% de chance de eu ter 
> COVID"... Se você tem uma reação negativa a essas frases, lembre o que elas 
> realmente significam (na segunda interpretação): claro que ou o cara cometeu 
> o crime ou não, não faz sentido dizer que ele cometeu o crime x% das vezes em 
> que fizermos um experimento de ele cometer o crime... mas o que aquilo 
> significa é "com a informação que eu tenho, numa escala de 0 a 1, eu tenho 
> x/100 de certeza que fulano cometeu o crime". E "certeza baseada em informaç�!
 �es" é sim quantificável -- e satisfaz exatamente as leis das probabilidades 
com as quais concordamos. "Subjetivo" não significa "posso falar qualquer 
coisa", significa apenas que a conta pode variar de pessoa para pessoa... mas, 
de novo, SE ESSAS PESSOAS TIVEREM INFORMAÇÕES DIFERENTES sobre o evento.
>
> Abraço, Ralph.
>
> On Wed, Jun 22, 2022 at 12:09 PM Rogerio Ponce  wrote:
>>
>> Olá Pedro e pessoal da lista!
>>
>> Segundo a opinião do Pedro, nao faz sentido perguntar qual a probabilidade 
>> de Jose ter conseguido um 6 ao jogar o dado ontem, pois isso ja' aconteceu, 
>> e, portanto, ja' esta' definido.
>>
>> Sera' que e' isso mesmo?
>>
>> []'s
>> Rogerio Ponce
>>
>>
>> On Mon, Jun 20, 2022 at 9:45 PM Pedro José  wrote:
>>>
>>> Eu na minha humilde opinião creio que a probabilidade exista quando pode 
>>> ser uma coisa ou outra. No caso já é definido o que os animais são. Então 
>>> já está tudo errado. A questão seria viável se dessem esses limitantes para 
>>> uma criança que pintaria os desenhos dos animais. Aí sim há probabilidade.
>>>
>>> Em sáb., 18 de jun. de 2022 03:33, Rogerio Ponce  
>>> escreveu:

 Ola' Vanderlei e pessoal da lista!

 Sem perda de generalidade, podemos imaginar que vamos fazer o seguinte:

 - uma pintura preta em um dos caes, escolhido aleatoriamente

 - uma pintura "malhada" em um dos animais, escolhido aleatoriamente entre 
 os 7 animais nao pintados

 - duas pintura pretas, em dois animais, escolhidos aleatoriamente entre os 
 6 animais restantes,

 - quatro pinturas brancas nos 4 animais restantes


 Analisando a afirmacao 04, por exemplo, verificamos que, no segundo passo 
 (pintura malhada) existem 4 opcoes de cachorro e 3 opcoes de gato.

 Assim, a probabilidade de haver um cachorro malhado (4/7) e' maior que a 
 probabilidade de haver um gato malhado (3/7).
 Portanto, a afirmacao 04 esta' correta.
 (e o gabarito esta' errado).

 []'s
 Rogerio Ponce



 On Wed, Mar 16, 2022 at 8:08 AM Professor Vanderlei Nemitz 
  wrote:
>
> Bom dia!
> Na questão a seguir, do vestibular da UEM, 

[obm-l] Re: [obm-l] Re: [obm-l] Re: [obm-l] Re: [obm-l] Questão de probabilidade

2022-06-22 Por tôpico Ralph Costa Teixeira
Ponce está provocando a gente... senti que esta flecha tinha um bocado a
minha direção...  :D :D :D

Olha, tem duas "visões" sobre o que "probabilidade" significa.

A primeira vai na linha de que só podemos falar de probabilidade sobre
coisas que ainda não aconteceram. Vai nessa linha: se os eventos estão no
passado, então já aconteceram, já estão definidos, e não faz sentido dizer
que tinha x de chance de ser assim ou y de ser assado. Se você já jogou a
moeda justa, não é mais 50/50 --  é 100% de ser cara, ou 100% de ser coroa,
dependendo do que ocorreu. Quem pensa assim vai dizer que dado um certo
evento (sempre no futuro), ele tem uma probabilidade dada; se duas pessoas
diferentes derem duas probabilidades diferentes para o mesmo evento, uma
delas errou.

Outra linha diz que podemos falar de probabilidade sempre que houver
incerteza; não interessa o que aconteceu ou o que vai acontecer, o que
interessa é o que você SABE sobre o acontecimento. Se você jogou a moeda
justa mas eu não sei nada mais sobre o lançamento, continua sendo 50/50
**PARA MIM**. Probabilidade passa a ser um conceito sobre INFORMAÇÃO, não
sobre os fatos em si (a probabilidade não está na moeda, está no que você
sabe sobre a moeda). Quem pensa assim vai dizer que a probabilidade do
evento depende não apenas do evento em si, mas da informação que se tem em
mãos. Quem pensa assim admite que duas pessoas diferentes podem dar
probabilidades diferentes ao mesmo evento SE SOUBEREM FATOS DIFERENTES a
respeito do evento, ou seja, probabilidade passa a ser bastante "subjetivo".

Eu talvez tenha descrito mal a primeira interpretação, pois sou ferrenho
defensor da segunda. Ela simplesmente engloba a primeira, porque você pode
ter informação parcial sobre fatos que ocorrem no futuro. E falar de
probabilidade para descrever incerteza presente ou passada é MUITO útil! Eu
quero poder expressar incerteza sobre fatos passados com frases do tipo
"fulano tem x% de probabilidade de ter cometido tal crime", ou "tem y% de
probabilidade de ter petróleo nesse poço", ou "tem z% de chance de eu ter
COVID"... Se você tem uma reação negativa a essas frases, lembre o que elas
realmente significam (na segunda interpretação): claro que ou o cara
cometeu o crime ou não, não faz sentido dizer que ele cometeu o crime x%
das vezes em que fizermos um experimento de ele cometer o crime... mas o
que aquilo significa é "com a informação que eu tenho, numa escala de 0 a
1, eu tenho x/100 de certeza que fulano cometeu o crime". E "certeza
baseada em informações" é sim quantificável -- e satisfaz exatamente as
leis das probabilidades com as quais concordamos. "Subjetivo" não significa
"posso falar qualquer coisa", significa apenas que a conta pode variar de
pessoa para pessoa... mas, de novo, SE ESSAS PESSOAS TIVEREM INFORMAÇÕES
DIFERENTES sobre o evento.

Abraço, Ralph.

On Wed, Jun 22, 2022 at 12:09 PM Rogerio Ponce  wrote:

> Olá Pedro e pessoal da lista!
>
> Segundo a opinião do Pedro, nao faz sentido perguntar qual a probabilidade
> de Jose ter conseguido um 6 ao jogar o dado ontem, pois isso ja' aconteceu,
> e, portanto, ja' esta' definido.
>
> Sera' que e' isso mesmo?
>
> []'s
> Rogerio Ponce
>
>
> On Mon, Jun 20, 2022 at 9:45 PM Pedro José  wrote:
>
>> Eu na minha humilde opinião creio que a probabilidade exista quando pode
>> ser uma coisa ou outra. No caso já é definido o que os animais são. Então
>> já está tudo errado. A questão seria viável se dessem esses limitantes para
>> uma criança que pintaria os desenhos dos animais. Aí sim há probabilidade.
>>
>> Em sáb., 18 de jun. de 2022 03:33, Rogerio Ponce da Silva <
>> abrlw...@gmail.com> escreveu:
>>
>>> Ola' Vanderlei e pessoal da lista!
>>>
>>> Sem perda de generalidade, podemos imaginar que vamos fazer o seguinte:
>>>
>>> - uma pintura preta em um dos caes, escolhido aleatoriamente
>>>
>>> - uma pintura "malhada" em um dos animais, escolhido aleatoriamente
>>> entre os 7 animais nao pintados
>>>
>>> - duas pintura pretas, em dois animais, escolhidos aleatoriamente entre
>>> os 6 animais restantes,
>>>
>>> - quatro pinturas brancas nos 4 animais restantes
>>>
>>>
>>> Analisando a afirmacao 04, por exemplo, verificamos que, no segundo
>>> passo (pintura malhada) existem 4 opcoes de cachorro e 3 opcoes de gato.
>>>
>>> Assim, a probabilidade de haver um cachorro malhado (4/7) e' maior que a
>>> probabilidade de haver um gato malhado (3/7).
>>> Portanto, a afirmacao 04 esta' correta.
>>> (e o gabarito esta' errado).
>>>
>>> []'s
>>> Rogerio Ponce
>>>
>>>
>>>
>>> On Wed, Mar 16, 2022 at 8:08 AM Professor Vanderlei Nemitz <
>>> vanderma...@gmail.com> wrote:
>>>
 Bom dia!
 Na questão a seguir, do vestibular da UEM, penso que o espaço amostral
 tem 105 elementos, pois um cachorro é preto (desconsideramos esse). Porém,
 com esse pensamento, não consigo obter o gabarito, que diz que 02 e 16 são
 corretas.
 Alguém poderia ajudar?
 Muito obrigado!

 *Em um pet shop há 3 gatos e 5 cães. Sab

[obm-l] Re: [obm-l] Re: [obm-l] Re: [obm-l] Questão de probabilidade

2022-06-22 Por tôpico Rogerio Ponce
Olá Pedro e pessoal da lista!

Segundo a opinião do Pedro, nao faz sentido perguntar qual a probabilidade
de Jose ter conseguido um 6 ao jogar o dado ontem, pois isso ja' aconteceu,
e, portanto, ja' esta' definido.

Sera' que e' isso mesmo?

[]'s
Rogerio Ponce


On Mon, Jun 20, 2022 at 9:45 PM Pedro José  wrote:

> Eu na minha humilde opinião creio que a probabilidade exista quando pode
> ser uma coisa ou outra. No caso já é definido o que os animais são. Então
> já está tudo errado. A questão seria viável se dessem esses limitantes para
> uma criança que pintaria os desenhos dos animais. Aí sim há probabilidade.
>
> Em sáb., 18 de jun. de 2022 03:33, Rogerio Ponce da Silva <
> abrlw...@gmail.com> escreveu:
>
>> Ola' Vanderlei e pessoal da lista!
>>
>> Sem perda de generalidade, podemos imaginar que vamos fazer o seguinte:
>>
>> - uma pintura preta em um dos caes, escolhido aleatoriamente
>>
>> - uma pintura "malhada" em um dos animais, escolhido aleatoriamente entre
>> os 7 animais nao pintados
>>
>> - duas pintura pretas, em dois animais, escolhidos aleatoriamente entre
>> os 6 animais restantes,
>>
>> - quatro pinturas brancas nos 4 animais restantes
>>
>>
>> Analisando a afirmacao 04, por exemplo, verificamos que, no segundo passo
>> (pintura malhada) existem 4 opcoes de cachorro e 3 opcoes de gato.
>>
>> Assim, a probabilidade de haver um cachorro malhado (4/7) e' maior que a
>> probabilidade de haver um gato malhado (3/7).
>> Portanto, a afirmacao 04 esta' correta.
>> (e o gabarito esta' errado).
>>
>> []'s
>> Rogerio Ponce
>>
>>
>>
>> On Wed, Mar 16, 2022 at 8:08 AM Professor Vanderlei Nemitz <
>> vanderma...@gmail.com> wrote:
>>
>>> Bom dia!
>>> Na questão a seguir, do vestibular da UEM, penso que o espaço amostral
>>> tem 105 elementos, pois um cachorro é preto (desconsideramos esse). Porém,
>>> com esse pensamento, não consigo obter o gabarito, que diz que 02 e 16 são
>>> corretas.
>>> Alguém poderia ajudar?
>>> Muito obrigado!
>>>
>>> *Em um pet shop há 3 gatos e 5 cães. Sabemos que 3 desses animais são
>>> pretos, 4 são brancos e 1 é malhado. Além disso, pelo menos 1 cachorro é
>>> preto. Assinale o que for correto. *
>>> *01) A probabilidade de haver exatamente 1 cachorro preto é de 1/6. *
>>> *02) A probabilidade de haver pelo menos 1 gato branco e pelo menos 2
>>> cachorros brancos é de 2/3.*
>>> *04) A probabilidade de haver um cachorro malhado é maior do que a
>>> probabilidade de haver um gato malhado. *
>>> *08) Se um animal for escolhido ao acaso, a probabilidade de ele ser um
>>> cachorro preto é de 1/8. *
>>> *16) Se um animal for escolhido ao acaso, a probabilidade de ele ser um
>>> gato malhado é de 1/16.   *
>>>
>>> --
>>> Esta mensagem foi verificada pelo sistema de antivírus e
>>> acredita-se estar livre de perigo.
>>
>>
>> --
>> Esta mensagem foi verificada pelo sistema de antivírus e
>> acredita-se estar livre de perigo.
>
>
> --
> Esta mensagem foi verificada pelo sistema de antivírus e
> acredita-se estar livre de perigo.

-- 
Esta mensagem foi verificada pelo sistema de antiv�rus e
 acredita-se estar livre de perigo.



[obm-l] Re: [obm-l] Re: [obm-l] Questão de probabilidade

2022-06-20 Por tôpico Pedro José
Eu na minha humilde opinião creio que a probabilidade exista quando pode
ser uma coisa ou outra. No caso já é definido o que os animais são. Então
já está tudo errado. A questão seria viável se dessem esses limitantes para
uma criança que pintaria os desenhos dos animais. Aí sim há probabilidade.

Em sáb., 18 de jun. de 2022 03:33, Rogerio Ponce da Silva <
abrlw...@gmail.com> escreveu:

> Ola' Vanderlei e pessoal da lista!
>
> Sem perda de generalidade, podemos imaginar que vamos fazer o seguinte:
>
> - uma pintura preta em um dos caes, escolhido aleatoriamente
>
> - uma pintura "malhada" em um dos animais, escolhido aleatoriamente entre
> os 7 animais nao pintados
>
> - duas pintura pretas, em dois animais, escolhidos aleatoriamente entre os
> 6 animais restantes,
>
> - quatro pinturas brancas nos 4 animais restantes
>
>
> Analisando a afirmacao 04, por exemplo, verificamos que, no segundo passo
> (pintura malhada) existem 4 opcoes de cachorro e 3 opcoes de gato.
>
> Assim, a probabilidade de haver um cachorro malhado (4/7) e' maior que a
> probabilidade de haver um gato malhado (3/7).
> Portanto, a afirmacao 04 esta' correta.
> (e o gabarito esta' errado).
>
> []'s
> Rogerio Ponce
>
>
>
> On Wed, Mar 16, 2022 at 8:08 AM Professor Vanderlei Nemitz <
> vanderma...@gmail.com> wrote:
>
>> Bom dia!
>> Na questão a seguir, do vestibular da UEM, penso que o espaço amostral
>> tem 105 elementos, pois um cachorro é preto (desconsideramos esse). Porém,
>> com esse pensamento, não consigo obter o gabarito, que diz que 02 e 16 são
>> corretas.
>> Alguém poderia ajudar?
>> Muito obrigado!
>>
>> *Em um pet shop há 3 gatos e 5 cães. Sabemos que 3 desses animais são
>> pretos, 4 são brancos e 1 é malhado. Além disso, pelo menos 1 cachorro é
>> preto. Assinale o que for correto. *
>> *01) A probabilidade de haver exatamente 1 cachorro preto é de 1/6. *
>> *02) A probabilidade de haver pelo menos 1 gato branco e pelo menos 2
>> cachorros brancos é de 2/3.*
>> *04) A probabilidade de haver um cachorro malhado é maior do que a
>> probabilidade de haver um gato malhado. *
>> *08) Se um animal for escolhido ao acaso, a probabilidade de ele ser um
>> cachorro preto é de 1/8. *
>> *16) Se um animal for escolhido ao acaso, a probabilidade de ele ser um
>> gato malhado é de 1/16.   *
>>
>> --
>> Esta mensagem foi verificada pelo sistema de antivírus e
>> acredita-se estar livre de perigo.
>
>
> --
> Esta mensagem foi verificada pelo sistema de antivírus e
> acredita-se estar livre de perigo.

-- 
Esta mensagem foi verificada pelo sistema de antiv�rus e
 acredita-se estar livre de perigo.



[obm-l] Re: [obm-l] Questão de probabilidade

2022-06-17 Por tôpico Rogerio Ponce da Silva
Ola' Vanderlei e pessoal da lista!

Sem perda de generalidade, podemos imaginar que vamos fazer o seguinte:

- uma pintura preta em um dos caes, escolhido aleatoriamente

- uma pintura "malhada" em um dos animais, escolhido aleatoriamente entre
os 7 animais nao pintados

- duas pintura pretas, em dois animais, escolhidos aleatoriamente entre os
6 animais restantes,

- quatro pinturas brancas nos 4 animais restantes


Analisando a afirmacao 04, por exemplo, verificamos que, no segundo passo
(pintura malhada) existem 4 opcoes de cachorro e 3 opcoes de gato.

Assim, a probabilidade de haver um cachorro malhado (4/7) e' maior que a
probabilidade de haver um gato malhado (3/7).
Portanto, a afirmacao 04 esta' correta.
(e o gabarito esta' errado).

[]'s
Rogerio Ponce



On Wed, Mar 16, 2022 at 8:08 AM Professor Vanderlei Nemitz <
vanderma...@gmail.com> wrote:

> Bom dia!
> Na questão a seguir, do vestibular da UEM, penso que o espaço amostral tem
> 105 elementos, pois um cachorro é preto (desconsideramos esse). Porém, com
> esse pensamento, não consigo obter o gabarito, que diz que 02 e 16 são
> corretas.
> Alguém poderia ajudar?
> Muito obrigado!
>
> *Em um pet shop há 3 gatos e 5 cães. Sabemos que 3 desses animais são
> pretos, 4 são brancos e 1 é malhado. Além disso, pelo menos 1 cachorro é
> preto. Assinale o que for correto. *
> *01) A probabilidade de haver exatamente 1 cachorro preto é de 1/6. *
> *02) A probabilidade de haver pelo menos 1 gato branco e pelo menos 2
> cachorros brancos é de 2/3.*
> *04) A probabilidade de haver um cachorro malhado é maior do que a
> probabilidade de haver um gato malhado. *
> *08) Se um animal for escolhido ao acaso, a probabilidade de ele ser um
> cachorro preto é de 1/8. *
> *16) Se um animal for escolhido ao acaso, a probabilidade de ele ser um
> gato malhado é de 1/16.   *
>
> --
> Esta mensagem foi verificada pelo sistema de antivírus e
> acredita-se estar livre de perigo.

-- 
Esta mensagem foi verificada pelo sistema de antiv�rus e
 acredita-se estar livre de perigo.



[obm-l] Re: Bóson de Higgs

2022-05-30 Por tôpico Israel Meireles Chrisostomo
considere o universo newtoniano.

Em seg., 30 de mai. de 2022 às 16:06, Israel Meireles Chrisostomo <
israelmchrisost...@gmail.com> escreveu:

> Olá pessoal, desculpe-me te incomodar.Vim propor um desafio para vcs(a
> challenge for you):provar usando o Princípio da Boa Ordenação que o
> universo possui mais do que uma
> partícula fundamental, isto é, uma partícula que compõe tudo o que há no
> universo.
> --
> Israel Meireles Chrisostomo
>


-- 
Israel Meireles Chrisostomo

-- 
Esta mensagem foi verificada pelo sistema de antiv�rus e
 acredita-se estar livre de perigo.



[obm-l] Re: [obm-l] Série

2022-05-08 Por tôpico Anderson Torres
Em sex., 29 de abr. de 2022 às 23:09, Israel Meireles Chrisostomo
 escreveu:
>
> Alguém aí consegue calcular o limite contida no arquivo desse link logo 
> abaixo?
> https://www.overleaf.com/project/624ee701e9cd2d14986e6f48
>

Link indisponível.

obrigado...

> --
> Israel Meireles Chrisostomo
>
> --
> Esta mensagem foi verificada pelo sistema de antivírus e
> acredita-se estar livre de perigo.

-- 
Esta mensagem foi verificada pelo sistema de antiv�rus e
 acredita-se estar livre de perigo.


=
Instru��es para entrar na lista, sair da lista e usar a lista em
http://www.mat.puc-rio.br/~obmlistas/obm-l.html
=


[obm-l] Re: [obm-l] Sistema de equações lineares

2022-04-18 Por tôpico Anderson Torres
Para de spammar

Em dom., 17 de abr. de 2022 às 01:16, Felippe Coulbert Balbi
 escreveu:
>
> Eu tenho um sistema de equações lineares com 12 variaveis: x1, x2,...,x12. 
> Essas variaveis assumem valor somente no conjunto {0, 1, 1/2, 1/3}.
>
> Eu tenho 8 equações
>
> 4 equações é um sistema linear que pode ser escrito como:
>
> Ax= b
>
> A é uma matriz de 4 linhas e 12 colunas, b é uma matriz de 4 linhas e 1 coluna
>
> As outras 4 equações são:
>
> x1+x2+x3 = 1
>
> x4+x5+x6 = 1
>
> x7+x8+x9 = 1
>
> x10+x11+x12 = 1
>
> Para quais valores de A e b, esse sistema tem solucao? Quando a solucao desse 
> sistema é unica?
>
> Grato,
> Felippe
>
> --
> Esta mensagem foi verificada pelo sistema de antivírus e
> acredita-se estar livre de perigo.

-- 
Esta mensagem foi verificada pelo sistema de antiv�rus e
 acredita-se estar livre de perigo.


=
Instru��es para entrar na lista, sair da lista e usar a lista em
http://www.mat.puc-rio.br/~obmlistas/obm-l.html
=


[obm-l] Re: [obm-l] Dúvida e ajuda.

2022-04-10 Por tôpico Anderson Torres
Em sex., 8 de abr. de 2022 às 11:17, Pedro José  escreveu:
>
> Bom dia!
> Posso concluir que um número representado por uma infinidade de algarismos 
> decimais é racional se e somente se tem um período de repetições desses 
> algarismos?
> A ida é fácil se tiver o período é racional.
> Já a volta não sei se é verdade e se for há como provar?

Acho que dá para fazer isso mais algoritmicamente.

Um número da forma

0,(A)

onde A é um período de k dígitos (por óbvio, zeros à esquerda são
permitidos, como em 0,010101010101...) é essencialmente um racional da
forma A/..9 com k noves - ou melhor escrevendo, (A/(10^k-1)).

Já números da forma 0,B(A) onde B tem m dígitos são a mesma coisa que
10^(-m)*(B+A/(10^k-1)), o que, após simplificar, dá (maçaroca
qualquer)/(10^m*(10^k-1)).

Qualquer racional por definição é da forma p/q com q natural. Bastaria
demonstrar que todo natural q tem um múltiplo da forma
(10^m*(10^k-1)), o que sai de uma aplicação de Euler-Fermat ou mesmo
de casa do pombo.

(Aliás, quem foi o BR que traduziu "princípio do escaninho" para
"princípio de casa de pombo"?)

>
> Meu objetivo primário é saber se:
> 0,123456789112233445566778899111222333444555666777888999... é racional. As 
> reticências se referem ao aumento de mais um algarismo repetido a cada 
> sequência, ou seja a primeira aparição de 1 será 1, a 2a 11 a 3a 111 e assim 
> sucessivamente, o mesmo vale para os demais algarismos.
>
> Alguém poderia me ajudar?
> Grato,
> PJMS
>
> --
> Esta mensagem foi verificada pelo sistema de antivírus e
> acredita-se estar livre de perigo.

-- 
Esta mensagem foi verificada pelo sistema de antiv�rus e
 acredita-se estar livre de perigo.


=
Instru��es para entrar na lista, sair da lista e usar a lista em
http://www.mat.puc-rio.br/~obmlistas/obm-l.html
=


[obm-l] Re: [obm-l] Re: [obm-l] Dúvida e ajuda.

2022-04-08 Por tôpico Pedro José
Grato a todos!
Já, já tenho de voltar ao trabalho.
Depois dou uma olhada.
Mas achei a demonstração usando casa de pombos, simples e prática.
Já que tem de haver um p/q com pp temos w=x+p/q,
onde x é a parte inteira de w/q, então pq e os restos só podem q-1, uma hora tem de
repetir e aí volta a sequência.
Mas saindo do trabalho dou uma olhada.
Mais uma vez, minha gratidão.

Cordialmente,
PJMS



Em sex., 8 de abr. de 2022 às 13:02, Claudio Buffara <
claudio.buff...@gmail.com> escreveu:

> A volta é fácil também: ao calcular a representação decimal de a/b (a e b
> naturais), nas divisões sucessivas por b só existem b-1 restos possíveis
> (resto = 0 em alguma etapa implica numa decimal finita) e, portanto, após
> não mais do que b-1 divisões, um resto vai se repetir, marcando o início de
> um novo período na representação decimal.
>
> Agora, suponha que  X =
> 0,123456789112233445566778899111222333444555666777888999... seja racional.
> Então existirão n e p naturais tais que, a partir da n-ésima casa decimal
> (1/10^n), os algarismos de X vão se repetir numa sequência com período p.
>
> Mas, pela lei de formação de X, vai existir uma sequência de n+p+1
> algarismos iguais a 1, e esta sequência vai começar após a n-ésima casa
> decimal.
> Ou seja, a sequência vai estar incluída na parte periódica da
> representação decimal de X.
> Mas como o período é p, isso implica que a parte periódica teria que
> ser 111..11 (p algarismos 1) ==> contradição à lei de formação de X.
>
> []s,
> Claudio.
>
>
> On Fri, Apr 8, 2022 at 11:17 AM Pedro José  wrote:
>
>> Bom dia!
>> Posso concluir que um número representado por uma infinidade de
>> algarismos decimais é racional se e somente se tem um período de repetições
>> desses algarismos?
>> A ida é fácil se tiver o período é racional.
>> Já a volta não sei se é verdade e se for há como provar?
>>
>> Meu objetivo primário é saber se:
>> 0,123456789112233445566778899111222333444555666777888999... é racional.
>> As reticências se referem ao aumento de mais um algarismo repetido a cada
>> sequência, ou seja a primeira aparição de 1 será 1, a 2a 11 a 3a 111 e
>> assim sucessivamente, o mesmo vale para os demais algarismos.
>>
>> Alguém poderia me ajudar?
>> Grato,
>> PJMS
>>
>> --
>> Esta mensagem foi verificada pelo sistema de antivírus e
>> acredita-se estar livre de perigo.
>
>
> --
> Esta mensagem foi verificada pelo sistema de antivírus e
> acredita-se estar livre de perigo.

-- 
Esta mensagem foi verificada pelo sistema de antiv�rus e
 acredita-se estar livre de perigo.



[obm-l] Re: [obm-l] Dúvida e ajuda.

2022-04-08 Por tôpico Claudio Buffara
A volta é fácil também: ao calcular a representação decimal de a/b (a e b
naturais), nas divisões sucessivas por b só existem b-1 restos possíveis
(resto = 0 em alguma etapa implica numa decimal finita) e, portanto, após
não mais do que b-1 divisões, um resto vai se repetir, marcando o início de
um novo período na representação decimal.

Agora, suponha que  X =
0,123456789112233445566778899111222333444555666777888999... seja racional.
Então existirão n e p naturais tais que, a partir da n-ésima casa decimal
(1/10^n), os algarismos de X vão se repetir numa sequência com período p.

Mas, pela lei de formação de X, vai existir uma sequência de n+p+1
algarismos iguais a 1, e esta sequência vai começar após a n-ésima casa
decimal.
Ou seja, a sequência vai estar incluída na parte periódica da representação
decimal de X.
Mas como o período é p, isso implica que a parte periódica teria que
ser 111..11 (p algarismos 1) ==> contradição à lei de formação de X.

[]s,
Claudio.


On Fri, Apr 8, 2022 at 11:17 AM Pedro José  wrote:

> Bom dia!
> Posso concluir que um número representado por uma infinidade de algarismos
> decimais é racional se e somente se tem um período de repetições desses
> algarismos?
> A ida é fácil se tiver o período é racional.
> Já a volta não sei se é verdade e se for há como provar?
>
> Meu objetivo primário é saber se:
> 0,123456789112233445566778899111222333444555666777888999... é racional. As
> reticências se referem ao aumento de mais um algarismo repetido a cada
> sequência, ou seja a primeira aparição de 1 será 1, a 2a 11 a 3a 111 e
> assim sucessivamente, o mesmo vale para os demais algarismos.
>
> Alguém poderia me ajudar?
> Grato,
> PJMS
>
> --
> Esta mensagem foi verificada pelo sistema de antivírus e
> acredita-se estar livre de perigo.

-- 
Esta mensagem foi verificada pelo sistema de antiv�rus e
 acredita-se estar livre de perigo.



[obm-l] Re: [obm-l] Dúvida e ajuda.

2022-04-08 Por tôpico Caio Costa
Para a volta considere a repetição dividida por 9...9 onde há o mesmo
número de algarismos na repetição e no denominador, incluindo possíveis
zeros à esquerda.

Exemplo

0.3520012001200120012...

= 0.352 + (0012/)/1000

Em sex., 8 de abr. de 2022 11:17, Pedro José  escreveu:

> Bom dia!
> Posso concluir que um número representado por uma infinidade de algarismos
> decimais é racional se e somente se tem um período de repetições desses
> algarismos?
> A ida é fácil se tiver o período é racional.
> Já a volta não sei se é verdade e se for há como provar?
>
> Meu objetivo primário é saber se:
> 0,123456789112233445566778899111222333444555666777888999... é racional. As
> reticências se referem ao aumento de mais um algarismo repetido a cada
> sequência, ou seja a primeira aparição de 1 será 1, a 2a 11 a 3a 111 e
> assim sucessivamente, o mesmo vale para os demais algarismos.
>
> Alguém poderia me ajudar?
> Grato,
> PJMS
>
> --
> Esta mensagem foi verificada pelo sistema de antivírus e
> acredita-se estar livre de perigo.

-- 
Esta mensagem foi verificada pelo sistema de antiv�rus e
 acredita-se estar livre de perigo.



[obm-l] Re: Dúvida e ajuda.

2022-04-08 Por tôpico Pedro José
Bom dia!
Última forma!
Achei uma demonstração simples e bela, usando casa dos pombos. Uma hora
haverá de ter repetição, portanto, tem que ter um grupamento de dígitos que
se repita caso seja uma série infinita de algarismos decimais.
Portanto o número é irracional.
Grato!
PJMS

Em sex., 8 de abr. de 2022 às 11:06, Pedro José 
escreveu:

> Bom dia!
> Posso concluir que um número representado por uma infinidade de algarismos
> decimais é racional se e somente se tem um período de repetições desses
> algarismos?
> A ida é fácil se tiver o período é racional.
> Já a volta não sei se é verdade e se for há como provar?
>
> Meu objetivo primário é saber se:
> 0,123456789112233445566778899111222333444555666777888999... é racional. As
> reticências se referem ao aumento de mais um algarismo repetido a cada
> sequência, ou seja a primeira aparição de 1 será 1, a 2a 11 a 3a 111 e
> assim sucessivamente, o mesmo vale para os demais algarismos.
>
> Alguém poderia me ajudar?
> Grato,
> PJMS
>

-- 
Esta mensagem foi verificada pelo sistema de antiv�rus e
 acredita-se estar livre de perigo.



[obm-l] Re: Questão de probabilidade

2022-03-16 Por tôpico Professor Vanderlei Nemitz
Hummm...
Acho que descobri o que o autor pensou.
Parece que 6 casos, distribuindo P, P, B, B, B, B, M (2 pretos, 4 brancos e
1 malhado) entre 3 gatos e 4 cachorros.
Mas...esse espaco amostral é equiprovável???

Em qua., 16 de mar. de 2022 07:58, Professor Vanderlei Nemitz <
vanderma...@gmail.com> escreveu:

> Bom dia!
> Na questão a seguir, do vestibular da UEM, penso que o espaço amostral tem
> 105 elementos, pois um cachorro é preto (desconsideramos esse). Porém, com
> esse pensamento, não consigo obter o gabarito, que diz que 02 e 16 são
> corretas.
> Alguém poderia ajudar?
> Muito obrigado!
>
> *Em um pet shop há 3 gatos e 5 cães. Sabemos que 3 desses animais são
> pretos, 4 são brancos e 1 é malhado. Além disso, pelo menos 1 cachorro é
> preto. Assinale o que for correto. *
> *01) A probabilidade de haver exatamente 1 cachorro preto é de 1/6. *
> *02) A probabilidade de haver pelo menos 1 gato branco e pelo menos 2
> cachorros brancos é de 2/3.*
> *04) A probabilidade de haver um cachorro malhado é maior do que a
> probabilidade de haver um gato malhado. *
> *08) Se um animal for escolhido ao acaso, a probabilidade de ele ser um
> cachorro preto é de 1/8. *
> *16) Se um animal for escolhido ao acaso, a probabilidade de ele ser um
> gato malhado é de 1/16.   *
>

-- 
Esta mensagem foi verificada pelo sistema de antiv�rus e
 acredita-se estar livre de perigo.



[obm-l] Re: Desigualdades integral

2022-01-31 Por tôpico Israel Meireles Chrisostomo
Esqueçam esse post eu me confundi, to na madrugada inteira tentando provar
a irracionalida de pi e acho que agora finalmente eu consegui, por isso
minha mente está cansada

Em seg., 31 de jan. de 2022 13:54, Israel Meireles Chrisostomo <
israelmchrisost...@gmail.com> escreveu:

> Olá pessoal.uma dúvida surgiu(desculpem minha ignorância) é o seguinte eu
> tenho uma função f que no ponto a é zero e no b também.existe um k fixo tal
> q f até b de f é menor que f(b)- f(a) em outras palavras a integral é nula?por
> favor me expliquem o que está acontecendo.
>

-- 
Esta mensagem foi verificada pelo sistema de antiv�rus e
 acredita-se estar livre de perigo.



[obm-l] Re: Integral

2022-01-31 Por tôpico Israel Meireles Chrisostomo
Acho que isso deve ter alguma coisa

Em seg., 31 de jan. de 2022 09:35, Israel Meireles Chrisostomo <
israelmchrisost...@gmail.com> escreveu:

> existe uma diferença entre a integral do contra exemplo com a que eu estou
> estudando: na integral do contra exemplo a "variável" do limite coincide
> com a variável que está sendo integrada
>
> Em seg., 31 de jan. de 2022 09:33, Israel Meireles Chrisostomo <
> israelmchrisost...@gmail.com> escreveu:
>
>> Me disseram que A função f(x)=1/x tende a zero quando x tende ao infinito
>> mas a integral de 1/x é o logaritmo natural de x, Ln(x), que claramente não
>> tende a zero.
>>
>> Em seg., 31 de jan. de 2022 08:56, Israel Meireles Chrisostomo <
>> israelmchrisost...@gmail.com> escreveu:
>>
>>> Eu estou interessado na seguinte integral int 0 até 1 de (n choose
>>> k)t^{k}*c^{n}/n! dt Com n tendendo ao infinito
>>>
>>> Em seg., 31 de jan. de 2022 05:07, Israel Meireles Chrisostomo <
>>> israelmchrisost...@gmail.com> escreveu:
>>>
 Olá pessoal.Eu estava resolvendo um problema e me deparei com uma
 dúvida.A dúvida é a seguinte: a integral de uma função que tende a zero é
 igual a zero?

>>>

-- 
Esta mensagem foi verificada pelo sistema de antiv�rus e
 acredita-se estar livre de perigo.



[obm-l] Re: Integral

2022-01-31 Por tôpico Israel Meireles Chrisostomo
existe uma diferença entre a integral do contra exemplo com a que eu estou
estudando: na integral do contra exemplo a "variável" do limite coincide
com a variável que está sendo integrada

Em seg., 31 de jan. de 2022 09:33, Israel Meireles Chrisostomo <
israelmchrisost...@gmail.com> escreveu:

> Me disseram que A função f(x)=1/x tende a zero quando x tende ao infinito
> mas a integral de 1/x é o logaritmo natural de x, Ln(x), que claramente não
> tende a zero.
>
> Em seg., 31 de jan. de 2022 08:56, Israel Meireles Chrisostomo <
> israelmchrisost...@gmail.com> escreveu:
>
>> Eu estou interessado na seguinte integral int 0 até 1 de (n choose
>> k)t^{k}*c^{n}/n! dt Com n tendendo ao infinito
>>
>> Em seg., 31 de jan. de 2022 05:07, Israel Meireles Chrisostomo <
>> israelmchrisost...@gmail.com> escreveu:
>>
>>> Olá pessoal.Eu estava resolvendo um problema e me deparei com uma
>>> dúvida.A dúvida é a seguinte: a integral de uma função que tende a zero é
>>> igual a zero?
>>>
>>

-- 
Esta mensagem foi verificada pelo sistema de antiv�rus e
 acredita-se estar livre de perigo.



[obm-l] Re: Integral

2022-01-31 Por tôpico Israel Meireles Chrisostomo
Me disseram que A função f(x)=1/x tende a zero quando x tende ao infinito
mas a integral de 1/x é o logaritmo natural de x, Ln(x), que claramente não
tende a zero.

Em seg., 31 de jan. de 2022 08:56, Israel Meireles Chrisostomo <
israelmchrisost...@gmail.com> escreveu:

> Eu estou interessado na seguinte integral int 0 até 1 de (n choose
> k)t^{k}*c^{n}/n! dt Com n tendendo ao infinito
>
> Em seg., 31 de jan. de 2022 05:07, Israel Meireles Chrisostomo <
> israelmchrisost...@gmail.com> escreveu:
>
>> Olá pessoal.Eu estava resolvendo um problema e me deparei com uma
>> dúvida.A dúvida é a seguinte: a integral de uma função que tende a zero é
>> igual a zero?
>>
>

-- 
Esta mensagem foi verificada pelo sistema de antiv�rus e
 acredita-se estar livre de perigo.



[obm-l] Re: Integral

2022-01-31 Por tôpico Israel Meireles Chrisostomo
Eu estou interessado na seguinte integral int 0 até 1 de (n choose
k)t^{k}*c^{n}/n! dt Com n tendendo ao infinito

Em seg., 31 de jan. de 2022 05:07, Israel Meireles Chrisostomo <
israelmchrisost...@gmail.com> escreveu:

> Olá pessoal.Eu estava resolvendo um problema e me deparei com uma dúvida.A
> dúvida é a seguinte: a integral de uma função que tende a zero é igual a
> zero?
>

-- 
Esta mensagem foi verificada pelo sistema de antiv�rus e
 acredita-se estar livre de perigo.



Re: [obm-l] Re: Polinomio

2022-01-29 Por tôpico Israel Meireles Chrisostomo
Muito obrigado pessoal

Em sáb., 29 de jan. de 2022 19:14, Artur Costa Steiner <
artur.costa.stei...@gmail.com> escreveu:

> Creio que vc se refere a polinômios reais.
>
> Se P tiver grau par positivo  então:
>   Se o coeficiente líder for positivo, P tem  um mínimo global. Se for
> negativo,  P tem um máximo global.
>
> Se P tiver  grau ímpar, P não tem mínimo nem máximo globais.
>
> Limitado inferior e superiormente, só se P for constante
>
> Artur
>
>
>
> Em sáb., 29 de jan. de 2022 às 18:41, Esdras Muniz <
> esdrasmunizm...@gmail.com> escreveu:
>
>> O único polinômio limitado é o constante.
>>
>> Em sáb, 29 de jan de 2022 14:03, Carlos Juarez <
>> carlosjuarezmart...@gmail.com> escreveu:
>>
>>> k=p(c)+1 não vale sempre?
>>>
>>> Em sáb, 29 de jan de 2022 09:27, Israel Meireles Chrisostomo <
>>> israelmchrisost...@gmail.com> escreveu:
>>>
 Desculpe me o que eu quis dizer é que dado um c real existe um k
 positivo tal que p(c)>>>
 Em sáb., 29 de jan. de 2022 09:12, Israel Meireles Chrisostomo <
 israelmchrisost...@gmail.com> escreveu:

> Olá pessoal.Eu gostaria de saber se um polinomio é limitado, isto é,
> dado P(x) existe um k positivo tal que P(x)

 --
 Esta mensagem foi verificada pelo sistema de antivírus e
 acredita-se estar livre de perigo.
>>>
>>>
>>> --
>>> Esta mensagem foi verificada pelo sistema de antivírus e
>>> acredita-se estar livre de perigo.
>>
>>
>> --
>> Esta mensagem foi verificada pelo sistema de antivírus e
>> acredita-se estar livre de perigo.
>
>
> --
> Esta mensagem foi verificada pelo sistema de antivírus e
> acredita-se estar livre de perigo.

-- 
Esta mensagem foi verificada pelo sistema de antiv�rus e
 acredita-se estar livre de perigo.



Re: [obm-l] Re: Polinomio

2022-01-29 Por tôpico Artur Costa Steiner
Creio que vc se refere a polinômios reais.

Se P tiver grau par positivo  então:
  Se o coeficiente líder for positivo, P tem  um mínimo global. Se for
negativo,  P tem um máximo global.

Se P tiver  grau ímpar, P não tem mínimo nem máximo globais.

Limitado inferior e superiormente, só se P for constante

Artur



Em sáb., 29 de jan. de 2022 às 18:41, Esdras Muniz <
esdrasmunizm...@gmail.com> escreveu:

> O único polinômio limitado é o constante.
>
> Em sáb, 29 de jan de 2022 14:03, Carlos Juarez <
> carlosjuarezmart...@gmail.com> escreveu:
>
>> k=p(c)+1 não vale sempre?
>>
>> Em sáb, 29 de jan de 2022 09:27, Israel Meireles Chrisostomo <
>> israelmchrisost...@gmail.com> escreveu:
>>
>>> Desculpe me o que eu quis dizer é que dado um c real existe um k
>>> positivo tal que p(c)>>
>>> Em sáb., 29 de jan. de 2022 09:12, Israel Meireles Chrisostomo <
>>> israelmchrisost...@gmail.com> escreveu:
>>>
 Olá pessoal.Eu gostaria de saber se um polinomio é limitado, isto é,
 dado P(x) existe um k positivo tal que P(x)>>>
>>>
>>> --
>>> Esta mensagem foi verificada pelo sistema de antivírus e
>>> acredita-se estar livre de perigo.
>>
>>
>> --
>> Esta mensagem foi verificada pelo sistema de antivírus e
>> acredita-se estar livre de perigo.
>
>
> --
> Esta mensagem foi verificada pelo sistema de antivírus e
> acredita-se estar livre de perigo.

-- 
Esta mensagem foi verificada pelo sistema de antiv�rus e
 acredita-se estar livre de perigo.



Re: [obm-l] Re: Polinomio

2022-01-29 Por tôpico Esdras Muniz
O único polinômio limitado é o constante.

Em sáb, 29 de jan de 2022 14:03, Carlos Juarez <
carlosjuarezmart...@gmail.com> escreveu:

> k=p(c)+1 não vale sempre?
>
> Em sáb, 29 de jan de 2022 09:27, Israel Meireles Chrisostomo <
> israelmchrisost...@gmail.com> escreveu:
>
>> Desculpe me o que eu quis dizer é que dado um c real existe um k positivo
>> tal que p(c)>
>> Em sáb., 29 de jan. de 2022 09:12, Israel Meireles Chrisostomo <
>> israelmchrisost...@gmail.com> escreveu:
>>
>>> Olá pessoal.Eu gostaria de saber se um polinomio é limitado, isto é,
>>> dado P(x) existe um k positivo tal que P(x)>>
>>
>> --
>> Esta mensagem foi verificada pelo sistema de antivírus e
>> acredita-se estar livre de perigo.
>
>
> --
> Esta mensagem foi verificada pelo sistema de antivírus e
> acredita-se estar livre de perigo.

-- 
Esta mensagem foi verificada pelo sistema de antiv�rus e
 acredita-se estar livre de perigo.



Re: [obm-l] Re: Polinomio

2022-01-29 Por tôpico Carlos Juarez
k=p(c)+1 não vale sempre?

Em sáb, 29 de jan de 2022 09:27, Israel Meireles Chrisostomo <
israelmchrisost...@gmail.com> escreveu:

> Desculpe me o que eu quis dizer é que dado um c real existe um k positivo
> tal que p(c)
> Em sáb., 29 de jan. de 2022 09:12, Israel Meireles Chrisostomo <
> israelmchrisost...@gmail.com> escreveu:
>
>> Olá pessoal.Eu gostaria de saber se um polinomio é limitado, isto é, dado
>> P(x) existe um k positivo tal que P(x)>
>
> --
> Esta mensagem foi verificada pelo sistema de antivírus e
> acredita-se estar livre de perigo.

-- 
Esta mensagem foi verificada pelo sistema de antiv�rus e
 acredita-se estar livre de perigo.



[obm-l] Re: Polinomio

2022-01-29 Por tôpico Israel Meireles Chrisostomo
Desculpe me o que eu quis dizer é que dado um c real existe um k positivo
tal que p(c) escreveu:

> Olá pessoal.Eu gostaria de saber se um polinomio é limitado, isto é, dado
> P(x) existe um k positivo tal que P(x)

-- 
Esta mensagem foi verificada pelo sistema de antiv�rus e
 acredita-se estar livre de perigo.



  1   2   3   4   5   6   7   8   9   10   >